SIGNIFICANT SAQs
SIGNIFICANT SAQs
SIGNIFICANT SAQs
SIGNIFICANT SAQs
TOPICS WITH SAQs
No. Topic SAQs/Comments Page
1 Enteric Fever SAQ-1
2 Malaria SAQ-3,4 (in 3, pharma treatment is
incomplete
3 Pinworm SAQ-6
4 Vaginal Discharge SAQ-8, 113
5 Hepatitis B/pregnancy & newborn SAQ-10, 11
hepatitis B
6 Herpes Zoster SAQ-12
7 Varicella Zoster SAQ-14
8 Tuberculosis SAQ-15, 16 (15 with pt related need to add
safety netting with community based TB
9 Mumps SAQ-18
10 Rabies vaccination/dog bite SAQ-152, 153
11 Swine flu SAQ-139 (rare SAQ)
12 Fever in adults SAQ-198
13 Persistent Cough SAQ-199
14 Allergic Rhinitis SAQ-29
15 Asthma in adults SAQ-22
16 Emphysema checkup SAQ-205
17 Adult Pneumonia SAQ-24, 215
18 Respiratory infections SAQ-199
19 Benign paroxysmal positional vertigo SAQ-27
20 Otitis Externa SAQ-28
21 Acute otitis media SAQ-35
22 Acute sinusitis SAQ-32
23 Red eye SAQ-41
24 Conjunctivitis SAQ-44
25 Squint SAQ-43
26 Dysphagia SAQ-227 (rare but important SAQ)
27 Emesis SAQ-50
28 Dysentry SAQ-56
29 Peptic disease or gastritis SAQ-51
30 Appendicitis SAQ-61
SIGNIFICANT TOPICS WITH SAQs 2
73 Dysmenorrhoea SAQ-120
74 Endometrial CA SAQ-122
75 Subfertility SAQ-133
76 Secondary amenorrhoea SAQ-130
77 Antenatal care SAQ-123
78 Anaemia of pregnancy SAQ-128, 129
79 Ectopic pregnancy SAQ-131, 132 (confusing, needs
discussion)
80 Ruptured ectopic pregnancy SAQ-134
81 Possible pre-eclampsia SAQ-135
82 Postpartum complications SAQ-136, 137
83 Postmenopausal checkup SAQ-203
84 Postmenopausal bleeding SAQ-124
85 Paeds post immunization SAQ-138
86 Fever in children SAQ-140
87 Epilepsy or seizures SAQ-141, 154, 157, 165
88 Diarrhoea in Paeds SAQ-142
89 Malabsorption SAQ-144
90 Rhinitis in Paeds SAQ-146
91 Paediatric Asthma SAQ-20
92 Viral Croup SAQ-25
93 Acute respiratory infection with SAQ-147
diarrhoea in paeds
94 Jaundice in paeds SAQ-148
95 Measles in paeds SAQ-149
96 Nocturnal enuresis SAQ-95
97 UTI in Paeds SAQ-150
98 Cystic fibrosis SAQ-151
99 Constipation in Paeds SAQ-233
100 TIA or stroke SAQ-155, 158, 160 (different scenarios)
101 Amaurosis fugax SAQ-190 (rare SAQ)
102 Headache SAQ-156, 159, 161, 163, 164
103 Bell’s palsy SAQ-162
104 Depression SAQ-166
105 Depression/Anxiety SAQ-169, 170
106 Anxiety with panic attacks SAQ-171
107 Posttraumatic stress disorder SAQ-172
108 Malingering SAQ-173
109 Ethics SAQ-175, 176
110 Ethics SAQ-177, 179
111 Ethics SAQ-180, 181
112 Ethics SAQ-182, 183
113 HIV SAQ-178
114 Tension pneumothorax SAQ-184
SIGNIFICANT TOPICS WITH SAQs 4
9
SIGNIFICANT TOPICS WITH SAQs 5
1.
An 18 years old young man presents with fever without rigors for a week. He has received
chloroquine and Augmentin given by his family physician for three days but there has been no
response. Tongue is coated and edge of liver and tip of spleen is palpable.
ANSWER
A
Diagnosis
Enteric fever
B
Investigations:
✓ CBC, ESR, LFTs, BUN, serum Cr, Blood and stool culture, Malarial parasite, urine dr
C
Management:
• Clinical evaluation with complete history and physical examination
✓ History; abdominal pain/ rashes/ bruises or petechiae/ epistaxis/ headache / joint pain or body
aches, back pain/ respiratory sx, fever continuous or intermittent
✓ Examination; general condition, appearance and mental status, vital signs, HEENT and neck,
Abdominal exam, CVS, Chest
• Investigations as discussed
• Non-pharmacological;
✓ Reassurance and explanation to the patient
✓ Provide related information and health education, esp. encourage individual and collective
hygiene
✓ Advise limitation of close contacts esp with susceptible individuals
✓ Rest, adequate hydration and soft nutritious diet
• Pharmacological;
✓ If the patient general condition is stable, can tolerate oral Rx and there are no s/sx of any
complications, start empirical treatment as early as possible if there is a high clinical suspicion of
Typhoid fever, advise;
✓ Antipyretics as needed (Acetaminophen PO 15mg/kg every 4-6 hours)
✓ Azithromycin 8-10 mg/kg OD PO 07 days (assuming Quinolone resistant) or cefixime 400mg od
for 14 days
• If the patient is clinically unstable, refer for admission and further management.
Red flags
Fits unconsciousness, dehydration, severe any bleeding
Followup
With lab results earlier if pt develops any red flag signs or condition worse visit ER
SIGNIFICANT TOPICS WITH SAQs 6
2.
A 24 year old office worker presents to your clinic with the complaints of fever ranging from 38 C
to 41 C since a week. He states that he shivers at the onset of fever, which then comes down to
normal with a great deal of sweating after every 2 days.
ANSWER
A
Diagnosis:
Most likely diagnosis relapsing quartan fever associated with p malariae malaria
B
Investigations:
1. MP Peripheral blood film (thick and thin)
2. CBC
3. urine dipstick
4. Blood culture and sensitivity
5. LFTs
6. USG
C
Pharmacological treatment;
Cholroquine sensitive areas quinine sulphate 600 mg po tds +doxycycline 100 mg po bd for 7 days
or
-Tab Artemether + Lumefantrine
Chloroquine 600 mg of base initially followed by 300 mg of base at 6, 24, 48 hours after the
intitial dose
ACT can also be used
Choloroquine resistant areas according to weight
If >35 kg tab artemether 80 mg +lumefantrine 480 mg (co artem ds)administer 24 tab over 3
days
Day 1 4 tab stat + 4 tab after 8 hrs
Days 2 & 3 4 tabs bd (morning and evening) take with food
Repeat the dose if patient vomits within 1-2 hrs
D
Symptomatic therapy
-Tepid sponging
-Adequate hydration fruit juices and clear soups preferred)
-Antipyretics (Acetaminophen 500 -1000 mg or 10-15mg/kg every 4-6 hours) as needed
SIGNIFICANT TOPICS WITH SAQs 7
3.
A 22 year old man presents with high grade fever, severe body aches and headaches for 3 days
associated with mild nausea. He denies vomiting or diarrhea. There is no cough or cold
symptoms.
On examination, temp is 39 C, HR 116/min, RR 18/min and BP 110/80 mmHg. Tongue is dry.
Neck is supple. Rest of the examination is normal.
ANSWER
A
Likely Diagnosis:
Malaria
B
Differential Diagnoses:
1. Viral fever
2. Typhoid
3. Urti
4. Hepatitis
5. Autoimmune
6. Lymphoma/leukemia
C
Investigations
• CBC – look for leukopenia; thrombocytopenia; elevated hematocrit
• LFTs – assessment of liver function
• Serum Albumin – look for low levels indicate plasma leakage
• Serology – (Dengue Ig M, usually negative in first 5 days of onset of sx)
• MP ict
• Blood culture
• Urine dipstick
D
MANAGEMENT
1. Detailed history, further important points in hx;
• Onset of fever , other associated sx like chills or rigors, rashes, jaundice
• Red flag sx; intolerable abdominal pain, mucosal bleeds, dyspnea, lethargy, restlessness,
impaired consciousness,
• Sick contacts
• Socioeconomic hx and living conditions
• Drug hx
2. Physical Examination,
• General and vital signs
SIGNIFICANT TOPICS WITH SAQs 8
4.
20 year old Fatima Bibi, resident of Pathan colony came to you with history of high grade fever,
intermittent, more in the evening and on alternate days for the last 5 days. She has been
treated by a local GP who gave her some antibiotics.
Her smear for MP shows plasmodium vivax. On enquiry, she has suffered from fever for the last
several months.
ANSWER
A
Management
Patient has relapsing malaria
1- Detailed hx;
• Assess severity of illness; hx of generalised weakness or lethargy, breathing difficulties, any
bleeding, fits or convulsions, persistent vomiting
• Hx and nature of previous febrile illnesses, medications used and compliance to Rx
• Socioeconomic hx and living conditions
• If married , pregnancy status
2- Examination;
• Look for warning signs of severe malaria;
• General condition, mental alertness and state of hydration, jaundice, clinical anemia-pallor
• Vitals, capillary refill, respiratory distress, acidotic breathing, SpO2 ,
• Any sign of systemic complications; esp for palpable spleen
3- Investigations;
• RBS
• CBC
• LFTs
• BUN and creatinine
• Urinalysis
• G6PD level
4- Pharmacological management: If clinically stable with no warning signs, start treatment;
• CHLOROQUINE (150 mg base tablets) PO OD for 3 days, give 4 tabs STAT, 2 tabs after 6
hour, 2 tabs daily on day 2 and 3
• PRIMAQUINE 20 mg 1 tab PO OD for 2 weeks
• ACETAMINOPHEN 500-1000 mg PO 4-6 hr PRN
5- Non pharmacological management
• Reassurance that the disease is not contagious
• Health education, explanation of the condition and instructions on the reduction and
prevention of mosquito bites esp from dusk to dawn
• Provide written information in the native language
• Discuss and educate about treatment, benefits and possible side effects and encourage
compliance
SIGNIFICANT TOPICS WITH SAQs 10
5.
A 30 year old male, laborer by profession, comes with a 3 day history of high grade fever,
headache and generalized body aches. He describes chills and rigor with this fever. He reports
having malaria 6 months back.
Blood malaria parasite report at this time shows plasmodium vivax infection.
A. Considering fever, headache and body aches what 3 differentials come to your mind apart
from malaria?
B. What clinical examination will you do to assess the severity of malaria?
C. Write a prescription for this patient (including generic name, dose and duration)
D. What advices will you give to prevent malaria in future?
ANSWER
A
Differential Diagnosis
1. Viral fever
2. Meningitis
3. Typhoid
B
Clinical Examination
Look for warning signs of severe malaria;
• General condition- look for extreme weakness and lethargy, pallor, jaundice, dehydration,
respiratory distress, acidotic breathing,
• Mental status -any Impairment of consciousness, check GCS
• Vitals signs- asses for circulatory shock, respiratory distress, decreasing SaO2,
• Any sign of systemic complications; esp for palpable spleen
• any active bleeding or petechiae
C
Prescription
• Chloroquine phosphate 1000mg po at 6, 24, 48 hr total dose-2500mg (250,500mg)
• PRIMAQUINE 20mg 1 tab PO OD for 2 weeks
• Artemether 40/240 and or 80/480 1 tab bd for 3 days
• ACETAMINOPHEN 500-1000mg PO 4-6 hr PRN
D
Advice to the patient
• Avoid mosquito bites esp from dusk to dawn, by using insect repellent, covering your arms
and legs, and using a mosquito bed net
• It is curative to complete your therapy without interruption esp the course of Primaquine
treatment to prevent relapse
• If you get vomiting within 1-2 hours, take another dose.
• Avoid travel to endemic areas
SIGNIFICANT TOPICS WITH SAQs 12
6.
A 22 years old mother brings her 2 years old daughter to your clinic. The family moved to Karachi 2
years ago from the Northern areas of Pakistan. The child is scratching her perianal area for the previous
one week. The child does not appear to be well kept and her clothes are dirty. There are two other
baby girls in the family
ANSWER
A
Most likely diagnosis:
Worm infestation - pinworm
B
Differential diagnosis
1- Scabies
2- Allergic contact Dermatitis
3-Psychogenic
C
Psychosocial factors:
1- Mother's lack of awareness, or suffering from stress or depression
2- Child neglect due to other children at home
3- Unhygienic practices among households
4- Child's thumb sucking habit
5- Child contact with infected persons
D
Management
1-Complete history
• known allergies
• hx of itching on other body sites
• any hx of diarrhoea
• is the child irritable or restless esp at night or not able to sleep properly
• any health related issues in other households
• hygienic practices
• socioeconomic hx
2- Physical examination
• Child general condition, hygiene, visible dirt/ skin rashes on other sites/ status of clothes
• Vitals, pallor/ signs of malnutrition
• Local perianal examination:
o erythema/ skin tags/ fecal soilage/
3- Investigations:
• The adhesive tape test for 3 consecutive days
4- Non pharmacological management
SIGNIFICANT TOPICS WITH SAQs 13
5- Pharmacological management
• Anti-Helminthic therapy
o Mebendazole 100mg 2 doses 2weeks apart
o Treatment for all household members.
SIGNIFICANT TOPICS WITH SAQs 14
7.
A 37 year old married woman attends your clinic with four days history of foul smelling, greenish frothy
discharge. There is no fever or abdominal pain. On pelvic examination, petechiae are found on cervix
and vagina is inflamed with copious discharge. There is no cervical motion tenderness.
ANSWER
A
Differential diagnosis:
1- Trichomoniasis
2- Chlamydia trachomatis inf
B
Management:
1- Complete history;
• missed period or confirmed pregnancy
• nature of discharge esp quantity, quality and odour
• any presence of blood
• any relation with menstruation
• associated local sx like pruritus, perineal burning/ soreness
• urinary sx
• dyspareunia
• Pelvic pain/ low back pain or abdominal pain
• associated systemic sx like fever, nausea/vomiting, rashes, joint pain
• use of IUCDS
• drug hx esp OCPs
• any frequent use of topical vaginal preparations
• any sx in partner
• previous hx of any STIs
• result of Pap smear if done before
2- Physical examination:
• Vaginal speculum examination for directly visualising vagina and cervix, for taking high
vaginal swab
• bimanual exam
3- Investigations:
• pH test
• HVS for microscopy, culture and sensitivity
• full STI testing
4- Non pharmacological management:
• Explanation and pt education
• Advise to avoid sexual intercourse until completion of therapy and absence of sx for the pt
and partner
5- Pharmacological:
SIGNIFICANT TOPICS WITH SAQs 15
8.
A 30 year old married lady comes to your clinic complaining of excessive discharge from her vagina,
which is quite concerning for her.
A. Enlist 6 relevant points you would ask in the history to evaluate her condition.
B. Enlist 4 causes you would consider in this patient.
C. Enlist 2 office based tests which are performed in this condition.
D. Outline drug treatment for three differentials listed in “Answer B”.
ANSWER
A
Relevant Points in History
1) any history of fever
2) missed period or confirmed pregnancy
3) onset, duration and nature of discharge esp color, quantity, quality and odour
4) any presence of blood
5) associated local sx like pruritus, perineal burning/ soreness
6) dyspareunia
7) pelvic pain/ low back pain or abdominal pain
B
Differentials
1- Trichomoniasis
2-Bacterial vaginosis
3-Vulvovaginal candidiasis
4- Chlamydia trachomatis
C
Point of care tests
1- pH testing of discharge
2- Whiff test
D
Pharmacological management
1- Trichomoniasis
• METRONIDAZOLE/ TINIDAZOLE 2g PO single dose (OR)
• METRONIDAZOLE 500mg PO BID for 7 days
2- Bacterial Vaginosis
• METRONIDAZOLE 500mg PO BID for 7 days
3- Vulvovaginal Candidiasis
• FLUCONAZOLE 150mg PO single dose (OR)
• MICONAZOLE Vaginal suppositories 400mg QHS 3 days
SIGNIFICANT TOPICS WITH SAQs 17
9.
A married young woman aged 20 years reports to your clinic with the complaints of foul smelling and
smoothly vaginal discharge and itching since 7 days.
A. What questions would you want to ask to reach the provisional diagnosis?
B. What features and signs would you look for in the pelvic examination?>
C. What preventive measures would you advise?
ANSWER
A
History:
1- are you pregnant or have u missed ur periods?
2- tell me more about the discharge
• is it profuse or scanty ?
• how does it look like, color and consistency?
• is it offensive.
• is there any specific odour of the discharge that u can relate to ?
• is it blood tinged ?
❖ If it is offensive: think about BV, TV or foreign body
3- are you having any other local sx ?associated with discharge ?
• any vulvar itching ?
• any vulvar or perineal -pain/ soreness / burning
• do u feel pain during intercourse ?
• any hx of postcoital bleeding
❖ If pruritus: think about VVC, TV,
❖ If painful: Genital herpes, BV
4- are there any other associated complaints?
• are u feeling generally unwell
• any fever
• any abdominal pain ?
• pelvic pain ?
• low back pain ?
• nausea or vomiting
• any pain or burning with urination ?
• any pain during defecation ?
• any intermenstrual bleeding ?
5- sexual history
• are u sexually active ?
• any sx in partner
• do u use any contraceptives ?
• have u been diagnosed with any STI previously, if yes what was the outcome?
B
Pelvic Examination
1- vaginal Speculum exam;
SIGNIFICANT TOPICS WITH SAQs 18
10.
A 40 year old gentleman presented with a report showing Hepatitis B surface antigen positive done 15
days ago. He is otherwise asymptomatic.
ANSWER
A
Risk factors to be assessed;
• birth to an infected mother
• Household exposure to infected or symptomatic liver disease cases
• sharing items such as razors, toothbrushes, needles or syringes
• Exposure to sharp instruments or needles at work or home
• contact with blood or open sores of an infected person
• sexual contact with an infected or susceptible person
• any practice of body-piercing, skin tattoos or acupuncture therapy
• blood transfusions/ organ transplantation
• iv drug users
• health care professionals
B
Further investigations in HBsAg positive cases;
• Liver profile
• Serum albumin, total protein
• Serum creatinine
• CBC
• PT/INR
• HBV serology including HBeAg and anti-HBe, HBV DNA, Anti HBc IgM
• HIV Ab, Anti HAV IgM and Anti HCV
C
Management
1- Detailed hx
• exposure to blood or blood products,
• injection drug abuse,
• sexual exposure,
• occupational exposure
• family history of hepatitis B
• recent history of medical procedures particularly if done in outpatient
• Exposure to immigrants from countries with a high prevalence of HBV infection
• detailed medication history including past and present medications with specific attention
to over-the-counter, herbal and alternative medications
• personal habits regarding smoking, alcohol consumption, drug abuse and sexual activity
• history of previous treatment for HBV infection and complete examination
SIGNIFICANT TOPICS WITH SAQs 20
11.
A 25 years old woman who is hepatitis B surface antigen positive, has just delivered her first child three
hours ago.
A. What management steps need to be taken to prevent the neonate from acquiring hepatitis B?
B. What advice should be given to the mother regarding breast feeding of her infant?
ANSWER
A
Management
1- Detailed history and essential systemic examination.
2- Immediate immunoprophylactic management for the newborn;
• Administer HBIG (0.5 mL, IM) and single-antigen hepatitis B vaccine (0.5 mL, IM) at separate
injection sites within 12 hours of birth.
3- Nonpharmacological steps in prevention;
• Reassure mother and provide psychological support and extensive counselling related to her
illness and the care of newborn esp. as she is the first time mother.
• Document the details of vaccination in the newborn’s medical record and give it to the mother
• Explain the importance of completing the hepatitis B vaccine series to protect her baby.
• Remind her to bring the record card each time her baby sees a provider.
• Notify the local or concerned health department of the infant’s birth with the details of
immunoprophylaxis given.
• Provide instructions to the mother, tell her;
o She may safely breastfeed her infant.
o it is critically important for the protection of her baby’s health that the baby receives the
full hepatitis B vaccine series on the recommended schedule.
o the blood tests (HBsAg and antibody to hepatitis B surface antigen [anti-HBs]) need to be
drawn from the baby 1–2 months after completion of the 3- or 4-dose hepatitis B vaccine
series and also no earlier than 9–12 months of age to determine if the child developed a
protective immune response to vaccination or needs additional management.
o about modes of HBV transmission and the need for testing and vaccination of susceptible
household, sexual, and needle-sharing contacts
o she needs to have a medical evaluation for chronic hepatitis B, including an assessment of
whether she is a candidate for antiviral treatment
B
Breastfeeding advice to mother;
1- Reassure her and provide health education
2- Provide written information in native language regarding importance of breastfeeding.
3- Advise that it is safe to breast feed her baby and it doesn't pose any additional risk to the
baby or to herself
4- Inform that she should take good care of her nipples to avoid cracking and bleeding.
5- Safety netting
SIGNIFICANT TOPICS WITH SAQs 22
12.
A 50 years old lady with no comorbids presents to you with a history of painful rash on the trunk. On
examination, the rash is vesicular with an erythematous base and has a dermatomal distribution.
ANSWER
A
Shingles/ Herpes Zoster
B
Management
1- Detailed history
• Rash:
o Onset, acute or chronic
o Duration
o Distribution, rashes on other body sites
o evolution of rash
o previous similar rash
o Contact with at-risk/ susceptible individuals
• Associated sx
o itching , pain or other abnormal skin sensations
o Constitutional symptoms -fever/fatiguability/ headache
• Past hx
o hx of any known immunocompromised disorder
o if married reproductive and sexual hx
o drug hx
o immunization, recent vaccination
o social hx
2- Physical examination
• General appearance, vital signs
• examination of rash;
o distribution-localised or generalised, pattern, appearance, characteristics, active
discharge, tenderness
3- Investigations
• the diagnosis is mostly clinical
• confirm pregnancy if premenopausal and sexually active
• assessment of renal function if antiviral Rx is indicated.
4- Non pharmacological management
• Reassurance, explanation and health education
• Inform the pt about probable duration of rash that is usually 14-21 days
• Encourage good hygiene and proper skincare.
• Adequate hydration
• Inform the pt for potential of dissemination that can be suspected if there is constitutional
illness and/or spreading rash
SIGNIFICANT TOPICS WITH SAQs 23
13.
A 70 year old lady presents with unilateral vesicular, painful rash on her chest.
ANSWER
A
Detailed history
1- how did the rash appear and since how many days ?
2- any trigger / contact allergies
3- how is it distributed, is the rash covering other body sites ?
4- how did the rash evolve / change from before ?
5- do u have other sx associated with rash like localised itching , pain or other abnormal skin
sensations ?
6- any occular sx
7- Do u have any fever, lethargy or fatiguability
8- Do u have any susceptible household contacts ?
9- Any known comorbs/ immunocompromised disorder / malignancy
10- Are u currently using any Rx or drugs esp prolong use of steroids
11- have u received any recent vaccination
12- Any known allergies, drug or nondrug related
B
Diagnosis:
Shingles/ Herpes Zoster
C
Management
1- Detailed history as above
2- Physical examination
• General appearance, vital signs
• examination of rash;
o distribution-localised or generalised, pattern, appearance, characteristics, active
discharge, tenderness
3- Investigations
• the diagnosis is mostly clinical
• assessment of renal function if antiviral Rx is indicated.
4- Non pharmacological management
• Reassurance, explanation and health education
• Inform the pt about probable duration of rash that is usually 14-21 days
• Encourage good hygiene and proper skincare.
• Adequate hydration
• Inform the pt for potential of dissemination that can be suspected if there is constitutional
illness and/or spreading rash
• Inform about postherpetic neuralgia
SIGNIFICANT TOPICS WITH SAQs 25
14.
A mother brings her 4 years old child because she has noticed eruptions on the child’s skin since
yesterday. She also reports that the child has a mild fever. On examination, the child looks well. He has
multiple vesicular crop like eruptions on the trunk and face with some lesion in the mouth.
ANSWER
A
Diagnosis
Varicella / Chicken pox
B
Management
1- Detailed history
• prev hx of varicella infection
• known hx of any immunosuppressive disorder
• fever concurrent with rash
• any specific pattern of rash/ involvement of other body sites as hands and feet
• evolution of rash
• assoc sx as localised pain/ pruritus/ purulent discharge from the rash/headache/ respiratory
sx
• appetite issues
• sick contacts
• susceptible or at-risk contacts at home
• Immunisation status
• known allergies
2- Physical exam
• General appearance
• vitals
• appearance of rash
• involvement of eye
3- Investigations
• diagnosis based on clinical presentation
4- Non pharmacological management
• explore and adress mother's ideas, concerns and expectations regarding the child's illness
• Reassurance, health education and advice to the mother
o inform that its course is mostly benign and the immunity is usually lifelong
o Child should remain at home, away from susceptible or at-risk persons until no new
eruptions and all vesicles crusted
o Child may return to school when lesions have scabbed.
o General dietary, hydration and hygiene advice
o Advise nail clipping to prevent scarring and secondary infection from itching
o Keep the lesions cool
• topical preparations for pruritus - Calamine lotion
SIGNIFICANT TOPICS WITH SAQs 27
5- Pharmacological
• Acetaminophen oral as antipyretic / analgesic
• avoid ASPIRIN/ IBUPROFEN to avoid risk of developing Reye’s syndrome.
• Antihistamines for itching
• Consider Acyclovir if immunosuppressed
6- Follow up as needed
SIGNIFICANT TOPICS WITH SAQs 28
15.
A 35 year old gentleman, clerk by profession, presents with 2 months’ history of low grade fever, cough
and weight loss. On examination, his temperature is 99.4 F and weight is 60 kg. Bronchial breath
sounds in right upper lobe. Sputum AFB positive.
ANSWER
A
Most likely diagnosis:
Active pulmonary tuberculosis
B
Non pharmacological management
1- Reassurance and extensive patient education;
• if available provide written information related to the disease in native language
2- Advice and instructions;
• measures to prevent spread
• if possible stay at home
• wear surgical mask when go outside or near other persons/ contacts esp younger children
and immunocompromised
• general advice on good healthy diet (high protein diet) and hydration
• Reinforce compliance to treatment
• provide awareness regarding possible side effects of medications and when to seek medical
help
3- Notification to local health / TB control dept and plan for contact investigation and testing in
collaboration
• Screening for all persons who came into close contact with the patient before initiation of
therapy should be screened for TB
4- Follow up plan
• to assess improvement in clinical condition, adherence to treatment and evaluation for
medication side effects at least once every month
• for sputum microscopy and culture at least once a month until 2 consecutive negative
specimens are obtained.
C
Anti TB treatment
1- Initial phase 2 months (avg. 8 weeks)
i. ISONIAZID 300 mg (5 mg/kg) PO QD
(Side effect: Neuropathy, hepatitis) AND
ii. PYRIDOXINE 25 mg PO QD AND
iii. RIFAMPICIN 600 mg (10 mg / kg) PO QD
(Side effect: hepatotoxicity, orange discolouration of body fluids)
iv. PYRAZINAMIDE 1500 mg (25 mg/kg) PO QD
(Side effect: hyperuricemia or symptomatic gout)
SIGNIFICANT TOPICS WITH SAQs 29
16.
A boarding school mistress comes to you with the concern that a child at her school has recently
been diagnosed with pulmonary TB, 1 week ago.
ANSWER
A
Further questions to ask;
1- how and where is the diagnosis confirmed?
2- has it been reported to local public health dept ?
3- are there any suspicious sx in close contacts or other children
4- what is the immunisation status of children esp BCG
5- how appropriate are the living conditions in the school
6- how often is periodic health check up performed for students
B
Important steps/measures to prevent spread of TB
1- exclude the child diagnosed with active pulmonary TB from school until completion of at least 2
weeks of treatment
2- notification of the case to local health dept
3- source tracing and screening of contacts esp any immunocompromised in collaboration with
local TB control dept
4- thorough health education and awareness of all school staff along with parents / guardian
5- Follow up for the affected child regarding adherence to treatment and clinical improvement.
C
Screening for TB
1- TST - tuberculin skin test (testing skin induration after PPD injection)
2- interferon γ release assay -IGRA
3- Sputum for microscopy / AFB
- three consecutive samples 8-24 hour interval (1 at least early morning)
4- CXR
D
1- Yes this child needs isolation and follow up for treatment, he can return to school as early as
after 2 weeks of successful completion of Rx and when he starts feeling improvement.
2- No vaccination is indicated for the child.
SIGNIFICANT TOPICS WITH SAQs 31
17.
A 25 year old man is suffering from cough with sputum, weight loss and fever since 1 month. He
has not had TB before. His sputum AFB smear is positive. His weight is 50 kg.
A. Write a prescription for this patient with the name of individual drugs and their dosage.
B. List 4 non-pharmacological advices to this patient.
ANSWER
A
Anti TB treatment
1- Initial phase 2 months (avg 8weeks)
i. RIFAMPICIN 500mg (10mg / kg) PO QD AND
ii. ISONIAZID 250mg (5mg/kg) PO QD AND
iii. PYRIDOXINE 25mg PO QD AND
iv. PYRAZINAMIDE 1250mg (25mg/kg) PO QD AND
v. ETHAMBUTOL 750 mg (15mg/kg) PO QD
2- Continuation phase 4-5 months (avg 18 weeks)
i. ISONIAZID 250mg PO QD AND
ii. PYRIDOXINE 25mg PO QD AND
iii. RIFAMPICIN 500mg PO QD
B
Non pharmacological advice
1- prevention of spread of infection;
• Primary environmental control by using local exhaust ventilation techniques
• Reinforce treatment compliance – offer enrolment with patient-centered care/DOT
program if on analysis patient found out to be incapable of self care or without reliable
concerned attendants.
• if possible stay at home
• wear surgical mask when go outside or near other persons/ contacts esp younger children
and immunocompromised.
• Spit the sputum in disposable bags.
• general advice on good healthy diet and hydration
• Reinforce compliance to treatment
• provide awareness regarding possible side effects of medications and when to seek medical
help
2- inform that the disease needs to be notified to local health / TB control dept
3- Screening for all persons who came into close contact with the patient before initiation of
therapy
4- Encourage to keep regular follow up;
• to assess improvement in clinical condition, adherence to treatment and evaluation for
medication side effects at least once every month
• for sputum microscopy and culture at least once a month until 2 consecutive negative
specimens are obtained.
SIGNIFICANT TOPICS WITH SAQs 32
18.
A 30 year old man presents with acute painful swelling of the right parotid since 1 day
ANSWER
A
Questions on History:
• MMR vaccine
• Pain on chewing/swallowing
• Recent dental infection or dental procedures.
• Fever
• Constitutional symptoms
• History of mumps and ask immunization against it.
• History of chewing pan and tobacco.
B
Examination:
• Bilateral parotid glands: Check the swelling, inspect and determine extent. Look and check
any redness and tenderness, any discharging fistula. Check both sides and compare
(unilateral or bilateral)
• Face: Look facial asymmetry (involvement of facial nerve)
• Check cervical lymph nodes and tenderness
• Check ear infection
• Check cranial nerve VII
• Look for any associated swelling.
• Testes: For testicular swelling and tenderness
• Tonsils: For tonsillitis
C
Management Plan:
• Detailed History
• Examination: Complete general and systemic examination. Local examination of parotid.
• Treatment Plan:
Reassurance and provision of information about disease.
Plenty of fluids
Topical application of hot or cold packs to swollen parotid.
Soft diet until swelling resolved.
Acetaminophen/ibuprofen
Watch for complications like orchitis, meningitis, pancreatitis, joint pain.
If associated orchitis, provide scrotal support and bed rest.
Antibiotics if there is any superimposed bacterial infection
Not going to work until 5 days after start of symptoms. Notify
Covering nose and mouth when sneezing and coughing
SIGNIFICANT TOPICS WITH SAQs 33
19.
A year old presents with fever for 3 days and a pruritic rash which is more dense over the trunk.
The rash consists of papules, pustules and vesicles turning to pustules in some areas. His sister
had a similar rash 10 days ago. In this case.
A. What is the likely diagnosis and what conditions would you consider in differential
diagnosis?
B. What treatment will you prescribe.
C. What advice will you give to the patient?
ANSWER
A
Most likely Diagnosis
Chickenpox (Varicella zoster virus)
Differential diagnosis;
1- Impetigo
2- Scabies
3- Atopic / dermatitis or eczema with superimposed infection
4- Folliculitis
5- Herpes simplex
6- Herpes zoster
7- Eczematous herpetiform molluscum
8- Herpes simplex
B
Treatment
Non-Pharmacological
Maintain hygiene
Give bath to child
Trim nails or cover hands with gloves
Isolate the child
Give less spicy and high protein diet
Adding 2 cups of oatmeal to a lukewarm bath
Adequate fluid intake
Pharmacological:
Give diphenhydramine for severe itching
Calamine lotion on itchy area (avoid eyes and genitalia
Acetaminophen 10-15 mg/kg PO as needed Q 4-6 hourly
Avoid aspirin
Avoid ibuprofen
C
Advice to the patient
1- health education for patient and family regarding possible cause, its contagious nature,
prevention of spread and followup care
2- Stay at home, away from susceptible or at-risk persons (pregnant or immunocompromised
persons at home) until the lesions scab.
3- General dietary and hydration advice
SIGNIFICANT TOPICS WITH SAQs 35
20
Mrs. Ata brings her 6-year-old child to your clinic for his persistent cough and bouts of wheezing
off and on for the last 2 years
ANSWER
A
Further History
1- Frequency of episodes to assess severity
2- Nature of cough;
• dry / moist
• productive - any abnormal amount or color of sputum / presence of blood
3- Pattern of symptoms;
• seasonal or perennial
• early morning or night time
4- Precipitating factors;
• viral respiratory infections
• environmental allergens / stimulants
• cold air
• physical exertion
• emotional triggers like crying or laughing
• medication related
• food allergens
5- Associated sx
• fever/ nasal sx/ breathing difficulties/ frequent throat clearing/ other respiratory sx
6- how these episodes have been controlled
•any prev use of inhalers and response to therapy
7- Past / known medical history
• asthma or atopy - allergic rhinitis or conjunctivitis, eczema
• any serious respiratory illness in early childhood or later that required admission to hospital
• recurrent respiratory infections
• gastroesophageal reflux sx
• immunization hx
8- Family hx of asthma or atopy, any pulmonary disease,
9- Social and environmental hx
• living conditions
• exposure to pets
• passive smoking / exposure to cigarette smoke
B
Possible Triggers
1- Viral respiratory infections
2- Environmental allergens / stimulants;
SIGNIFICANT TOPICS WITH SAQs 37
• cold air/ dust mites/ pollens/ tobacco smoke/ animal dander/ cockroaches / perfumes
3- Physical exertion
C
Management
1- Confirmation of diagnosis
• Detailed history as above
• Physical examination
21
A 25 years old gentleman presents with on and off history of runny nose and cough for past 3-4
years. There is no associated fever or breathlessness.
ANSWER
A
History
1- any known precipitants
• seasonal or perennial allergens - pollens, animals
• occupational exposure
• environmental irritants or stimulants - smoke / strong odours/ cold air
2- detailed hx of cough; dry or productive or moist esp. look for hemoptysis,
3- associated sx like throat discomfort or frequent throat clearing, voice change, shortness of
breath or wheezing, chest pain, headaches
4- Medical hx:
• known personal or family hx of any respiratory or any other chronic diseases
• allergies/ atopy
• medications hx
B
Most likely ;
Seasonal allergic rhinitis
C
Non-pharmacological
1- Exploring and addressing ICE
2- Minimize exposure to precipitants:
• avoid known precipitants
• Improve ventilation in the living area
• use personal protective measures
3- Discourage continuous use of topical nasal decongestants
4- Prevention against colds or influenza ( influenza vaccination if indicated, washing hands with
soap or an alcohol-based hand rub)
5- Saline Nasal douching
D
Pharmacological Management
1- Intranasal corticosteroids
• MOMETASONE nasal spray 100 mcg QD
OR
2- Leukotriene receptor blocker;
• MONTELUKAST 10 mg PO QD
AND
SIGNIFICANT TOPICS WITH SAQs 39
3- Oral antihistamines
• LORATADINE 10 mg PO QD
SIGNIFICANT TOPICS WITH SAQs 40
22.
A 30 years old male comes to your clinic with 6-month history of dry, intermittent cough. The
cough is worst late in the evening and disturbs his sleep. He experiences shortness of breath
with it at times.
ANSWER
A
Diagnosis:
Cough variant asthma
B
Supportive History Points
1- episodic reversible symptoms mostly in response to triggers like;
•environmental allergens (pollens, dust mites, pets)
• irritants/ stimulants (smoke, cold air/ strong odour or fumes)
• Tobacco smoking. If yes, then how many pack-years.
• respiratory infections
• Physical or emotional exertion
2- associated chest tightness or wheezing
3- dry cough that aggravates during sleep
4- previous use of short acting inhaled bronchodilators and improvement of sx with its use
5- personal hx of atopy like eczema or allergic rhinoconjunctivitis
6- family hx of bronchial asthma or atopy
7- occupational exposure with precipitants
8- drug history, i.e. ACE-I use.
9- effect on daily activities
C
Management
1- Detailed hx (as above)
2- Physical exam, esp. look for;
• GPE: general appearance, respiratory rate and effort, SaO2, finger clubbing,
• ENT: nasal passages
• CHEST auscultation: wheezing, prolonged exp phase
• Skin: any eczema
3- Investigations
• Spirometry and PEFR
• CBC and differential
• CXR to exclude other aetiologies of chronic cough
• Allergy testing if indicated
4- Treatment
• Non pharmacological therapy
SIGNIFICANT TOPICS WITH SAQs 41
• Pharmacological therapy
- Rescue therapy;
> to minimise acute sx
> Short acting beta agonist inhaler as needed
SALBUTAMOL inhaler 200mcg PO prn 4-6 hourly with spacer
- Controller therapy
> to prevent exacerbation
> Inhaled corticosteroids
FLUTICASONE inhaler 250 mcg bid for 2 months
23.
A 30 years old man presents with dry cough since 2 months
ANSWER
A
Chronic dry cough - History
1- onset and timings
• how was it started? sudden or gradual
• is it persistent or intermittent or triggered cough ?
• any day/ night time or postural variation ?
2- Any known triggers ?
• environmental allergens / stimulants/ irritants
• Physical or emotional exertion
3-Any associated sx/ features ?
• preceding resp infection
• shortness of breath
• wheezing or chest tightness
• chest pain
• fever
• coughing up blood
• throat pain
• nasal sx
• heartburn
• weight loss
4- nature of work / occupation and smoking hx, active or passive
5- medication hx esp ACEI
6- Any known personal or family hx of atopy or respiratory illness ?
B
Physical examination
1- General physical
• Vital signs, appearance, respiratory effort, pallor, cyanosis, finger clubbing
2- ENT
• nasal mucosa and passages, nasal flaring, oropharynx and tonsils
3- CHEST
• deformities, use of accessory muscles of respiration, prolonged expiration ,
• auscultation- wheeze, crackles,
C
Differential Diagnosis
SIGNIFICANT TOPICS WITH SAQs 43
1- Bronchial asthma
2- GERD
3- upper airway cough syndrome
4- Drug - ACEI related
5- Post-infectious cough (TB)
6- Eosinophilic bronchitis
D
Investigations
1- Spirometry with bronchodilator
2- CXR
3- Sputum swab C&S and for AFP for 3 consecutive days.
4- CBC
SIGNIFICANT TOPICS WITH SAQs 44
24.
A 26-year-old gentleman presents with fever, cough, and breathlessness for 1 week. He has no
known comorbids. On examination, temperature is 39 C and respiratory rate is 23/min. Chest
examination reveals crepitations in right mid zone
ANSWER
A
Further points in history:
• The pattern and grade of fever.
• Is the fever associated with chills
• Is the cough productive in nature (if yes then ask..)
- the amount, frequency and appearance of sputum
- is there any blood in the sputum
• Any associated chest pain ( pleuritic chest pain)
• Is the patient taking any medicines
• History of smoking
• History of recent travel or exposure to sick contacts
B
Investigation to be ordered in a patient with a suspected case of pneumonia:
• Pulse oximetry: Used to assess severity. If oxygen saturation is ≤ 92% in air, the patient is
hypoxic and requires admission.
• Chest X Ray PA View (expected to show opacity in the right mid zone). If diagnostic
uncertainty or symptoms not resolving < 6 weeks after recovery. Persistent changes on CXR
> 6 weeks after recovery require further investigation.
• Complete blood count (increased WBC count, increased ESR and increased neutrophils
indicate an acute infection).
• Blood tests for acute and convalescent titres to confirm ‘atypical pneumonia (Legionella, C.
psitacci, M. pneumoniae)
• Sputum for Culture and sensitivity (helps in identifying the organism and the choice of
antibiotic) if not responding to treatment. If weight loss, malaise, night sweats or risk
factors for TB (ethnic origin, history of TB exposure, social deprivation, or elderly), request
mycobacterium culture.
C
Management Plan:
1- Reassurance and Explanation
• explain and inform about the disease and its severity
• address ICE
• explain that after starting treatment symptoms should steadily improve.
2- Patient education
SIGNIFICANT TOPICS WITH SAQs 45
4- Oral Abx
• AZITHROMYCIN 500mg PO QD on day 1 then 250mg PO QD for 4 days
OR
• CLARITHROMYCIN 500mg Q12 hrly for 5 -7 days
5- Followup
• for reassessment in 24-72 hrs
• with culture and sensitivity report and change in Abx accordingly
• Rule out any complications of pneumonia during the followup visit.
SIGNIFICANT TOPICS WITH SAQs 46
25.
A mother brings her 3-year-old son who has a barking cough, runny nose and low-grade fever
since 2 days. He has stridor when he cries
ANSWER
A
Most likely diagnosis:
Viral croup
B
Examination
-GPE: For general look, restlessness, distress, cyanosis, pallor, nasal flaring, stridor at rest.
-Vital signs: esp. pulse, temperature, SpO2 (if available).
-Chest examination for Respiratory Rate, tracheal tug, intercostal, subcostal, suprasternal
recessions.
-Auscultation for breath sounds.
C
1).History:
-H/o URTI: duration and severity.
-ask about fever, duration, severity, pattern.
-H/o cough, more at night?
-any h/o cyanosis, altered consciousness or h/o recurrent URTI.
2).Examination: As mentioned in part B.
3).Diagnosis: Is mostly clinical.
4).Non pharmacological treatment:
-Reassure and explain to mother.
-Inform about disease that it is self limiting and child's condition will improve with treatment.
-Steam inhalation.
Pharmacological treatment:
-Prednisolone 1 mg/kg PO single dose.
-give Antipyretic for fever.
Reassess in 1 hr and send home if stable.
-Advise mother to take child to specialist immediately if cyanosis, altered consciousness and
increasing distress.
SIGNIFICANT TOPICS WITH SAQs 47
26.
A thin and lean young man of 25 years presents with sudden pain in right upper chest and
progressive dyspnea (2 hours duration). On examination, cyanosis and BP 100/70 mmHg, pulse
rate 110/min, RR of 32/min and diminished breath sound on right upper chest
ANSWER
A
Most likely diagnosis:
Spontaneous pneumothorax
B
Investigations
1- Upright CXR PA view;
• showing visceral pleural line
• a visible rim between lung margin and chest wall if <2 cm when measured at level of hilum-
most likely small pneumothorax
• rim of > 2 cm - large pneumothorax
C
Management
1- Confirmation of diagnosis, largely clinical with;
• Targeted hx
• Chest exam: Look for signs of pneumothorax.
o hyperresonance on percussion
o decreased vocal fremitus
o displaced trachea
o decreased breath sounds.
2- Reassurance and explanation to the patient
3- Monitor ABC, as the patient is in cardiorespiratory distress evident by tachypnoea, tachycardia
and cyanosis - most likely due to tension pneumothorax
4- Decompress the affected side even before knowing the CXR result
• Immediate insertion of a standard 14-gauge IV catheter into the pleural space at the
intersection of the mid-clavicular line and the second or third intercostal space
5- Referral as emergency to pulmonologist for further management likely tube thoracostomy.
SIGNIFICANT TOPICS WITH SAQs 48
27.
A 25-year-old man presents with dizziness since 3 days. He describes it as a spinning of the
room. It occurs when he suddenly turns his head or on turning to the side in bed, each episode
lasts for a minute.
ANSWER
A
History ;
1- Any previous history of similar symptoms
2- Any associated complaints like ;
• headaches
• hearing or visual impairment
• feeling of ringing in the ears
• fullness or pressure in ears
3- Any significant past hx esp;
• head trauma esp. of whiplash type
• ear surgeries
• migraines
• ear infections
B
1- Most likely: Benign Paroxysmal Positional Vertigo
2- Differential: Ménière’s disease
C
Diagnostic test for BPPV
Dix – Hallpike Maneuver
D
Management:
1- Confirmation of diagnosis and rule out other diagnoses with complete history, physical
examination, investigations
2- Non-Pharmacological:
• Reassurance, explanation, patient education and instructions
• advice to limit symptoms by getting out of bed slowly and reducing head movement
• Consider repositioning manoeuvres – Epley’s manoeuvre in case of posterior canal BPPV
3- Refer to ENT- Dizzy clinic for assessment and management of other causes
SIGNIFICANT TOPICS WITH SAQs 49
28.
A 35 year old male presents with few days history of left sided ear pain, discomfort and itching.
His symptoms started after swimming.
ANSWER
A
Most likely-
1- ACUTE OTITIS EXTERNA
2- ACUTE OTITIS MEDIA
3- FURUNCULOSIS
4- CONTACT DERMATITIS of ear canal
B
RISK FACTORS (Otitis externa)
• Canal obstruction
- deformities
- Impacted cerumen
• Warm humid climate
• Fresh water swimming
• Skin diseases
- Eczema
- seborrhea
- psoriasis
• Local trauma to ear canals from frequent manual wax cleaning or use of foreign objects
- scratching from matchstick or cotton buds
• use of external hearing aids/ auditory devices
• Topical drug allergies such as from neomycin ear drops
• Diabetes and other immunocompromised state
• prolong / frequent use of topical Abx inhibiting normal flora and promoting fungal growth
C
History Points
1. Presence of risk factors as discussed above
2. Otalgia and its onset
• Pain of otitis externa usually has rapid onset
• Any pain on manipulation of tragus
• Indicates ear canal inflammation
3. Associated sx from ear canal inflammation
• Itchiness in the ears starts early
• feeling of aural fullness
• Otorrhea
• Hearing impairment commonly from obstruction of canal due to inflammation
SIGNIFICANT TOPICS WITH SAQs 50
• Investigations :
- Ear swab for culture and sensitivity
2- Non Pharmacological
• Reassurance, explanation and health edu of the patient
• Explore and address ICE
• Instructions to the patient
- Identify and control the risk factors
- Keep the ears dry
- Avoid the use of any foreign object inside the ears
- Use occlusive ear plugs while swimming
- Proper use of medication/ ear drops
• Aural toilet with gentle suction and dry mopping with wisp of cotton
3- Pharmacological
• Analgesics
- ACETAMINOPHEN 1g PO PRN QID
OR
IBUPROFEN 400mg PO PRN TID
• Topical antibacterial drops
> majority of infections are bacterial)
> prior to the use of drops cleaning of ear canal of any debris or wax is needed if present
- NEOMYCIN-POLYMYXIN-HYDROCORTISONE 4 drops into the affected ear 3-4 times daily for 7-
10days
OR
- ACETIC ACID 2% 3-5 drops TID for 7-10 days
29.
A 25 year old gentleman presented with history of sneezing, rhinorrhea and frequent throat
clearing on and off for past 3-4 months. He has exacerbation of these symptoms with seasonal
variation.
A. What is the most probable diagnosis?
B. List 4 points on examination with likely findings pertinent to the diagnosis?
C. Outline your management plan in 5 points.
ANSWER
A
Diagnosis:
• Seasonal Rhinitis
• Allergic Rhinitis
B
Finding on examination
• On eyes: Swollen + red
• Nasal Examination:
o Nasal crease may be present caused by upward rubbing of tip of nose by palm of
hand.
o Nasal speculum examination: Mucosa of turbinate may be pale, swollen (boggy)
o Thin, watery secretions.
• Throat: Look for
o Erythema
o Postnasal drip
o Any evidence of infection
• Chest: For signs of infection or asthma
• Skin: Signs of allergy or eczema
C
Management
• Few More Questions in History
1. Any history of triggering factors such as allergens, e.g., house dust mite, grass, tree and
weed pollen
2. Sensitivity to environmental irritant, smoke/cold air/strong odor.
3. Any associated symptoms, h/o shortness of breath, cough, fever, wheeze.
4. Any previous attacks.
5. H/O allergy/atopy/eczema
6. Any pet animals at home
7. Personal or family history of any chronic respiratory diseases. Family h/o asthma/atopy
8. Medication history.
• Detailed Examination: As mentioned above.
• Patient’s Education:
1. About his condition, explain severity of illness and preventive measures.
2. Reassurance and address ICE
• Non-Pharmacological Management
SIGNIFICANT TOPICS WITH SAQs 53
30.
Anwer, a three year old child is brought to your clinic with the complaints of a runny nose and
low grade fever. He has developed stridor since the last 4 hours.
ANSWER
A
Further history
1- How rapidly the sx developed - severity and progression
2- Any h/o cough. If yes then, nature of cough, with or without sputum.
3- Any associated hoarseness
4- Any hx of choking
5- Duration of upper respiratory illness, runny nose and fever.
6- Any h/o drooling/dysphagia
7- Any rashes
8- Previous h/o same symptoms or is it the first time.
10- Any sick contacts at home.
11- Any personal family history of atopy/asthma
12- Immunization status
13- As about birth history, developmental history.
B
Examination:
• Signs of severe airway narrowing; look for:
1-Respiratory distress RE > 45 (tachypnea), temperature: may be febrile.
2-Pallor or cyanosis
3-If patient using accessory muscles for breathing.
4-Tracheal tug
5-Lethargic
6-Reduced attentiveness.
• ENT: If child is in distress, then skip it.
• Chest Examination: Decreased air entry, reduced breath sounds +/- added sounds.
C
Management
1- History: As discussed.
2- Examination: Relevant is as discussed.
3-Emergency Management/Investigations:
• If child is in severe to moderate respiratory distress, we can skip investigations. In that
situation, first give child urgent treatment.
o Check for airway, breathing and circulation.
o Check oxygen sat with pulse oximeter (if available)
o Discuss with parents about condition of child, and arrange referral to tertiary care
hospital with pediatric care.
SIGNIFICANT TOPICS WITH SAQs 55
• If there are no signs of respiratory compromise, detailed history, examination, then order x-
ray neck (most probably croup to see thumb sign) and x-ray chest (to exclude any lung
pathology)
4- Nonpharmacological Treatment:
Parents' education (in mild to moderate cases)
- Explore and address ICE, reassure parents about disease being manageable with supportive and
symptomatic treatment.
- Educate parents if child develops signs of of upper airway narrowing (discussed in exam section)
to visit ER immediately.
• Oxygen inhalation if Sat O2 is <92%
• Steam inhalation at home BD for 3 days.
5- Pharmacological
- Syr. ACETAMINOPHEN 15 mg/kg PO q. 6h. for 3-4 days.
- Oral dexamethasone 0.6 mg/kg PO (single dose for mild to moderate croup)
6- Follow-up
- After 3 days or if child is not improved, then may visit earlier
SIGNIFICANT TOPICS WITH SAQs 56
31.
A 6 year old child presents with a runny nose, sore throat and a nonproductive cough for 4 days.
On examination, the child has temperature of 37.8 F, throat is slightly hyperemic, and both sides
of the nose look congested. Ear appears normal and neck glands are not palpable.
ANSWER
A
Provisional Diagnosis:
Viral pharyngitis (based on history and modified Centor criteria)
B
Diagnosis is clinical;
At this stage I will not order any investigation but if pt s condition get worse or not respnding with
symptomatic treatment I will order following test.
Investigations:
Peripheral smear
GABHS rapid antigen detection test
Throat swab
CBC, ESR, CRP and imaging studies not required.
> WBC count decrease to fewer than 5000 cells
> Atypical lymphocytes
C
Management
Detailed History of presenting complains like fever ,checked at home.continous or
intermittent,does temp increases more than recorded in clinic,associated symptoms like cough ear
pain headache or loss of appetite.any skin rash.
Previous hx of sore throat,any sick contacts at home or school.
Physical Examination
> Edema & erythema on the pharyx
> Redness around the external Nate’s
> Nasal mucosa is often erythematous
> pharyngeal erythema with exudates >
> Non pharmacological:
Non-pharmac ological
• Health education.
• Parent’s reassurance about self limiting course of disease.
• Watchful waiting and immediate seeking of medical advice in case of worsening of
symptoms.
• Watchful screening of other family members too for development of similar complaints.
• Lukewarm salted water gargles.
• Avoiding cold, spicy and sour edibles/drinks.
SIGNIFICANT TOPICS WITH SAQs 57
32.
A 40 years old female presents with fever, facial pain associated with greenish nasal discharge
for the last one week
ANSWER
A
*The most likely diagnosis is
Acute sinusitis ( maxillary)
B
Management
Detailed History:
• How long you have fever, associated symptoms of fever, i.e., rigors or chills. How it
relieves.
• History of facial pain, i.e., is it continuous or relieved by itself or by analgesic.
• How long she has the pain did she tired any medications for this problem
• Is it aggravated on movement/bending.
• Any association with weather, drugs, food
• Is there any history of cough or flu or headache
• How long she has greenish nasal discharge. What was the color in the beginning of
symptoms.
• History of weight loss, postnasal drip?
Examination:
• GPE: Anaemia, jaundice, cyanosis, lymphadenopathy
• Vitals: BP, Temp, Pulse, RR
• Inspection of throat and examination of oral cavity, tonsils.
• Inspection of nose, examination for polyps, congestion, foreign body, etc., DNS & stuffiness
in nose.
• Examination of face by pressing sites of all sinuses.
Investigations:
• X-ray PNS
• CBC
Treatment Plan:
• Non pharmacological:
* Reassurance & address ICE
* After complete physical examination & confirmation of the diagnosis , explain nature of
the disease
* Apply warm, moist washcloths to reduce congestion
* Drink plenty of fluid to thin the mucus
* inhale steam atleast 3times a day
* use a humidifier
* Do nasal douching with N/S.
SIGNIFICANT TOPICS WITH SAQs 59
• Pharmacological
*tab Acetaminophen 1gm PO, 8hrly for 3 days then if fever subside then SOS
* Antibiotic usually not prescribed, but as patient has facial pain , so in this situation will
give him broad spectrum like tab Amoxicillin 500 mg ,PO , 8hrly x 10 days
* Steroid nasal spray (beclometasone) 2 puffs each nostril BD FOR 7DAYS
* Follow-up: Revisit after 3 days.
* Refer to ENT , If symptoms persist after 2week treatment
SIGNIFICANT TOPICS WITH SAQs 60
33.
A 20 years old male presents in your clinic with 2 days history of fever 102 F, headache and sore
throat. On examination; temperature 101 F, throat is mildly hyperemic. Rest of the
examination is unremarkable.
ANSWER
A
• Viral sore throat
• Streptococcal sore throat
• Glandular fever
B
No investigation usually. May order throat swab, rapid antigen test,CBC Baseline if symptoms
severe and not relieved by supportive measures.
C
Management
Detailed History:
• History of fever, duration, onset, associated symptoms, pattern of fever.
• History of headache, duration, intensity, characteristics, aggravating/relieving factors.
• History of weight loss, postnasal drip?
Examination
• GPE: Anaemia, jaundice, cyanosis, lymphadenopathy
• Vitals: BP, Temp, Pulse, RR
• Inspection of throat and examination of oral cavity, tonsils.
• Inspection of nose, examination for polyps, congestion, foreign body, etc., DNS & stuffiness
in nose.
• Examination of face by pressing sites of all sinuses.
Treatment
• Nonpharmacological:
o Reassurance, address ICE.
o Increase fluid intake
o Warm saline gargles
• Pharmacological:
o Advice analgesia & antipyretics ( paracetamol/ibuprofen)tab brufen400 mg 8hrly for
3days
o Identify indication of antibiotic : Tonsillar exudate, tender anterior cervical
lymphadenopathy, h/o fever, absence of cough, generally very unwell, complications
like quinsy/retropharyngeal abscess
o 90% recover without treatment
• Follow-up: In 3 days.
• Referral to ENT if not relieved in 2 weeks
SIGNIFICANT TOPICS WITH SAQs 61
SIGNIFICANT TOPICS WITH SAQs 62
34.
Mr. Rasheed 28 years old presents with history of fever with nasal stuffiness and facial pain
since 5 days. On examination, he looks unwell with tenderness over the maxillary sinuses and
temperature is 102 F.
35.
A 2 year old boy brought by his mother with complaints of fever for 3 days. Child has been
pulling his right ear and now mother has noticed discharge coming from that ear.
ANSWER
A
Diagnosis:
Acute otitis media
A
Relevant Questions :
➢ Is it abrupt in onset: As otitis media presents with sudden onset of ear pain.
➢ Is pain increased during night or nap time: Otalgia exhibits during lying due to increased
eustachian tube dysfunction in AOM.
➢ History of fever: High grade with chills, does touch the baseline, did mother checked temp
at home Two-thirds of children with acute otitis media have history of fever.
➢ Hearing Loss: Most young childrent cannot provide an accurate history. Parents may
suspect a hearing loss or child as inattentive
C
Relevant Examination:
Pneumatic Otoscopy
• Standard examination technique
• 90% sensitive & 80% specific
• Otoscopic criteria for diagnosis: Fullness, bulging, cloudiness and redness of ear drum
Adjunctive Screening Techniques:
• Tympanometry
• Acoustic reflectometry
• Audiometry
Temporal Bone CT or MRI: To assess for associated complications like;
• Mastoid effusion
• Subperiosteal abscess
• Bony destruction
• Meningitis
Never Ignore:
• Other than ear, always examine entire Head & Neck region, external ear and tympanic
membrane
D
Management: After complete GPE, confirmation of diagnosis,
Non-Pharmacological:
SIGNIFICANT TOPICS WITH SAQs 64
Reassure mother & explain abt the nature of the disease and that 90% resolve in 7-8 days
of treatment.
Apply warm moist wash cloth over the infected ear
No bottle feeding
Avoid cigarette smoking
Wash hands and toys frequently to reduce chances of getting cold
Pharmacological:
Oral pain killers: Ibuprofen, acetaminophen 15mg /kg
Topical analgesics: Benzocain ear drops, antipyrine ear drops
Antibiotic Treatment:
Amoxicillin 80mg /kg/per day 8hrly for 7days
Amoxicillin/clavulanate, if resistance.
Referral: If no improvement of AOM, refer for surgery. It may consist of;
Adenoid removal
Ear tubes: Insert tiny tubes, allow air and fluid to drain from middle ear.
Follow-up: Call mother to come back with child after 3 days and reassess if child condition has
improved or not.
SIGNIFICANT TOPICS WITH SAQs 65
36.
A lady brings to you her 5 year old son who has again developed shortness of breath with the
onset of cold weather in Karachi. There is no family history of atopy. In his school he used to
develop runny nose when he played in the sandy play area.
ANSWER
A
Therapeutic Plan:
1. Symptomatic treatment.
a) Oral antihistamine, i.e., Syr. Cetirizine 5 mg/5 ml) 5 ml OD or Syr. Loratadine (5 mg/5 ml) 5
ml OD
b) Azelastine intranasal antihistamine applied three times daily upon followup if symptoms
not controlled, then ask for follow up in two weeks.
2. Preventive Treatment
a. Avoid causative allergens.
b. Follow-up in two weeks if symptoms not adequately controlled.
B
Measures to be given to mother:
• Use of preventive clothing and avoidance of cold exposure.
• Avoidance of sand/dust
• Identify other things that cause allergy like pet at home
• Undergo skin or blood tests for allergy if symptoms are persistent or other allergens are
detected.
SIGNIFICANT TOPICS WITH SAQs 66
37.
A 54 years old multi para lady has a swelling on the right side of her neck. She chews 25 pans
every day for the last 30 years.
A. Give three possible diagnoses. Mention points in favor and against each diagnosis.
B. Give five investigations and reasons for doing them.
C. What will be your line of management?
ANSWER
A
Most probable diagnoses are;
i) Malignant lesion: Malignant melanoma, leukemia, malignant lymphadenopathy
a. Points in favor: Fever, dark color, ulceration, skin fixation, bleeding, hard texture, weight
loss.
b. Points against: No fever, mobile mass, normal appearance of skin, no weight loss.
ii) Benign tumor: Lipoma, lymphoma, etc.
a. Points in favor: Mobile mass, normal appearance of skin, no signs of infection
b. Points against: dark color, ulceration, skin fixation, bleeding, hard texture
iii) Infective lesion: Epstein Barr virus, cytomegalovirus, HIV, lymphadenitis
a. Points in favor: Signs of infection (redness, swelling, warmth, drainage)
b. Points against: No signs of infection
B
Investigations:
i) Full Blood Count: To rule out infective lesion with sepsis, assess for leukemia.
ii) LFTs, ESR.
iii) Thyroid Function Tests: To rule out a thyroid cause of the neck swelling
iv) Chest X-ray: To rule out carcinoma of the lungs as the primary lesion.
v) Tumor Markers: To rule out malignant lesion.
vi) Ultrasound of the Neck: For unexplained neck swelling and to rule out soft tissue sarcoma.
C
Line of Management:
i) History: Assess the patient for concerning points in history, like trauma, being a smoker,
alcohol use, worrying symptoms like weight loss, night sweats, dyspnea, dysphagia,
unexplained fever, persistent fatigue, previous radiation to neck, immunocompromised state,
family history of endocrine cancer, recent travel history, any cough or sore throat.
ii) Physical Examination: Do a complete general and systemic exam, especially focusing on
following:
a. Check the neck for any redness, warmth, discharge, discoloration, fixation to underlying
structures, movement with swallowing, pulsatality.
b. Examine skin for pallor, petechiae, bruising (leukaemia), excoriation (lymphoma);
generalized rash (adenovirus, cytomegalovirus, Epstein-Barr virus); or jaundice
(cytomegalovirus, Epstein-Barr virus).
c. Examine all structures in the head (eyes, ears, nose, mouth, throat) for any source of
infection or malignancy.
d. Examine chest for any signs of lung disease
SIGNIFICANT TOPICS WITH SAQs 67
38.
45 years old Hassan Pari, an Afghan refugee came to you with history of cough, low-grade fever
and night sweating for the last three months. She has been taking herbal medicine off and on
but has not responded.
This evening, she presented with passage of red blood in her sputum. She is thin, looks anxious
and pale. Temperature is 99.5 F, pulse rate 120/min and she is breathless.
ANSWER
A
Differential Diagnosis:
❖ Primary pulmonary tuberculosis
❖ Lung cancer
❖ Bronchiectasis
❖ Constrictive pericarditis
B
Investigations :
* CXR -PA view (mostly cavitation is found in the apex of lungs ) cbc ESR baseline
* Sputum samples for culture (looking for acid -fast bacilli)
* Tuberculin test .baseline LFT Before starting treatement
C
Management :
Confirmation of Diagnosis
By detailed history, physical examination, after interpretation of chest x-ray, interpretation of
tuberculin test.
Non pharmacological Management:
Contact tracing and public health notificiation is very important
Stay home till the sputum fir AFB will become negative
Adequate ventilation in the room
Cover mouth with a mask means not spread infection
Regular follow is the most important to check the response of medicine
Pharmacological Management
• Standard treatment with anti tuberculosis treatment
• Four drugs (Rifampin , Isoniazid, Ethambutol , Pyrazinamide ) for two months & two drugs
combinations (Rifampin & Isoniazid) for two months
• Dosage pt should take medicine early in morning before breakfast
o Tab Isoniazid = 5 - 10 mg /kg per day
o Tab Ethambutol = 10-15mg/kg per Day
o Tab Rifampin =15-20mg/kg per day
o Tab Pirazinamide = 20-2G/kgPER DAY
o Tab Pyridoxine 10 mg OD
SIGNIFICANT TOPICS WITH SAQs 69
• Patients often forget pills, so consider directly observed therapy (DOT) as follows:
o Initial Phase: 8 weeks for drugs
▪ Rifampicin 600 to 900 mg PO 3 times per week.
▪ Isoniazid 15 mg/kg PO 3 times per week
▪ Pyrazinamide 2.5 g PO 3 times per week
▪ Ethambutol 30 mg/kg PO 3 times per week
o Continuation Phase: 16 weeks on 2 drugs
o Rifampicin and isoniazid at same doses
o Continue pyridoxine
• Adviceto Patient:
o First should explain in writing the side effects of anti tuberculosis drugs
o Explain the patients that the color of urine will be orange due to Rifampin ,
o Contact to health care provider if develop eye related issues
• Follow up should be regular to reassess patients condition
• Consultation can be made if resistance to the drugs
SIGNIFICANT TOPICS WITH SAQs 70
39.
An 18 year old engineering student presents with episodes of recurrent sneezing, stuffy nose,
profuse watery rhinorrhea and itchy eyes since 2 months. The symptoms are interfering with his
studies as he cannot concentrate.
ANSWER
A
Questions Asked in History:
1. Any known precipitants
a. Seasonal or perennial allergens – pollens, animals
b. Occupational exposure
c. Environmental irritants or stimulants – smokey strong odors/cold air.
2. Detailed history of cough; dry or productive or moist, especially look for hemoptysis.
3. Associated history (symptoms) like throat discomfort or frequent throat cleaning, voice
change, shortness of breath or wheezing, chest pain, headache.
4. Medical history
a. Known personal or family history of any respiratory or any other chronic disease.
b. Allergies/atopy
c. Medication history
5. Associated with fever or H/O bronchial asthma.
B
Non-pharmacological:
• Exploring and addressing ICE
• Reassurance
• Lifestyle modification (avoid allergens and irritants)
• Minimize exposure to precipitants
o Improve ventilation in the living area
o Use personal protective measures
o Avoid known precipitants
• Discourage continuous use of topical nasal decongestants.
• Prevention against cold and influenza (by influenza vaccine before season starts)
• Wash hands with soap or alcohol based rubs.
• Saline nasal douching
Pharmacological:
• Intranasal corticosteroids
• Mometasone (Rhinocort) nasal spray 1000 mg OD (OR)
• Leukotriene receptor blocker (monteleukast 10 mg PO OD and
• Oral antihistamines (loratidine 10 mg PO OD)
• Followup in 5days in clinic
SIGNIFICANT TOPICS WITH SAQs 71
40.
An 18 year old student presents with bilateral itching of eyes for the last 3 weeks. There is no
associated discharge or pain. She also denies any cough, cold or fever.
On examination, both eyes are mildly infected, cobblestoning is noticed in palpebral conjunctiva
of both eyes.
ANSWER
A
Diagnosis:
Allergic conjunctivitis
B
Symptomatic Treatment
Pharmacological:
-Topical Antihistamines e.g sodium cromoglicate, or olopatadine eye drops to relieve itching.2-4
drops each eye for 4times a day
-Oral antihistaminics tab loratidine 10mg HS for 5 days
-Topical NSAIDs if associated with eye pain.
-Topical Mast cell stabilizers Cromolyn sodium+lodoamide.
Non pharmacological:
-Cold compresses
Use googles
-Wash eyes with cold water.
-Personal hygiene like hand washing.
-Avoid contact lenses.
-Refer if symptoms persist despite treatment or if vision is affected.
C
Prevention:
-Identification and avoidance of allergens like dust mites,pollens,animal fur,grass,weeds and
weeds.
-keep Windows shut including car Windows when pollen count is high.
-Wear glasses or sunglasses.
-Avoid grassy spaces.
SIGNIFICANT TOPICS WITH SAQs 72
41.
A 25 year old lady with no known comorbids presents with redness and pain in the right eye for
1 day. Other people in her family have also developed similar complaint.
ANSWER
A
Points on History:
1) h/o upper RTI
2) eyes crusting or stick together on waking, h/o itching in eyes and foreign body sensation.
3) start in one eye then spread in both eyes,
4) other family members affected or school / office members affected
5) rule out red flag signs like
a) decreased visual acuity
b) deep pain in eye
c) absent or sluggish pupil response
d) h/o trauma (time to refer)
e) h/o contact lenses
B
Probable Differentials:
❖ viral conjunctivitis
❖ bacterial conjunctivitis
❖ allergic conjunctivitis
❖ Keratoconjunctivitis
C
Differentiating Points between Differential Diagnoses:
✓ bacterial conjunctivitis( difficult to differentiate from viral, marked eyelid edema & purulent/
mucopirulent disscharge)
✓ allergic conjunctivitis is seasonal, contact with allergen, photophobia, FH of atopy,
✓ Keratoconjunctivitis with decrease visual acuity, very painful eye, blurred vision, photophobia,
profuse watering, circumcorneal injection
D
Management Plan:
Usually self limiting, bath with boiled cooled water in morning and night, hand washing,
avoid shares towels, avoid contact lenses.
If not improved in 3-5 days then topical chloramphenicol qds for 5 days
Advice to seek medical advice if dec visual acuity, eye become painful rather than sore,
significant photophobia, eyelid swelling, symptoms not improving in 5 days
SIGNIFICANT TOPICS WITH SAQs 73
42.
A 10 years old girl is brought by her mother to your clinic with the complaint of purulent discharge from
the right eye since 2 days. She is otherwise well.
ANSWER
A
Ask about
➢ discharge, is it purulent
➢ crusting or stickiness of eye in morning.
➢ association with upper respiratory symptoms (to differentiate between bacterial and viral
conjunctivitis).
➢ other eye involvement
➢ is it associated with pain
➢ eye symptoms in other family members or colleagues
➢ Rule out red flag signs
o decreased visual acuity
o deep pain in eye( not soreness)
o corneal damage on fluorescein staining
o H/o trauma
B
Most likely diagnosis:
Bacterial Conjunctivitis (because of purulent discharge and involvement of one eye).
C
Initial Investigations:
• conjunctival scrapping, culture, gram stain, geimsa stain for chlamydia,
• discharge smear
• fluorescent corneal staining.
D
Management Plan:
Non pharmacological advice:
use hygiene
hand washing with soap n water,
avoid towel , pillow case, linen, napkin sharing
Avoid contact lens use, discard used contact lenses, with case n solution
Avoid touching eyes
Avoid contaminating swimming pools
wash with cool boiled water twice daily.
Pharmacological treatment:
Use topical chloramphenicol qds for 5 days or topical polymyxin B+ trimethoprim or
gentamicin q. 4h. for 7-10 days. For severe cases, fluoroquinolone TID for 7 days
SIGNIFICANT TOPICS WITH SAQs 74
Povidone iodine 1.25% ophthalmic solution can replace antibiotics in developing countries
supportive treatment like topical NSAIDs, topical antihistamines, artificial tears, oral
analgesia may relieve symptoms.
Referral Considerations: Refer if.
decreased vision
painful eye
photophobia
Symptoms not improving in 5 days.
SIGNIFICANT TOPICS WITH SAQs 75
43.
An infant of 9 months is brought to your clinic by his mother. She is concerned that his left eye deviates
inwards when looks straight ahead, she has noticed this problem since birth, but was expecting that it
will correct itself when he gets older.
ANSWER
A
Clinical Features to look for:
i) General physical health of the child (age, height, weight, etc)
ii) Developmental milestones (for CP)
iii) Dysmorphic facial features (for Down’s syndrome)
iv) Torticollis, nystagmus, squinting of one eye.
v) Birth history (pre-term), natal, antenatal, postnatal.
vi) Family history of visual disorders (strabismus, amblyopia, retinoblastoma, childhood glaucoma,
childhood cataract, ocular or genetic syndrome).
B
How I will proceed with this consultation:
History: Onset, course, duration, associated symptoms such as eye discharge, vision
problems, birth history, family history as described above
Developmental milestones.
Physical Examination:
o General appearance, vital signs and growth parameters (height, weight, head
circumeference, etc.)
o Ocular examination:
▪ Eyelid: For ptosis.
▪ Orbit, conjuncivae, sclerae, cornea, iris with light.
▪ Corneal light reflex (symmetry)
▪ Fixation and alignment of eye or fix and follow test for visual acuity.
▪ Cover, cover-uncover and alternative cover test (for manifest and latent
strabismus)
▪ Red reflex (for ocular alignment, asymmetry, refractive error, cataract,
retinoblasoma)
If history and physical examination suggest strabismums, child would be referred to a
pediatric ophthalmologist for corrective lenses and eye patching to prevent amblyopia.
Counsel mother regarding prompt intervfention and prognosis.
SIGNIFICANT TOPICS WITH SAQs 76
44.
A 25 year old male presents in your clinic with right sided red eye for the last two days. He is having
yellowish discharge and eye feels sticky in the morning. His vision is normal.
A. Enlist three important questions you would like to ask in the history.
B. Enlist three important relevant steps you would perform in the examination.
C. Enlist the most probable cause.
D. Enlist your non-pharmacological and pharmacological management plan.
ANSWER
A
Three important questions to ask:
i) H/O trauma to eye or head or fall or accidental foreign body insertion in eye.
ii) H/O associated eye pain. If yes, degree of pain, either only surface irritation or deep eye
pain.
iii) H/O photophobia or blurring of vision.
iv) H/O preceding upper respiratory tract infection.
v) H/O contact with the person with similar complaints.
B
Three important steps on exam:
i) Light Reaction/reflex. Photophobia severe in anterior uveitis, moderate in conjunctivitis and
absent in acute glaucoma.
ii) Visual acuity. Decreased in anterior uveitis and acute glaucoma, but intact in conjunctivitis.
iii) Eye Exam:
a. Cornea. If steamy or hazy, indicative of acute glaucoma. Normal in anterior uveitis
and conjunctivitis.
b. Conjunctiva: Redness, vessels clearly branch from corner of eye towards the cornea
in conjunctivitis.
c. Pupil. Small in anterior uveitis, large in acute glaucoma, but normal in conjunctivitis.
C
Most Probable Cause
Acute Bacterial Conjunctivitis
D
Nonpharmacological advice:
Patient’s education about the condition, address his concerns. Counsel about self limiting
course of disease, 65% cases settle in 3-5 days without treatment.
To observe simple hygienic measures, handwashing with soap and water
Avoid towel, pillow case, linen, napkin sharing
Avoid contact lens use
Avoid touching eyes
Advise patient to bate eye in boiled cold water morning and evening.
Pharmacological treatment:
Review patient in 3-5 days. If unimproved, consider treatment with;
o Chloramphenicol eye drops 2 drops QDS for 5 days
o Polymyxin B+ trimethoprim at night OR
SIGNIFICANT TOPICS WITH SAQs 77
45.
A 55 year old man presents with history of pain around right eye for past several hours and vomiting.
Pupil on right is dilated.
ANSWER
A
Provisional Diagnosis
Acute angle closure glaucoma
Scenario does not mention the presence/absence of red eye, but other signs and symptoms and
acute onset of symptoms indicates acute angle closure glaucoma, as well as patient’s age above 40
years.
B
Management Plan:
i) History: In addition to points given in the scenario, ask about;
a) Previous history of subacute attacks (presence of episodic haloes around bright light)
b) Vomiting
c) Abdominal pain
d) Nausea
e) Impaired vision
f) Are the attacks precipitated in light or dark room
g) Are the attacks relieved by sleep or entering a bright environment.
h) Past medical history of DM
i) Any family member had same problem in past (strong family history in most cases)
ii) Examination:
a) Presence of red eye.
b) Appearance of cornea being steamy or hazy.
c) Pupil is fixed and dilated.
d) Eyeball feels hard.
f) Cataracts +/-
g) The intraocular pressure will be raised on tonometer testing (if available)
iii) Plan:
a) Urgent referral as it is a medical emergency.
b) Specialist treatment:
a. Miosis with miotic agents (e.g. pilocarpine 4% drops)
b. Acetazolamide +/- apraclonidine and/or latanoprost drops (decreases aqueous
production)
c. Surgery or laser treatment (peripheral iridectomy) to allow free aqueous circulation
once IOP is decreased.
d. Prophylactic surgery on right eye.
e. Regular eye checkups to rule out development of chronic glaucoma.
SIGNIFICANT TOPICS WITH SAQs 79
46.
A 10 year old girl is brought by her mother to your clinic with the complaint of watery discharge from
both eyes since 2 days. She is otherwise well.
ANSWER
A
Questions to ask on history:
➢ Is there any change in child’s vision.
➢ Does the child have any known allergies.
➢ Does the child try to avoid light exposure.
B
Most likely diagnosis:
❖ The child most likely has viral conjunctivitis.
❖ The other possibility would be allergic conjunctivitis.
C
Management Plan:
i) History: Points as discussed above.
ii) Physical Examination:
a) Visual acuity
b) Pupillary response
c) Corneal lustre
d) Presence of redness in the conjunctiva
e) Papillae under the upper eyelid (enlarged or not)
f) Pre-auricular lymph nodes (enlarged or not)
iii) Treatment/Referral Plan:
a) Non-pharmacological: Advise mother to;
a. Bathe the affected eye(s) with boiled, cooled water morning and night
b. Use simple hygiene measures (e.g. hand washing and not using shared towels)
b) Pharmacological:
a. Viral Conjunctivitis:
• Usually self-limiting condition; 65% settle in 2–5 days without treatment. So
reassure the mother and ask to follow up after 5 days if symptoms not improved.
• If symptoms are not improving in 3 to 5 days to follow up for review of diagnosis and
treatment with topical chloramphenicol qds for 5 days.
b. Allergic Conjunctivitis:
• Antihistamine drops, e.g., emedastine or olapatadine.
• Refer if not settling in a few days.
• Sodium cromoglicate and steroid drops (after advice from an ophthalmologist) may
help
c) Referral Considerations: Refer if symptoms are persistent despite treatment or if vision is
affected.
SIGNIFICANT TOPICS WITH SAQs 80
SIGNIFICANT TOPICS WITH SAQs 81
47.
A 44 year old lady has had dyspepsia intermittently for several years. She is very anxious as her titres
for helicobacter pylori antibodies are raised.
A. What will you tell her regarding the positive H. pylori test?
B. What non pharmacological advice will you give her?
C. Outline your pharmacological management plan.
ANSWER
A
Counseling Regarding Positive H. pylori:
-As patient is anxious I would explore her ideas concerns and expectations about disease.
-I would assess her knowledge about disease.
-I ‘ll educate her that it is a bacterial infection and explain that H pylori test is a serological test and
it will remain POSITIVE after eradication.
Follow Up:after 1 week
B
Non-pharmacological Advice:
i) Weight loss if needed.
ii) Avoid trigger foods, i.e., coffee, chocolate, tomatoes, fatty or spicy foods.
iii) Smaller meals and meals 3–4 hours before going to bed.
iv) Avoid smoking and alcohol.
vi) Avoid NSAIDs and ulcer-inducing drugs (aspirin, bisphosphonates, corticosteroids, potassium
supplements, SSRIs, crack/cocaine).
vii) Treatment for stress, anxiety or depression if needed.
C
Pharmacological Treatment Plan:
i) First Line Eradication Regime:
a) 7-day regime of PPI BD + Amoxicillin 1 g BD and either clarithromycin 500 mg BD or
metronidazole 400 mg BD.
b) If penicillin allergy, then 7-day triple regime of PPI BD + clarithromycin 500 mg BD +
metronidazole 400 mg BD.
c) If penicillin allergy and previous exposure to clarithromycin , then 7-day triple regime of PPI
BD + metronidazole 400 mg BD + levofloxacin 250 mg BD.
ii) Second Line Eradication Regime: If H. pylori positive at follow-up.
a) 7-days of PPI BD + Amoxicillin 1 g BD + either clarithromycin 500 mg BD or metronidazole
BD 400 mg (whichever not used first-line).
b) If previous clarithromycin and metronidazole, then 7-10 day therapy of PPI BD + amoxicillin
1 g BD + levofloxacin 250 mg BD or tetracycline hydrochloride 500 mg QDS.
c) If allergic to penicillin and no previous quinolone, then 7-10 day therapy of PPI BD +
metronidazole 400 mg BD + levofloxacin 250 mg BD.
d) If allergic to penicillin and had previous quinolone, then 7-day quadruple therapy of PPI BD
+ tripotassium dicitratobismuthate 240 mg QDS + metronidazole 400 mg BD + tetracycline
hydrochloride 500 mg QDS.
SIGNIFICANT TOPICS WITH SAQs 82
48.
An 18 year old male is brought to your clinic with complaints of fever and abdominal pain for last 5 days.
There is associated nausea, vomiting, and loss of appetite. He reports dark colored urine and 3 light
colored stools in past 24 hours. He lives in a small one room house with 3 other family members. He
does not have access to clean water.
On examination, he is febrile (temp 101 F), has yellowish discoloration of skin and sclera. Abdominal
examination is unremarkable. On labs, LFTs show total bilirubin 4.5 mg/dl, direct bilirubin 1.0 mg/dl,
SGPT 1500 U/L, alkaline phosphatase 198, GGT 70.
ANSWER
A
Most Likely Diagnosis:
Hepatitis A
B
Further Investigations:
-: Hepatitis A serology, IgM antibodies (diagnostic for hepatitis)
-Liver function tests including ALT,AST,S.billirubin.
-.Full blood count.to rule out infection.WCC,Hb% to R/O anaemia
-viral markers to R/O hepatitisB&C.
-.Abdominal USG (to rule out liver abscess)
C
Management: Supportive
1.History:in addition to given history I ‘ll further ask some related questions like detailed h/o of
fever and abdominal pain,immunization history for hepatitis and past medical history.
2.Examination:includes GPE,state of hydration,pallor,jaundice,Vital signsAbdominal examination
for tenderness of RHC and hepatomegaly.
3.Investigations:as mentioned above.
4.Treatment:
-Non pharmacological:
-Explaination to patient that it doesn’tween to be a serious disease, its viral in nature and it’s self
limiting.
-Educate the patient about disease and explore his ideas,concerns and expectations.
-Advice bed rest during acute phase of illness.
-Maintain personal hygiene,hand washing with soap and water.
-Adequate diet,avoid alcohol and medication which accumulate in liver.
Pharmacological Treatments:
-antiemetic for nausea, vomiting: metoclopramide
-antipyretic/ analgesia: acetaminophen not more than 3-4 gm per day inadults
-I/V fluids if dehydrated.
5.Follow up:f.up within 2 months.
D
SIGNIFICANT TOPICS WITH SAQs 83
Preventive Management:
1) hepatitis A vaccine and hep A human immunoglobulin to all household contacts
2) maintain strict enteric hygiene
3) provision of clean water, boiling of water or adding chlorine to cooking n drinking water
4) wash and peel fruits before consuming
5) avoid raw food or unhygienic source of food
SIGNIFICANT TOPICS WITH SAQs 84
49.
A 30 year old man presents with diarrhea since 2 months. The stools are loose, mucoid and explosive.
They occur 2-3 times/day. He has also noticed 1 kg weight loss since the symptoms occurred. He gives a
history of travel to Nepal recently 2 months ago.
ANSWER
A
Most Likely Diagnosis:
Traveler’s diarrhoea; most likely giardiasis due to prolonged duration.
B
Differential Diagnosis:
1. Irritable bowel syndrome
2. Inflammatory bowel disease
C
Steps in Management:
1. History:
Ask about:
-blood in stools
-foul smelling or offensive
-associated symptoms like fever, abdominal pain, vomiting
-History of food intolerance
-History of surgery
-History of stress
-History of medication or alcohol consumption.
2. Examination:
-Check BP, pulse ,temperature.
-Dehydration status
-abdominal examination for tenderness, distension or mass and bowel movements.
3. Investigations:
Blood: CBC, ESR, calcium, TFTs.
Stools microscopy for ova and parasites and culture & sensitivity.
Treatment:
Non pharmacological:
-use hygiene, proper hand washing after toilet use.
-Oral rehydration if dehydrated
-no specific diet restrictions in some cases lactose intolerance occurs so avoid milk n milk
products.
Pharmacological:
Tablet Metronidazole 400mg TDS for 7 days.
SIGNIFICANT TOPICS WITH SAQs 85
50.
A 5 year old boy is brought by his mother complaining of vomiting for the last 7 days. His weight and
height is within the 85th percentile.
ANSWER
A
Questions in History:
Vomiting:
• Duration, onset, and how many episodes in a day?
• Aggravating/relieving factors.
• Any h/o food taken before it started?
• h/o abdominal pain.
Specific Questions:
• Relationship to food
• Is it blood stained, bile stained, feculent?
• Any history of headache, neck stiffness.
• Is there history of abdominal pain, distension, constipation? (bowel obst. Ileus)
• Vertigo, hearing loss, tinnitus (Meniere’s disease)
• Weight loss, anorexia, cachexia (malignancy)
Past History of GI Disease: Any hisory of abdominal surgery, hernia
Past Hisory of Other Diseases:
• Any history of trauma
• Any history of thyroid, epilepsy, asthma, visual problems?
• Diabetes, renal disease, jaundice.
Any Investigation done before?
• Any USG abdomen, CT abdomen, upper GI endoscopy done before?
Drug History: Any drug allergies/reactions.
Family History:
• Specific – migraine, peptic ulcer
• Same problem of vomiting with other family members.
Social History:
• Ask mother, living accommodation?
• Any travel history?
• History of PICA?
• Anyone smoker or alcoholic in the house?
B
GIT related Differentials:
1. Gastroenteritis.
2. GI obstruction
3. Reflux (GERD)
4. Foreign body
SIGNIFICANT TOPICS WITH SAQs 86
5. Parasites
6. Colitis
Non GIT related Differentials:
1. Pertussis syndrome
2. Sinusitis
3. Otitis media
4. Head trauma
5. Systemic infections
SIGNIFICANT TOPICS WITH SAQs 87
51.
A 36 year old obese lady presents with retrosternal and epigastric pain since 3 months. Her stool H.
pylori antigen test is positive.
A. What pharmacological treatment would you prescribe?
B. List 4 non-pharmacological advices for this patient.
ANSWER
A
Pharmacological Treatment Plan:
i) First Line Eradication Regime:
a) 7-day regime of PPI BD + Amoxicillin 1 g BD and either clarithromycin 500 mg BD or
metronidazole 400 mg BD.
b) If penicillin allergy, then 7-day triple regime of PPI BD + clarithromycin 500 mg BD +
metronidazole 400 mg BD.
c) If penicillin allergy and previous exposure to clarithromycin , then 7-day triple regime of PPI
BD + metronidazole 400 mg BD + levofloxacin 250 mg BD.
ii) Second Line Eradication Regime: If H. pylori positive at follow-up.
a) 7-days of PPI BD + Amoxicillin 1 g BD + either clarithromycin 500 mg BD or metronidazole
BD 400 mg (whichever not used first-line).
b) If previous clarithromycin and metronidazole, then 7-10 day therapy of PPI BD + amoxicillin
1 g BD + levofloxacin 250 mg BD or tetracycline hydrochloride 500 mg QDS.
c) If allergic to penicillin and no previous quinolone, then 7-10 day therapy of PPI BD +
metronidazole 400 mg BD + levofloxacin 250 mg BD.
d) If allergic to penicillin and had previous quinolone, then 7-day quadruple therapy of PPI BD
+ tripotassium dicitratobismuthate 240 mg QDS + metronidazole 400 mg BD + tetracycline
hydrochloride 500 mg QDS.
B
Non-pharmacological Advice:
i) Weight loss if needed.
ii) Avoid trigger foods, i.e., coffee, chocolate, tomatoes, fatty or spicy foods.
iii) Smaller meals and meals 3–4 hours before going to bed.
iv) Avoid smoking and alcohol.
vi) Avoid NSAIDs and ulcer-inducing drugs (aspirin, bisphosphonates, corticosteroids, potassium
supplements, SSRIs, crack/cocaine).
vii) Treatment for stress, anxiety or depression if needed.
SIGNIFICANT TOPICS WITH SAQs 88
52.
A 20 year old young woman presents with 3 days history of anorexia, weakness and dark colour urine.
Her LFTs are:
S. bilirubin = 3.2 mg/dL (0.1 – 1.2 mg/dl)
Direct bilirubin = 2.2 mg/dL
Indirect bilirubin = 1.0 mg/dL
ALT (SGPT) = 1200 U/L (7-55 U/L)
Alk PO4 = 110 (45-115 U/L)
ANSWER
A
Diagnosis:
Acute viral hepatitis, likely viral hepatitis (ALT is elevated out of proportion to alkaline
phosphatase)
B
Test with Prognostic Value:
• PT/INR is the test of prognostic value (clinical signo of bad prognosis is encephalopathy)
C
Additional Tests:
1) CBC (cbc may show lymphocytosis, pancytopenia, coagulation profile reflects synthetic
function of liver, rbs should be checked to rule out hypoglycemia).
2) Urea and electrolytes
3) Hepatitis serology (IgM anti-HAV, HbsAg, IgM anti-HBC, anti-HCV)
4) Serum albumin
SIGNIFICANT TOPICS WITH SAQs 89
53.
A 35 year old smoker presents with recurrent dyspepsia for 4 months and is found to have a positive H.
pylori antibody test. On examination comfortable, BMI is 28 and abdominal examination is normal.
ANSWER
A
PHARMACOLOGICAL TREATMENT:
Helicobacter eradication therapy: Standard triple therapy for 7-10 days (up to 14 days)
i. Omeprazole 20 mg PO OD
ii. Amoxicillin 1 g PO q. 12h.
iii. Clarithromycin 500 mg PO q. 12h. (metronidazole 500 mg PO q. 12h. in case of previous
exposure to clarithromycin or resistance).
NON PHARMACOLOGICAL TREATMENT:
Healthy dietary modifications
-Avoid food that increases gastric acid secretions like fatty and spicy meals, caffeine,
chocolates.
-Good hydration.
-Use of probiotics in diet.
-Frequent small meals.
Avoid smoking and alcohol intake if any.
Reduce anxiety and stress.
Discourage frequent use of non-prescription NSAIDs
Promote continued use of antacids/alginates.
Raise head end of bed for dyspepsia
Having main meals well before going to bed.
SIGNIFICANT TOPICS WITH SAQs 90
54.
A 2 year old boy has been having bloody diarrhea and fever since 2 days.
ANSWER
A
Questions to be asked from mother:
Take detailed history from the mother
a. History of present illness: Number of stools, frequency, amount of blood, any mucus with
blood.
b. Nutritional history: Breastfed or on formula milk, weaning history. Water used,
boiled/unboiled.
c. Developmental history: If previous episodes of diarrhea, may affect length and weight of baby.
d. Immunization history: Especially for rota virus droplet as mostly cause of diarrhea in children is
viral.
e. Past medical History: Of diarrhea.
f. Drug History: As antibioitic induced – pseudomembranous colitis.
g. Travel History: For traveler’s diarrhea.
h. Any problems of GIT or surgery of colon in past.
i. Birth history and feeding after birth.
j. Any history of associated complaints, i.e., fever, abdominal pain, vomiting.
B
Points on Examination:
1. Assess hydration status, BP, pulse rate, dry mouth, decreased skin turgor, sunken eyes or
sunken fontanelle (babies) are all late signs.
2. Abdomen – masses, distension, tenderness, bowel sounds, stools.
3. Respiratory rate increased because of dehydration
4. Lethargic or not.
C
Management:
1. Detailed History + Systemic Examination: As discussed above.
2. Investigations:
a. FBC, ESR
b. Serum electrolytes
c. Stool microscopy for ova and parasites
d. Stool C&S
e. Abdominal USG prior to referral.
3. Treatment Plan:
a. Non-Pharmacological:
i. Increase fluid intake.
ii. Treat any identified cause.
iii. Rehydration – clear fluid intake (small amounts frequently) +/- rehydration salts.
SIGNIFICANT TOPICS WITH SAQs 91
iv. Food – bland diet (especially banana and yoghurt), avoid dairy products. Routine milk
feeding for breastfed/formula fed babies.
v. Admit if elderly, unable to take orally or greater fluid loss, i.e., associated frequent
vomiting.
b. Pharmacological:
i. ORS 10 ml/kg with each stool, Enflor sachet half BD for 3 days.
ii. If fever, antipyretic – Syr. paracetamol q. 4-6h.
iii. Antibiotic (according to cause)
4. Follow-up: Afer 5 days if not relieved.
5. Referral Criteria: Refer the patient if:
-History of seizures
-Abdominal distension
-Oliguria or anuria.
SIGNIFICANT TOPICS WITH SAQs 92
55.
A 25 year old labourer presents with the complaint of severe pain in the right iliac fossa since morning.
There is associated nausea and vomiting.
ANSWER
A
Further Questions in History:
I 'll ask about:
-Location of pain and from where pain started.
-Nature and severity of pain.
-Associated symptoms like epigastric pain, vomiting.
-h/o fever
-does pain aggrevate on movement like bending, walking or coughing.
-h/o anorexia, constipation or diarrhoea.
-urinary complaints ike dysuria, burning micturation, frequency, urgency or renal colic.
-past history of appendicitis.
B
Differential Diagnosis:
1. Acute appendicitis.
2. Acute gastroenteritis
3. UTI (acute pyelonephritis)
4. Crohn's disease.
5. Ureteric colic
C
MANAGEMENT:
1. History: ask questions as described in part A.
2. Examination:GPE:General look ,check for signs of dehydration (like dry mouth,sunken eye
tongue,skin turger),pallor.
Vital Signs:check BP,Pulse,Temperature
Abdominal examination:
Look for abd.distension.
Palpate abdomen for tenderness.Rebound tenderness and guarding in right iliac fossa(especially
over McBurney's point).
3. Investigations:
-CBC,ESR
-Urine d/r
-S.urea/electrolytes
-USG abdomen
Treatment:
Reassure and explain to patient about possible disease on basis of history and examination.(most
likely acute appendicitis).
SIGNIFICANT TOPICS WITH SAQs 93
Plan:
Refer to surgery.
Meanwhile give following conservative treatment.
-Maintain I/V line
-Give I/V fluids if dehydrated.
-Give I/V antiemetics.
-Nothing per oral
SIGNIFICANT TOPICS WITH SAQs 94
56.
A two year old girl has diarrhea with blood in the stools. She also has fever and is irritable.
ANSWER
A
Questions to be asked from mother: I will ask;
1. Frequency, volume and consistency of stools, odor.
2. Any mucus in the blood.
3. Any abdominal pain and cramps (excessive crying and irritability).
4. Any nausea or vomiting.
5. Any unhygienic diet, pain related to food intake.
6. Onset, duration, characteristic of fever.
7. Dysentery in family members.
8. Any seizures, signs of confusion, difficulty breathing or any other non-GI symptoms.
9. Recent travel history of the family.
10. Volume and colour of urine.
B
Management:
1. History: See above.
2. Examination:
• GPE: General look, signs of dehydration.
• Vital Signs
• Abdomen: For abdominaldistension, tenderness, rebound tenderness, and guarding.
• Examine chest, CNS for extra-GI manifestations of dysentry.
3. Investigations:
a. FBC, ESR
b. Serum electrolytes
c. Stool microscopy
d. Stool C&S
e. USG abdomen
4. Treatment Plan:
a. Non-Pharmacological:
i. ORT.
ii. Proper hygiene measures
iii. Wash hands after going to toilet.
b. Pharmacological:
1. Admission for IV fluids if indicated.
2. Antipyretics, e.g., paracetamol/ibuprofen.
3. Oral or IV antiemetics as indicated.
4. Antibiotics if severe case (according to suspected cause)
1. Amoebic dysentery:
SIGNIFICANT TOPICS WITH SAQs 95
a. Metronidazole (200 mg/5 ml) 30 mg/kg TID x 5 days in intestinal infection (for 5–
10 days in extra-intestinal infection) OR
b. Tinidazole 50–60 mg/kg (max. 2 g) once daily for 3 days or 5 days if hepatic
involvement.
2. Bacillary dysentry (shigellosis)
a. Azithromycin 10 mg/kg OD for 3 days OR
b. Ciprofloxacin (20 mg/kg) Suspension (250 mg/5 ml) BD for 5-7 days.
c. Alternatives (if micro-organism sensitive)
i. Amoxicillin 250 mg 3 times daily for 7 days or
ii. Trimethoprim 50 mg twice daily for 7 days
3. Campylobacter jejuni gastroenteritis
a. Clarithromycin 125 mg BD (if 12-18 kg) or 62.5 mg twice daily (8-11 kg) for 7 days
OR
b. Ciprofloxacin (dose as above)
4. E. coli infection (mainly ciprofloxacin or azithromycin – see above)
5. Referral Criteria: Refer if:
-History of seizures
-Abdominal distension
-Oliguria or anuria.
SIGNIFICANT TOPICS WITH SAQs 96
57.
A 40 year old lady P2+0 presents with epigastric pain on and off of last six months. The last episode was
one day ago. The pain is colicky and is associated with vomiting.
ANSWER
A
Points on History:
Symptom classical presentation
Pain
• Duration: Usually lasts more than 30 min often
• Severity: severe
• association with fever, rigors. Exclude (acute cholecystitis, cholangitis)
• radiation to the back: biliary colic, gallstone, pancreatitis
• constant or not
• pale(jaundice)
• dark urine
other symptoms
• indigestion
• intolerance to fatty foods
• or general abdominal discomfort
• nausea flatulence
drug history
• use of OCPs/HRT/clofibrate/octreotide
addiction
• smoking alcohol
B
Relevant Steps of Examination
General examination
Jaundice+- toxic look
Abdominal tenderness +-guarding increase on deep palpation Murphy’s sign + biliary colic
Bowel sounds sluggish or absent in gallstone ileus
C
Most Likely Differentials
• Peptic ulcer disease
• Gastritis
• IBS
• GERD
• Cholecystitis
• Pancreatitis for other causes
• Tumors of gallbladder, stomach, liver, gut and pancreas
SIGNIFICANT TOPICS WITH SAQs 97
• Acute hepatitis
• IBD
• Bile duct stricture
• IHD
D
Management plan
History examination as above
Concomitant medical problems
Dehydration, pregnant, diabetes, Addison’s disease, Crohn’s disease
Hypertriglyceridemia
Nonpharmacological
• Low fat diet
• Explain about the complications danger signs
• Explain investigations, procedure,expected to be done in tertiary care
• Persistent digestive symptoms after surgery are common
• There are no dietary restrictions after surgery
Pharmacological treatment
• Pethidine 50 mg IM/PO
Naproxen 500 mg PO+ prochlorperazine 12.5 mg IM
Or domperidone 10 mg PO/PR for nausea
• If biliary colic is suspected; start
Tab. Ciprofloxacin 500 mg BD
Tab. Buscopan BD after maintaining IV line and NPO and refer the patient.
Referral and admission
• Persistence of symptoms despite analgesia
• Suspicions of complications
• Medical problems(dehydration, pregnant, DM, Addison’s disease)
Safety netting
Following protocol give analgesia and advise immediate consultation if symptoms persists
Procedures expected in secondary care
MRCP or EUS (endoscopic ultrasound)
SIGNIFICANT TOPICS WITH SAQs 98
58.
A 45 years old lady attends your clinic with history of pain in her abdomen off and on for the 5 years.
Her pain is usually relieved by passing stool with mucus and flatus. She also complains of bloated
abdomen, which is worse when she is tense. There is no history of passing blood per rectum and no loss
of weight. A sigmoidoscopy examination in her past was normal.
ANSWER
A
Most Likely Diagnosis:
Irritable bowel syndrome
B
Relevant Examination:
• General Physical Examination
• Examination of oral cavity.
• Detailed Examination of GI system including perrectum examination.
• Examination for thyrotoxicosis.
• Mental state Examination.
C
MANAGEMENT:
1) History
a. Rome III criteria
b. Medical history including herbal history, family history.
c. Screen for depression.
2) Physical Examination: As above.
3) Investigations: CBC, Stool for C&S and O&P, fecal occult blood test (FOBT)
4) Nonpharmacological treatment
Discuss the diagnosis and prognosis with the patient.
Address the patient’s fear and provide reassurane.
Encourae consumption of healthy diet and sufficient fluid intake.
Avoid fast foods, excessive caffeine, alcohol, sorbitol (in gum and candies) and fructan.
Encourage to keep a diary of foods that trigger or relieve symptoms and note any
stressor.
Ensure sufficient dietary fibre if constipated.
Stress management, relaxation advice, quite time for eating and defecation.
2) Pharmacological Treatment::
Classify patient into IBS with constipation, IBS with diarrhea or IBS alternating between
constipation and diarrhea (using Bristool Stool form scale)
Treat the most troublesome symptom.
IBS with predominant diarrhea (Bristol Stool Form Scale type 6 or 7): Loperamide 2 to 4 mg
PO PRN.
SIGNIFICANT TOPICS WITH SAQs 99
IBS with predominant constipation (Bristol type 1): 2-4 tablespoon of raw bran or Psyllium
daily with plenty of fluids.
If not responding to laxative then lubiprostone for abdominal pain. Tablet amitriptyline 25
mg HS PO
Treat underlying depression if present.
Scheduled regular follow-up for ongoing support and reinforcement.
SIGNIFICANT TOPICS WITH SAQs 100
59.
A 38 year old woman presents with epigastric and retrosternal burning since 1 month.
ANSWER
A
Further Points in History:
• Any h/o regurgitation of food or sour liquid especially at lying down or bending.
• H/O drug cough, sore throat or feeling of lump in throat.
• Any h/o aggravating factors, spicy food or relieving factors, milk or antacid.
• Any associated symptoms, vomiting, diarrhea, epigastric pain.
• Red flags: Any h/o weight loss, haematemesis, dark (black stool, melena), dysphagia,
hoarseness, jaundice or dark urine.
• Any h/o previous attack. If yes, then for that h/o DM, HTN.
• Medication history: NSAIDS, steroids, SSRI
• Any h/o stress + anxiety.
• Any h/o smoking/alcohol
• Dietary habits/level of physical activity.
• Menstrual history: LMP (to rule out pregnancy)
B
Physical Examination Features
• General Physical Examination:
o Check for anaemia, pallor
o Lymph nodes in neck especially supraclavicular lymph nodes (Virchows lymph node)
o BMI
• Abdominal Examination: Epigastric tenderness/mass, hepatosplenomegaly
• Palpate abdomen to exclude pancreatitis, cholecystitis, nephrolithiasis.
• CVS examination like S3, S4, pulmonary rales and rhythm.
C
Nonpharmacological
Lifestyle
o Lose weight if obese.
o Stop smoking (if smoker) + alcohol
o Stay active.
Sleeping Advice:
o Elevate head end of patient’s bed by 6 injches e.g. by wedge pillow.
o Avoid large meal and reclining after meal.
o Take evening meal 3 hours before going to bed.
Diet:
o Avoid foods, which aggravate the symptoms, healthy nutrition.
Avoid NSAIDS + aspirin
Pharmacological
SIGNIFICANT TOPICS WITH SAQs 101
60.
A 25 years old male comes to your clinic with complaints of watery diarrhea with blood and mucus, for
last 3 days. He has associated generalized abdominal discomfort and is running temperature of around
101 to 102 F. He gives history of eating from roadside 4 days ago.
ANSWER
A
Likely Differentials:
❖ Dysentry due to Campylobacter
❖ Dysentry due to E. coli
❖ Dysentry due to Shigella
❖ Parasitic infection by Entamoeba histolytica
B
Options for Antibiotics:
1. Tab. Ciprofloxacin 500 mg BD for 3 days
2. Tab. Metronidazole 400 mg TDS for 3 days
C
Supportive Measures:
1. Drink boiled water.
2. Use ORS, especially drink 1 cup of ORS after each diarrhea episode.
3. Eat soft healthy food like khichri, banana, yoghurt.
4. Dietary modifications like KYB/BRAT
5. Avoid uncooked food.
6. Advice hand washing after going to toilet.
7. Provide all dietary and medication advice in written.
8. Consider antispasmodic to relieve discomfort.
SIGNIFICANT TOPICS WITH SAQs 103
61.
An 18 years old young man presents with six hour history of severe right lower abdominal pain. It
started around the umbilicus and later moved to right lower abdomen. He has no urinary symptoms.
He has a temperature of 100 F, pulse 110/min, regular and tenderness in right iliac fossa with rebound
ANSWER
A
Most Likely Diagnosis:
Acute appendicitis
B
Management Plan:
• Detailed History: In addition to history points given in scenario, ask for;
o Is the pain colicky or continuous/localized.
o Does it get worse on coughing, walking, or moving?
o Association with symptoms like nausea, vomiting, anorexia and fever.
o Noticeable change in bowel habits, (constipation – typical, diarrhea – rare), blood in
stool.
o Urinary symptoms – burning, dysuria,frequency
o H/O drugs (e.g. aspirin), smoking, heavy alcohol intake and any addiction (e.g. heroine,
cocaine)
o Past medical history, combordities.DM
o Any previous abdominal surgery.
• Examination:
o General appearance – unwell, ill looking, facial grimace, flushed, stooped walking.
o Vital Signs: Fever (temperature), pulse, RR, BP
o Oral cavity examination for furred tongue, fetor oris
o Skin – pallor, sweating
o Abdominal Examination: After informed consent, ask the patient to lay straight with
knees bent.
▪ Expose patient adequately
▪ Look for facial expressions (grimace) while proceeding with examination.
▪ Inspect: Abdomen and back for any bruises and surgical scar.
▪ Auscultate: For bowel sounds.
▪ Percuss: For succusion splash.
▪ Palpate: All quadrants. Note for:
• Involuntary guarding and rebound tenderness
• Suprapubic tenderness
• Check for psoas sign, obturator sign and Rovsing’s sign.
o Perform DRE: Tenderness anteriorly over the right aspect = pelvic appendix
• Investigations:
o Labs:
▪ Urgent CBC – leukocytosis
SIGNIFICANT TOPICS WITH SAQs 104
▪ CRP – raised.
▪ Urea and electrolytes – assessment of dehydration.
▪ Urine R/E
o Imaging Techniques:
▪ X-ray Erect Abdomen – local distension, blurred psoas shadow, fluid level in
caecum.
▪ USG Abdomen – Non-compressible viscus, thickened appendiceal wall,
increased diameter of appendix > 6 mm
• Treatment Plan:
o Non-Pharmacological:
▪ Explore and address patient’s ICE
▪ Health education and patient reassurance.
▪ Explain pt that my clinical diagnosis is acute appendicitis in which part of gut
(appendix) gets inflamed and for this reason I am going to refer to general
surgeon for appendectomy
o Pharmacological:
▪ Adequate documentation + analgesia
▪ Urgent referral for appendicectomy
▪ In case of unavailability of prompt surgery services:
• Keep NPO
• Maintain IV line + IV NS
• INJ metronidazole 500 mg IV stat then TDS
IV Cefuroxime 1.5 g IVstat thenTDS
• Inj. Metoclopramide 10 mg stat IM
• Documentation of GP services + arrangement for transferring patient
to surgical unit at earliest.
• Follow up after 1week if condition allows
SIGNIFICANT TOPICS WITH SAQs 105
62.
A 34 years old man presents with painful defecation and stools streaked with blood since 5 days. He
gives a history of chronic constipation.
ANSWER
A
Most Likely Cause of Symptoms:
Chronic constipation resulted in anal fissure.
B
Management:
o History:
▪ Take history of patient regarding anal fissure and constipation.
▪ Pain duration: before or after defecation.
▪ Any change of stool color
▪ Frequency of bowel movement, > 3 days in passing stool.
▪ Consistency of stool, soft or hard.
▪ Pain with defecation or bleeding.
▪ Any straining
▪ Abdominal discomfort or distension
▪ H/O IBS
▪ Urinary symptoms like frequency and urgency
▪ Dietary History: Type and quantity of liquid consumption.ask about food ,intake of fast
food?
▪ H/O weight loss,weight gain fatigue, depression(to rule out hypothyroidism)
▪ Past history of surgery, exercise frequency, sexual abuse, unusual sexual activity.
▪ Medical history of hypothyroidism, neurological or spinal cord injury.
▪ Family history of colorectal disease or cancer.
▪ Drug history of opioids, antacids, laxatives, antidepressants, anticonvulsants.
o Examination:
▪ Vitals: BP, temp, pulse, R/R
▪ GPE: J, A, C, P, O, LN, thyroid.skin rash
▪ Abd examination
▪ Inspection of anal mucosa, avoid digital rectal examination
o Investigations: At this stage, no investigations required, but if history indicates, then according
to investigations required: CBC, TSH, electrolytes.
o Treatment:
▪ Non-Pharmacological:
• Lifestyle Changes
o Increase fluid intake 8-10 glasses of water
o Exercise
o High fiber diet
o Allow enough time for bowel movements.
SIGNIFICANT TOPICS WITH SAQs 106
o Sitz bath
• Pharmacological:
o Stool softners (e.g., Ispaghula husk) two table spoon in 1glass of luke warm water
two times a day
o Analgesic suppositories 5% lidocaine ointment OTC haemorrhoid preparation or
o 0.4% nitroglycerine ointment application.
o Syp lactulose 30ml HS
o CONTINUE TRATMENT FOR 1WEEK.follow in 1week
o If pt will not improve then will arrange routine referral to general surgeon
SIGNIFICANT TOPICS WITH SAQs 107
63.
A 40 year old obese, married female presents in your clinic with the complaint of upper abdominal pain
associated with fever and vomiting since last night. Her symptoms started after having dinner at a
party. On examination, she is febrile (39 C), heart rate 110/min and blood pressure is 90/60 mmHg.
A. Enlist four relevant steps of examination you will perform in this patient?
B. What is the most likely diagnosis?
C. Enlist your management plan.
ANSWER
A
Relevant Steps on Examination
1. Vital Signs: Pulse, BP, temp (fever), R/R, weight of patient.
2. GPE: Overall appearance of the patient, weight (obese patients more at risk), anaemia,
jaundice.
3. Look, palpate and examine the abdomen for tenderness, palpable mass (gallstones)
4. Look for Murphy’s sign: Inspiration is inhibited by pain when examiner’s hand is placed along
costal margin.
5. Pain referred to shoulder or interscapular region.
6. Look for signs of complications of cholecystitis like biliary peritonitis, perforation of gallbladder,
pericholecystic abscess, etc.
B
Most Likely Diagnosis:
Acute cholecystitis (most likely due to gallstones, only 5% acute cholecystitis pts are acalculous)
C
Management:
Detailed History:
• History of gallstones
• Past medical history of diabetes mellitus, use of oral contraceptives, HRT, Crohn’s
disease,sickle cell disease
• History of smoking,alcohol
• Family history of gallstones
Examination: Please see above.
Investigations:
• CBC and ESR (to confirm inflammatory response)
• C-reactive protein (see above point)
• Serum amylase
• Abdominal USG (gallbladder is typically enlarged with a thickened wall, containing stones or
• debris, in the presence of a sonographic Murphy's sign)
NON PHARAMA treatment:
• Explain pt s ICE
• Counsel pt regarding diagnosis and let her know she mights need surgery.
•
Treatment Plan: Urgently admit/refer anyone with suspected acute cholecystitis for:
• Confirmation of the diagnosis (lab tests, imaging)
SIGNIFICANT TOPICS WITH SAQs 108
64.
A 16 year old boy presents with severe itching on his hands and body for the last one week. Itching is
worse at night. Examination shows papular lesion on wrists and abdomen with marks of excoriation.
ANSWER
A
Most likely Scabies
B
Management
Confirmation of diagnosis
• detailed hx
Is itching progressive in nature
- ask for close contacts with scabies
- itching worse at night
- known hx of atopy or allergies(food allergy)
- any immunocompromised state
• Relevant exam (wear sterilized gloves while examining given the contagious nature of the
disease)
- looking for burrows, erythematous, inflamed pustules and papules especially in between finger
webs
- checking other areas like wrist, axillae, chest, umbilicus, ankles and genital area
- secondary excoriations from scratching
Treatment
Nonpharmacological:
• Explore pt’s ICE
• Explanation and health education
• Specific instructions regarding sterilisation and disease containment
- Sterilise exposed laundry in hot water or discard them
- treatment for all intimate contacts and close household and family members
- specific instructions regarding use of treatment
- pt should be cautioned not to overuse the medications
- provide written info if available
• Close follow up after 2 weeks and then 1month.
• If symptom persist then reffer to the infectious diseases specialist
Pharmacological:
1-PERMETHRIN 5% cream and lotion: After bathing or showering, apply cream from the neck to
the soles of the feet; then wash off after 8 to 14 hours and repeat in 1 week.
2-1%lindane cream or lotion
3-10%sulpher ointment
4-25%benzyl benzoate cream
5-wide spread aur extensive scabies or norwagian scabies treated with ivermectin.
Safety Netting
SIGNIFICANT TOPICS WITH SAQs 110
65.
A 20 year old unmarried girl presents with new onset of papules and pustules on face since 1 month.
They are mildly pruritic. She also reports and excessively oily skin.
ANSWER
A
Pharmacological management
1- BENZOYL PEROXIDE 2.5 % topical gel twice daily
• strength and freq to be built up gradually as needed
PLUS
2- topical retinoids (isotretinion) low strength preparation topical gel bedtime
plus
2- CLINDAMYCIN 1% topical soln. Once or twice daily for at least 4 weeks
Treatment for 4 weeks if not improved then will start oral antibx
B
Non-pharmacological advice
1- Reassurance that it is treatable and needs pt’s active participation and, compliance to
treatment and follow ups
2- Explanation
• Explore and address pt’s ICE
• provide information about her illness
• inform that the treatment will take approximately 3-4 weeks to show desirable results
4- Advices and instructions
• Healthy skin care routines like
- proper cleansing
- avoid excessive face washing and limit it to twice a day
- avoid harsh soaps
- avoid aggressive scrubbing
- avoid exposure to harsh UV lights
- use of less oily sunscreens
- use of aqueous-based fragrance-free emollients
• the acne shouldn’t be picked up or squeezed in order to prevent scarring and
hyperpigmentations
• proper and regular use of medications as prescribed
5- Discuss treatment methods suitable for her, most likely side effects and how to deal with them.
6- FOLLOW UPS 4 weeks to check response and adherence to treatment and to address non
adherence issues if any
SIGNIFICANT TOPICS WITH SAQs 112
66.
A mother brings her 6 months old infant with nappy rash.
ANSWER
Steps in Management of Infant’s Nappy Rash
History:
➢ Detailed history of presenting complaints.
➢ Onset, duration, progression of rash, any aggravating/relieving factors.
➢ Ask symptoms, fever, appetite, bowel habits, any urinary symptoms, scratching and
crying.
➢ Feeding history, bottle feed/weaning history.
➢ Ask about hygiene of baby
➢ How many diaper change per day.
➢ Type of diaper; cotton or plastic diaper
➢ Recent illness, diarrhea, antibiotic use.
➢ Use of creams, ointments to cleanse diaper area
Examination of baby:
Examine baby from head to toe
Anterior fontanel depressed, signs of dehydration
Any skin rash apart of nappy rash
Check vital signs, esp. temperature to rule out infection.
Local Examination of periumbilical/genital area
Describe primary lesion morphology
Describe distribution of rash
Secondary lesion, i.e., scratch mark
Inflamed skin in diaper area
Genitocrural folds are spared in diaper dermatitis
Marked discomfort from intense inflammation
Mild forms consist of shiny erythema with or without scale.
Margins are not always evident.
Moderate cases have areas of papules, vesicles and small superficial erosions.
Diagnosis/Investigations:
It is basically a clinical diagnosis.
But in moderate to severe dermatitis cases, order baseline management like;
• CBC, CRP
• Blood C&S (to rule out infectious cause)
Treatment Plan:
Nonpharmacological Treatment:
Educate parents (mother) or caregiver about nature of disease.
Address parents’ ICE
Advice mother to;
change baby’s diaper often.
Keep the diaper loose enough to let air reach the inside of the diaper
SIGNIFICANT TOPICS WITH SAQs 113
Gently clean the affected skin with warm water. Put gently with clean and soft
towel.
Do not use wipes that contain alcohol or perfumes.
If mother using cloth diapers, wash them carefully with hot water and rinse
carefully.
Pharmacological Treatment:
First-Line Therapy:
Zinc oxide ointment applied topically to area involved 3 to 4 times per day.
Castor oil application.
Moderate to Severe Cases: Can prescribe combination therapy with antifungal + low
potency steroids like clotrimazole + hydrocortisone 1% topical 2 to 3 times per day.
Follow-up: With GP/paediatrician in 1-2 days
SIGNIFICANT TOPICS WITH SAQs 114
67.
A 10 year old child from a poor socioeconomic background is brought by his mother to your clinic. She
says that he has a rash on his legs, elbows and in the webs between the fingers since the last 2 months.
The rash is very itchy especially at night. She has four other children.
ANSWER
A
Most Likely Diagnosis
Scabies
Atopic eczema
B
Key Nonpharmacological Points
1-reassurance and counselling.
2-keep general hygiene measure.
3-stop sharing fomites
4-avoid skin to skin contact with other person.
5-keep trimmed nail short to prevent skin infection.
6-avoid scratching lesion again and again.
7-daily changes of clothes and bed linen
• All close contacts need treatment simultaneously, all occupants of residential homebeing
treated
• Apply treatment to whole body including scalp neck face and ears
• Ensure finger toe webs are covered
• Brush the lotion under the ends of fingers and toe nails
• Reapply to the whole body after one week
• And to the hands alone if washed with soap < 8 hours after the application
• Advise the patient to launder all worn clothings, towels and beddings used in last week in hot
water 60degree celcius or higher. personal use items that cannot b washed must be sealed in
plastic bag for 1 wk.
• Inform that Itching may persists for some time after the elimination of infection
• Allow lotion or cream to dry before dressing
C
Pharmacological Treatment
• Treat with permethrin 5% (8-12 hrs) or malathion lotion(24 hrs) Explain written and verbal
application
• Repeat after one week
• Oral sedating antihistamines like hydralazines to break ictch scratch itch cycle.
• Followup after two weeks n one month.
• Referral
• If two courses of insecticide failed
• Crusted Norwegian scabies recommend hospital admission
SIGNIFICANT TOPICS WITH SAQs 115
• or secondary lesions(cellulitis,folliculitis,boils,impetigo).
SIGNIFICANT TOPICS WITH SAQs 116
68.
25 years old Miss Rahila presents to the clinic with history of recurrent pruritic lesions occurring over the
body which disappear in a few hours since 2 months. She describes them as itchy red swelling. She gets
them 3-4 times every week. There is no associated facial swelling. On examination, you notice
erythematous wheals on her limbs and trunks. She has no further complaints.
ANSWER
A
Most Likely Diagnosis:
Chronic urticarial rash
B
Tests:
• CBC + ESR
• TFTs
• RBS
C
Management:
Detailed History
• Ask her about how rash appeared, small or large, site.
• Any h/o difficulty in breathing or associated with swelling of face.
• Drug history – sensitiy to any medicine like aspirin.
• Any triggering factor – like stress or overheating.
• H/O asthma
• Family history of allergy or asthma
• Relation to menstruation
• Associated fever with rash
• Relation with exposure to sunlight.
• Food history – eggs
• Dyes
Physical Exam:
• GPE – irritable, depressed, anxious. Look for swelling of face.
• Vital Signs – Temp., pulse, BP, RR
• Chest – auscultate for bronchospasm, wheeze
• Site of rash – distribution, itchy?, painful?
Nonpharmacological Treatment:
• Avoid triggering/aggravating factors
• Reassurance, education and address ICE
• Avoid overheating + sun exposure.
• Avoid stress and aspirin and foods that may cause it.
Pharmacological Treatment:
SIGNIFICANT TOPICS WITH SAQs 117
69.
A mother brings her three years old child with history of multiple pustules and boils on face and scalp
since one week. She reports that her younger child also had similar lesions. The child is otherwise well.
ANSWER
A
Most Likely Diagnosis:
Staphylococcal infection - impetigo
B
Management
Detailed History:
• How did the boils start.
• Ask about history of atopy or allergy.
• Socioecononic history
• Use of soap for cleaning
• Associated with fever
• Previous history of similar infection.
• Drug history
• Frequency of bathing + use of water – tanker or community tap
Examination:
• Site, size, any swelling or redness (cellulitis)
• Pustules – large pustules need to be drained.
Nonpharmacological:
• Mother’s Education:
o Address mother’s ICE
o Explain that disease is contagious but non-scarring.
o Explain about direct and indirect spread. Direct with discharge from the scabs of an
infected person; indirect by hands, toys, clothing, equipment and environment.
• Good hygiene practice
• Wash face + hands with soap
• Separate towels for all children daily.
• Reassurance – self limiting disease.
• Advise CEA bath
• Linen + clothes of children to be kept clean – and change frequently.
Pharmacological:
• Application of fucidic acid or neomycin with polymyxin sulphate 2 times daily.
• Topical retapamulin
• Topical mupirocin (Bactroban) if MRSA
• Treat all children in the household.
• If extensive disease:
o Oral flucloxacillin (5 ml (125 mg) QID)
SIGNIFICANT TOPICS WITH SAQs 119
70.
A 17 year old college student attends your clinic as she is concerned about comedones appearing on her
forehead and cheeks. On examination, there are open and closed comedones on cheeks and forehead.
There are no nodules or pustules.
ANSWER
A
Most Likely Diagnosis:
Grade 1 Acne vulgaris
B
Pharmacological Management:
As case seems of mild severity, topical treatment is applied to the whole area. It includes
1. Benzoyl Peroxide bd. It is started at lowest dose and then gradually increased as needed.
▪ Side effects: irritation and dryness.
2. Topical Retinoids(e.g isoretinoids):apply low strength preparation every 2-3 nights initially
and build up strength and frequency as tolerated.
▪ Side Effects:erythema and scaling in most patients which settle with time.Acne may
worsen initially for few weeks.
3. Topical Antibiotics:Clindamycin.
▪ It's resistance is increasing.use only in combination with benzoyl peroxide or if benzoyl
peroxide has failed.Avoid if using oral antibiotics.
Duration Of Treatment:
Try topical treatment for at least 4-8 weeks to see response,if it's working continue until
New lesions stop developing.
C
Advice to the Patient: I will advise patient to:
• Wash with soap and warm water twice daily.
• don't excessively scrub the skin.
• use a fragrance free water based emolient if dry skin.
• Avoid greedy preparation
• Drink plenty of water.
• Don't pick at acne it can cause scarring.
SIGNIFICANT TOPICS WITH SAQs 121
71.
A 25 year old male comes to your clinic complaining of a patch of hair loss on his scalp which was noted
by his wife. There is no pruritus. On examination, there is a smooth patch of hair loss over the occipital
area. Tiny 2-3 mm hairs are present at the margin of the patch.
ANSWER
A
Differential Diagnosis:
❖ Alopecia areata
❖ Trichotillomania
❖ Tinea capitis
❖ Scarring alopecia/posttraumatic alopecia
B
Pharmacological Advice: For this specific case with suspected signs of alopecia areata, I will advise
following treatment plan (fungal infection should be excluded)
• Watchful waiting
o Condition is self limiting. Best plan is to reassure patient and to encourage patience.
• Referral to dermatologist for the use of intralesional corticosteroids. ICS is the most
effective treatment option for patients with non-extensive alopecia areata
o Less than 0.1 ml injection per site and injections are spread out to cover the affected
areas (approximately 1 cm between injection site). Regrowth of hair minimally desired
3 months.
o Injections are administered every 4 to 6 weeks.
o Adverse Effects: Pain during injection and minimal transient atrophy.
• Topical corticosteroids and topical minoxidil can be used on this patient (over 6 years of
age) as patient is 25 years old while waiting for dermatologist referral of if patient wants
only primary care treatment.
o Topical corticosteroids include;
▪ Fluocinolone acetonide L/A cream 0.2% two times per day at least for 3 months (OR)
▪ Betamethasone dipropionate cream 0.05% L/A two times per day.
o Side effects: Local folliculitis
• Minoxidil 5%:
o 20 drops 2 times per day L/A or affected area at least for 12 weeks.
o Continue until satisfactory hair growth observed at affected area
o Side Effects: Distant hypertrichosis and irritation.
C
Counselling:
• Address patient’s ideas, concerns and expectations (ICE).
• Describe patient about benign nature of disease.
• Let patient know that in most cases disease is self limiting.
• Provide psychosocial suspport with frequent counselling visits.
SIGNIFICANT TOPICS WITH SAQs 122
SIGNIFICANT TOPICS WITH SAQs 123
72.
A mother brings her 16 year old son with a rash on his right leg for one week. Rash is itchy. There is no
associated pain or discharge. It is confined to the right leg only.
On examination, there is a circular, well circumscribed rash on right shin with borders and mild
erythema.
ANSWER
A
Tinea corporis
B
non pharmacological
#Education and Reassurance
# do not scratch rash area
# wash hand after handling pet
# keep good hygiene
# daily bath/shower
# trim nails to avoid secondary infection due to itching
Pharmacological
# local terbinafine cream twice a day for 3 - 4 weeks
# oral terbinafine 250mg x OD for 4wk if immunocompromised or extensive
C
preventive measures
# good hygiene
# daily bath
# proper hand washing
# avoid towel sharing
# avoid hot humid conditions
# loose fitting clothes
SIGNIFICANT TOPICS WITH SAQs 124
73.
A 3 year old child is brought to you with a history of rashes off and on over the flexor aspects of the
elbow,dorsum of hands and behind the ears since early childhood. His mother is a bit distressed as the
child scratches himself and cries at night.
ANSWER
A
Most Likely Diagnoses:
Atopic dermatitis
Scabies
B
Management:
Detailed History
• Onset, course, duration of symptoms.
• Aggravating factors: soaps, perfumes, specific clothing, food, weather, pets.
• Relieving factors: Bathing
• Associated symptoms: Asthma, h/o allergic rhinitis, food allergies, fever.
• h/o symptoms complaints before. If yes how often
• Family history of atopy (eczema, asthma, etc.)
• Impact on quality of life: Feeding, gaining weight.
Examination:
• GPE (height, weight), vitals signs, lymph nodes.
• Chest: Auscultation.
• Local Exam: Site of rash, morphology, pattern, signs of secondary bacterial infection,
erythema, sclaes, excoriation, burrowing lesions.
Treatment:
• Education and avoidance of triggers, irritating soap, perfumed products, wools and
synthetic fibers, dry grass, leaves, overheating.
• Apply non-irritating emollients BID and after bathing (if no eczema patch)
• Presence of dermatitis: Topical application of
o Hydrocortisone 1% BID behind the ear for 2 weeks.
o Betamethasone valerate 0.05% OD on hands and flexural aspects of elbows.
o Apply corticosteroids after removing emollient from the patch.Topical applic
• Petroleum gel BD – on rest of body parts and especially after warm bath
• Avoid hot bath
• If symptoms not relieved after 2 weeks, can try Tacrolimus ointment 0.03% BD for 2 weeks.
• Antibiotics in case of secondary bacterial infection.
• Wet wrapping on the affected side.
• Referral – if not responding to topical therapy.
C
Advice to Mother:
SIGNIFICANT TOPICS WITH SAQs 125
Mother should be informed about chronic recurring condition that can be controlled, not cured,
however, some children grow out as they grow old.
Remission by 15 years is 75%.
SIGNIFICANT TOPICS WITH SAQs 126
74.
A 36 year old female previously healthy comes with complaints of lethargy, weight gain, constipation
and irregular periods for the last 6 months. She is a school teacher and finds that the fatigue is affecting
her ability to work. Physical examination shows an overweight lady with a BP of 135/98 mmHg, HR of
62/min and dry skin. Rest of the examination is unremarkable.
ANSWER
A
Most likely diagnosis:
Hypothyroidism
B
Differential Diagnoses:
1- Polycystic ovarian syndrome
2- Depression
C
Investigations
1- serum TSH, free T4
• elevated serum TSH with low free T4
2- FBS +/- HbA1c
• screen for DM
3- serum free testosterone
• to screen for pcos
D
Management
1- Confirmation of diagnosis by detailed hx , physical examination and investigations as above
2- Pharmacological
- Consider to start treatment if TSH >7.0 mU/l in this age with signs and sx of hypothyroidism
• LEVOTHYROXINE 50-75mcg once daily
( advise to take it with water first thing in the morning on empty stomach 30-60 min before
breakfast)
3- Nonpharmacological advice
- Reassurance and patient education regarding the illness, need for treatment and regular initial
follow ups to adjust the thyroid replacement therapy
- inform that the sx may take time to subside completely
- informed about the possible side effects of the therapy and how to deal with it
- advice on healthy lifestyle and weight reduction strategies
4- Follow up in 6-8 weeks with repeat TSH and to adjust treatment,
- earlier visit to monitor BP
SIGNIFICANT TOPICS WITH SAQs 127
75.
A newly diagnosed diabetic, 45 year old executive presents at your clinic with a fasting blood sugar of
160 mg/dl, blood sugar 2 hour post prandial is 220 mg/dl
ANSWER
MANAGEMENT:
1. History;
• Explore and assess patient’s ideas, concepts and expectations regarding the disease
• ask about general state of health
• Detailed history focusing on active symptoms esp those of complications
• Enquire about
- any vision impairment,
- numbness or abnormal sensations,
- foot problems,
- weight or appetite changes, dizziness,
- fatigue,
- dyspnea and palpitations,
- edema,
- urinary sx/ incontinence or retention,
- abnormal bowel habits
• Identification of other risk factors
• Other known comorbs, smoking/ alcohol, lifestyle routine, activity level
• Family hx; DM, premature CHD, hyperlipidemia, Htn
2. Clinical examination;
• General and vitals, BMI,
• Waist circumference,
• Skin,
• Eye,
• cardiovascular,
• Neurological, foot exam,
• Urine dipstick
3. Investigations;
• Basic metabolic panel (blood glucose, urea, creatinine, electrolytes),
• HbA1c,
• CBC, ferritin
• liver and renal profile,
• coronary risk profile (lipid panel),
• Urine albumin creatinine ratio ACR, urinalysis
• baseline 12 lead ECG
4. Nonpharmacological;
SIGNIFICANT TOPICS WITH SAQs 128
• Patient education
- Reassurance, education and counseling for patient and family members/ care giver
- Provide written information and knowledge
- Discuss treatment goals for control of diabetes as well as BP and lipid management.
- Identification of hyperglycemic and hypoglycemic sx
- Diabetes self-management education/ self-monitoring of plasma glucose
- Discuss the benefits and risks of drug treatment, and the options available.
- Discuss medications side effects
- discuss and educate about foot care
• Healthy lifestyle modifications;
- choose healthy alternatives in diet with high fiber, low salt, low simple carbs and reduced
saturated and trans-fat intake
- adequate hydration
- increase physical activities
- If smoking, encourage cessation and refer for related program
• Review from diabetes educator, dietician and ophthalmologist.
5. Pharmacological;
• Drug treatment along with healthy lifestyle modifications
• Start with Metformin 500mg OD, increase dose every 1-2 weeks till 2.5g daily (to minimize the
risk of gastrointestinal side effects)
• ACEI (Lisinopril 5mg OD) to keep target BP below 140/90 or below 130/80 if there is eye, renal or
cardiovascular disease.
• Statins for primary prevention
6. Follow up care;
• Initial follow up 2-3 months for reassessment, labs and adjustment of treatment if needed.
SIGNIFICANT TOPICS WITH SAQs 129
76.
A 65 year old female with history of diabetes and hypertension, presents with fracture of right femur
after history of fall.
ANSWER
A
Most likely Reason for Fracture:
Degenerative bone disorders: “Senile Osteoporosis”
B
Risk Factors:
• Past h/o spontaneous, easy fractures.
• Early menopause – natural or surgical
• Familial (especially maternal) h/o of predisposition to fragility fractures and osteoporosis (hip,
wrist, vertebrae)
• Loss of height, kyphosis, cervical lordosis.
• Sedentary lifestyle
• Low BMI
• H/O smoking, alcohol intake, increased protein intake.
• Drug intake, e.g., glucocorticoid, long-term heparin, breast CA therapy, HRT, OCP, SSRI,
glitazones.
• Systemic disease evaluation for
o Endocrinal – thyroid, parathyroid, adenoid
o Chronic kidney disease
o Connective tissue/autoimmune disorders
o GI malabsorption syndrome.
o Visual impairment.
• Nutritional status – especially for Ca and Vit. D.
C
Investigations:
• Bone densitometry using dual energy x-ray absorptiometry (DEXA) scan.
• T-score:
o Score of -2.5 or less is diagnostic.
o Serum Ca, serum 25 ()H) vit D levels.
o Serum ALP level, PTH, creatinine, albumin, TSH
• Others:
o FBC
o ESR
o CRP
o Urea and electrolytes
o Serum protein electrophoresis (to rule out MM)
SIGNIFICANT TOPICS WITH SAQs 130
D
Management:
Detailed History & Examination
Explore and address patient’s ICE
Health education and reassurance
10-year fracture probability score: By Qfracture Score/FRAX score
BMI to be maintained at > 19 kg/m2
Non-Pharmacological Treatment:
• Lifestyle modification:
o Balanced diet – consumption of 1500 mg of diet origin calcium daily.
o Avoid high protein diet.
o Avoid tobacco, alcohol and excess caffeine.
o Prevention of further bone loss through weightbearing exercises.
• Fall prevention
o Avoiding sedatives
o Providing hip protection
o Avoid alcohol
o Avoid poor vision
o Avoid frequent, sudden posture changes.
• House modification:
o Remove scatter rugs and clutters.
o Install hand rails on steps and in washroom
o Extra light at night, particularly in washroom
Pharmacological Treatment:
• Rule out CKD first.
• Bisphosphonate – alenderonate 70 mg once weekly – take on empty stomach 30 minutes
before breakfast, sit upright for 30 minutes after taking drug.
• Calcium citrate/carbonate 500 mg PO OD
• Cholecalciferol vitamin D (2000 IU) PO
• In case of CKD or hypersensitivity to bisphosphonates, referral to orthopedics
SIGNIFICANT TOPICS WITH SAQs 131
77.
A 46 year old house wife, with known history of asthma attends with the complaint of gaining weight,
puffiness of face and high blood pressure for last 2 months. Her asthma control is much better since the
start of some hakim treatment for last 3 months. Her BMI is 28 mg/m 2, and her BP is 170/95 mmHg.
Her recent blood work showed Normal urine DR, CBC, urea, creatinine, serum electrolytes, Thyroid,
Renal and hepatic profile. Her fasting glucose is 130 mg/dl
ANSWER
A
Most Likely Diagnosis:
Cushing syndrome
B
Most Likely Cause:
Iatrogenic –exogenous steroids
C
Further History Points :
Ø Detailed medication / drug history
Ø Early osteoporotic symptoms, joint pains.
Ø Early onset &uncontrolled diabetes & hypertension
Ø Menstrual irregularities, hair growth, striae, bruising
Ø Unexplained depression & chronic headaches, polydypsia, polyuria.
Ø Difficult rising from chair as a prominent symptoms of proximal muscle weakness
Ø Easy bruising mostly without any trauma & poor wound healing
D
Examination findings
Ø Facial plethora
Ø Increased fat : distribution over upper back & supraclavicular
Ø Truncal obesity
Ø Thin extremities
Ø Skin thinning
Ø Striae, pigmentation, bruises
Ø Hirsutism& facial acne
E
Investigations
Ø ACTH levels
Ø Low dose dexamethasone suppression test
Ø 24 hour urinary free cortisol level
Ø MRI / CT
SIGNIFICANT TOPICS WITH SAQs 132
78.
A 45 year old obese lady with no previous comorbids presents for review of reports. On examination
her BMI was 28 with waist circumference of 95 cm and BP 150/90 mmHg. Systemic examination is
unremarkable. Lab investigations showed LDL 160, HDL 38, Triglycerides 250, FBS 140.
ANSWER
A
Most Probable Diagnosis:
Metabolic syndrome
B
Abnormal parameters (ATP 3 and ATP 4 criteria)
*Abdominal obesity waist circumference > 88 cm (women)
*HDL <50 mg/dl or < 1.2 mmol/L (women)
*Hypertriglyceridemia 150 mg/dl or more or > 1.8 mmol/L (fasting)
*BP SBP ≥ 135 DBP ≥ 85
*Fasting glucose ≥ 100 mg/dl
C
Management
i) Reassurance, explanation and health education
ii) Detailed history:
Ask about diet,exercise,level of activity.
Drug history,h/o steroid,b blockers
H/o smoking
Family history:of HTN,DM or metabolic syndrome
Examination:
GPE,vitals,skin xanthelesma,edema.
CVS and abdominal examination
Sensory examination for peripheral neuropathy.
iii) Investigations: Prior to starting statin therapy.
• Baseline blood tests: LFTs, CK, renal function (including eGFR), HbA1c, TSH
• Cardiovascular risk assessment
iv) Nonpharmacological advice
• Aggressive therapeutic lifestyle interventions;
• Increase physical activities, at least 30 min of daily, moderate-intensity physical activity
• Healthy dietary modifications
o -Low intake of saturated fats trans fats and cholesterol
o -Increase consumption of fruits vegetables and whole grains in diet
o -Reduced intake of simple sugars
• Dietician review
iv) Follow-ups to check targets.
SIGNIFICANT TOPICS WITH SAQs 133
v) Pharmacological intervention
• Add drug therapy for lipids and non-lipid risk factors if targets not achieved with lifestyle
interventions alone.
• Offer a high-intensity statin treatment with atorvastatin 20 mg unless contraindicated (e.g.
in pregnancy)
• Address patient’s ICE regarding statin therapy, i.e., it decreases risk of CVD, has mild and
reversible side effects and severe side effects like rhabdomyolysis and myopathy are rare.
• Advocate and ensure compliance with drug as well as with lifestyle measures stated above.
• If statins are contraindicated, seek specialist advice.
SIGNIFICANT TOPICS WITH SAQs 134
79.
A 42 year old female presents to your clinic with a 3 months history of “Ghabrahat,” weight loss,
anxiety, tremors and diarrhea. On examination she has a pulse of 115/min, fine tremors of outstretched
hands and a smooth diffusely enlarged thyroid.
ANSWER
A
Provisional Diagnosis:
Hyperthyroid goiter, Graves’ disease or simply hyperthyroidism
B
Thyroid Profile
• Thyroid function tests: Might show > TSH, low & T3, T4 will be high.
• Antithyroid antibodies (thyroid peroxidase antibodies TPO) to confirm Graves disease.
• .Blood sugar levels
• Thyroid scan
C
Management
• Detailed history:I 'll further ask about history of palpatations,menstrual history,history of
heart disease.
• Examination:GPE,Vital signs,eye examination for proptosis,exophtholmas,lid lag,
• cardiac examination for murmurs,heart sounds.arrhythmias.
• Investigations:as given above in part B
• Non pharmacological :
o Education about the nature of the disease & side effects of the drugs
• Pharmacological :
o Beta Blocker (propranolol, atenolol) for symptom control.
o Carbimazole
▪ Inhibits syntehesis of thyroid hormones
▪ Ineffective in thyroiditis
▪ May be given:
• Short term: Prior to surgery or radioactive iodine therapy to render
patient euthyroid.
• Long Term: 12-18 months to induce remission
• Adverse Effects: Agranulocytosis, hepatitis, aplastic anaemia, lupus-
like syndrome.
• Warn: If develop sore throat or other infection to stop carbimazole
and seek urgent medical attention.
o Radioactive iodine I131 :
▪ Takes 3-4 months to be effective.
▪ Chance of becoming hyperthyroid years after therapy.
▪ Monitor TFTs long-term.
SIGNIFICANT TOPICS WITH SAQs 135
▪
Risk of thyroid malignancy
o Surgery:
▪ Partial or total thyroidectomy if large goitre or patient declined I131.
▪ Risk of damage to parathyroid or recurrent laryngeal nerve.
o Thyrotoxic crisis/storm: Admit/refer if coma, confusion, fever wieth diarrhoea, acute
abdomen, tachycardia/ atrial fibrillation.
SIGNIFICANT TOPICS WITH SAQs 136
80.
Mr. Waheed, a 50 years old businessman attends your clinic for the first time with history of increased
appetite and thirst. His random blood sugar on your practice glucometer is 235 mg and urine has
glucose 2+.
ANSWER
A
Further History Points:
➢ time span for such sx
➢ wt loss or gain
➢ polyurea / hx of vulvar itching/other infections
➢ sexual appetite/ irritability/ depression/ lethargy
➢ hand/ feet tingling or pain
➢ drug history( eg steroid) / pancreatitis to exclude secondary diabetes
➢ blurred vision( cataract/ retinopathy)
➢ diet: type/ meals per day
➢ lifestyle: sedentary or active/ exercise
➢ smoking/ alcoholism/ other addiction
➢ other medical illness: htn,CHD, dyslipidemia, previous impaired GTT
➢ family hx: DM, CHD etc
B
Management Plan:
Detailed history as above.
GPE: Pulse, BP, chest, CVS examination, check eyes, check reflexes in lower limbs. Look for any
diabetic foot or ulcer.
Investigations: HbA1c, FBS, lipid, urea and electrolytes, chest x-ray, ECG, kidney function test.
Non pharmacologic:
• Reassurance, education (about diabetic control and what is the impact on life if not controlled,
how it affects life if complications occur, progressive nature etc
• Dietary Advice: Avoid high-CHO diet, condensed sweets, Fibre rich CHO diet with proteins,
fruits n veggies, but less fat. Avoid ready made diabetic diet / canned or packet food, minimize
alcohol consumption
• Exercise: 30 min daily walk or as per pt requirement
• Smoking cessation
• immunisation: pneumococcal, influenza
• Psychological problems: thru counselling, self help, support groups.
• drug compliance
• advise screening of renal, eye, cardiovascular, foot problems regularly.
Pharmacological
• Metformin ( Glucophage 500 mg bd)
o Adjust dosage as per pt requirement to up to 2 gm if needed.
• Add gliclazide for further control
SIGNIFICANT TOPICS WITH SAQs 137
• if compliance is not good with oral hyperglycemia drugs then start inj insulin sc , as in this age
insulin is best choice
Follow-up:
o Check HbA1c after 3 months to see if it is controlled or not. If not, we can increase
metformin to maximum dose or add third drug.
o Regular follow-up with endocrinologist until diabetes is well controlled.
SIGNIFICANT TOPICS WITH SAQs 138
81.
A 72 year old woman presents with back pain and gradually increasing kyphosis over the last few years.
ANSWER
A
Most likely Diagnosis
Osteoporotic vertebral collapse
We shouldn't just focus on management of Osteoporosis in this case But the main aim to
manage the possible compression fracture.
B
Investigations
1- Labs:
Serum Ca and Ph, 25OH VitD, Alk Phos, PTH,
CBC
Complete bone profile also include these
albumin, magnesium, creatine along with Serum calcium, ph, alk ph, PTH, 25(OH) VD
TSH
2- FRAX estimation (without BMD)
3- Imaging:
- Plain Xray spine
- hip and lumbar BMD with DEXA
MRI is not a routine investigation, it is recommended if there is suspicion of secondary
cause.
C
Management
Detailed history and examination
Management of Pain
The management of pain is important by nonpharmacological and pharmacological means.
a) Non pharmacological:
>Reassurance and explanation
>fall risk prevention;
_identify and manage the factors predisposing to fall like gait problems, poor vision_
>Advise exercise for strength and balance training
>home safety assessment
>advise a balanced diet with adequate intake of vitamin D, calcium, protein, vegetables
>Vit D and Calcium supplementation/ sunlight exposure.
(Treat if deficient, 600,000 IU IM stat, maintenance with VitD3 800-1000 IU daily, Calcium
600-1000 mg daily)
c) Pharmacological management
>Bisphosphonates
*Alendronate 70mg (Fosamax) once weekly , (Take on an empty stomach with plenty of
water first thing in the morning, ≥30min before food/other medication, sit upright for
30min after taking the med.)
Calcitonin may be added if indicated.
SIGNIFICANT TOPICS WITH SAQs 140
82.
Mr. Latif Shah, a 60 years old businessman, a known diabetic comes to your clinic. His fasting and 2 hour
post prandial blood sugars are 240 mg/dl and 320 mg/dl respectively. He has stopped taking his oral
hypoglycaemics because he feels well. He has been brought to your clinic by his son.
A. How would you persuade Mr. Shah to restart his oral hypoglycaemic?
B. Outline four major complications of uncontrolled diabetes.
C. Outline your plan of management. Outline 4 major steps.
D. What 4 lifestyle measures you must emphasize?
ANSWER
A
I will counsel the patient regarding diabetes and the potential dangers of not controlling it. I will
educate him that
diabetes is a progressive disease and needs to be continuously treated to control sugar and avoid
complications. I will explain to him how improper control of diabetes will lead to major
complications in the future and how it can affect
his lifestyle and extra cost in matter of medical expenses as an undue burden to him and his family
and how having a
better control of diabetes will lead to a better life and better life expectancy.
B
Major Complications
1. CVS: Increased risk of MI, stroke, peripheral vascular disease
2. Renal: UTIs, nephropathy
3. Eye: Blurred vision, cataract, glaucoma, retinopathy.
4. CNS: Neuropathy, depression, diabetic foot
C
Steps of Management:
• History: I will enquire:
o When and how he was diagnosed diabetic
o Does he has any symptoms right now, as polyuria, polyphagia, polydipsia, , fatigue,
vision problems, current or recurrent infections, other comirbidities or illness.
o What medicines were prescribed esrlier n how was the sugar control when he was
using thise medicines
o Did he experiance any side effects with his medicines?
o His dietary habits, method n timing of checking sugars.
• Non-Pharmacological: Patient education regarding
o healthy eating
o exercise
o smoking cessation.
• Pharmacological:
o First line oral hypoglycemics, biguanides (metformin500 mg 1gm OD for 14 days,
then BD for 14 days, then reassess) or sulfonylureas(gliclazide 80mg bd).
o Add statin (simvastatin 40mg of if any risk factor found)
o Screening for renal, eye complications, risk of arterial disease.
SIGNIFICANT TOPICS WITH SAQs 141
• Follow up:
o Glucose monitoring and HbA1c.
o Screening as above.
o Check BP, lipid profile
o Offer influenza and pneumococcal vaccines.
o *hypoglycemic episode awareness n treatment.
D
Lifestyle Measures
1. Advice and assistance on smoking cessation and reduction of alcohol intake.
2. Stress management strategies
3. Diet: Healthy, fiber rich with minimum fat, increase portion of fresh fruits and vegetables.
4. Exercise: Encourage regular exercise, walking to and from work, sports and other physical
activities.
5. Weight reduction
6. Diabetic foot care
SIGNIFICANT TOPICS WITH SAQs 142
83.
Mr. Sultan is a new diabetic. He is 50 years old with no history of any previous illness.
ANSWER
A
Main objectives of diabetic care are:
1. Alleviation of symptoms through better glycemic control via lifestyle measures,
pharmacological means.
2. Dietary advice regarding smoking/alcohol cessation/reduction, low-fat diet, increased use of
fruits and vegetables.
3. Minimization of complications by screening for renal, eye complications, risk of arterial disease
on an annual basis.
4. Decrease in early mortality by calculating CVD risk and advising measures to control risk factors
5. Education of patient and family carers for better understanding of disease and for support
regarding patient’s care.
B
Main Examination points in his Foot Care include:
-Foot shape, deformity, joint rigidity and shoes.
-Foot skin condition: Fragility, cracking, oedema, callus, ulceration, sweating and presence of hair.
-Foot and ankle pulses.
-Sensitivity to 10 g monofilament or vibration.
84.
A 50 years old banker with no known co-morbids, presents for a general health examination. On
examination, his height is 172 cm and weight is 102 kg with a BMI of 34.5. His BP is 138/89 mmHg. He
has following laboratory findings. Cholesterol 278 mg/dl, TG 260 mg/dl, LDL 193 mg/dl, HDL 26 mg/dl,
HbA1c 6.2%
85.
A 50 year old businessman attends your clinic for the first time with history of increased appetite and
thirst. His random blood sugar on your practice glucometer is 235 mg/dl and urine has glucose of 2+.
ANSWER
A
Further Questions in History:
➢ Onset, duration and severity of the above-mentioned symptoms
➢ Glucose level checked prior to the visit; if high, for how long.
➢ Pain, tingling, burning sensation in extremities worsening at night
➢ Delayed gastric emptying
➢ Depression, anxiety, lifestyle, exercise
➢ Family history of diabetes/first degree relatives
➢ Drug history: Use of beta blockers, steroids and thiazide diuretics
B
Factors to be Evaluated/Investigated:
• According to the US Preventive Servicestask force(UPSTF
o Recommend screening for asymptomatic adult with sustained BP(treated or
untreated)>135/80 mmhg
o Symptomatic should be fully screened
o ADA diagnostic criteria for DM
o Symptoms of diabetes plus a random plasma glucose concentration >200mg/dl
o A fasting plasma glucose of >126mg/dl on 2 separate occasions
o A plasma glucose > 200 mg/dl 2 hrs after a 75 gm glucose load during the OGTT
• Assess for signs of infections or illnesses as they may raise blood sugar levels
• Calculate BMI
• Screen for diabetic nephropathy
• Screen for cardiovascular risk assessment
• Smoking status
• Assess for any vision problems.
• Lipid modification screen by Qrisk 2 tool
• Foot screen
C
Non pharmacological
• Advise smoking/alcohol cessation/reduction
• Explain risk of cerebrovascular complications
• Promote physical activity and weight loss
• Educate regarding avoidance of foods having high glycemic index (GI).
• Immunization advise for influenza and pneumococcal
• Advise regarding foot care hygiene and frequent visists if suspected complication
SIGNIFICANT TOPICS WITH SAQs 146
Pharmacological
• Glycemic Control:
o If HbA1c 48 mmol/mol: Metformin
o If contraindicated: then sulfonylurea, pioglitazone or dpp4 inhibitors
o If HBA1C rises to 58 mmol/mol consider dual therapy
▪ Metformin plus pioglitazone, SU, dpp4 inhibitors or SGLT
▪ If metformin contraindicated then Dpp4 +PIO, DPP4+SU Or PIO+SU
o If still not controlled and HBA1c >58 mmol/mol, consider triple therapy (any one)
▪ MET+SU+DPP
▪ MET+SU+PIO
▪ MET+PIO+SGLT
▪ MET +PIO+SGLT
o If above fails, consider MET+SU+GLP1
o If still uncontrolled, consider insulin program
• Assess for HTN and stroke risk; add ACE inhibitors and anticoagulants for prevention of MI,
stroke.
SIGNIFICANT TOPICS WITH SAQs 147
86.
A 60-year-old patient has severe chest pain while waiting to see you for his hypertension. The pain is
going across his chest and down to the left arm with lots of sweating and restlessness. The pain had
started 20 minutes ago. His BP is 100/70 mmHg.
ANSWER
A
Immediate Treatment:
• Reassure and counsel the patient
• ECG stat
• Brief history and chest auscultation
• BP, HR, SaO2 monitoring
• Administer supplementary oxygen if sat <94%
• Maintain IV access
• Medications: STAT Aspirin 300 mg, Clopidogrel 300 mg, SL GTN, and Morphine 5 -10 mg IV
at 1–2 mg/minute
• Call EMS and arrange transfer to hospital
• If transfer is expected to be delayed for more than 2 hours, discuss and involve ED physician
or cardiologist in initiating fibrinolytic therapy.
SIGNIFICANT TOPICS WITH SAQs 148
87.
A 55 years old male, salesman by profession comes to you following discharge from hospital 3 days ago,
having been in hospital for 5 days for acute chest pain. He gives you a note stating the diagnosis of
acute inferior MI and Streptokinase administered. The note also states that his hospital course has been
uneventful.
A. List 6 relevant questions in the history and stage (point-wise) how each question will help
you understand the seriousness of the problem.
B. List 3 important things in examination that you must perform in this case.
C. List at least 3 points in the advice that you give to the patient regarding care after MI.
ANSWER
A
Questions to ask
1- *Any recent significant sx after discharge*
-assess for any recurrence of anginal sx or shortness of breath
2- *Compliance to Rx*
-to explore reasons for non-adherence as it can lead to increase morbidity and mortality
postMI also increased risk of rehospitalization.
-any side effects of medications
3- *Compliance to Lifestyle habits*
*smoking* *alcohol consumption* and *physical activity*
*Explore Smoking hx*
-persistent smoking post MI is associated with lower life expectancy
4- *post MI Depression /anxiety screening*
-as untreated depression or anxiety can lead to unhealthy behavioral pathyway
>like less compliance, smoking, unhealthy diet, and inactivity
5- *Review of ASCVD risks*
- explore if there is any poor control of individual risk factors
6- Follow up with Cardiac Rehab
B
Examination
• BMI
• BP
• Chest auscultation, look for arrhythmias/
• EKG
C
Advice to patient
1- *Lifestyle advice*
-reinforced smoking cessation, increase physical activities, healthy alternatives in diet
2- *reinforce compliance to medications*
3- *keep follow up apt *
SIGNIFICANT TOPICS WITH SAQs 149
88.
A 60 years old man presents with history of substernal chest pain for the past one hour. It is associated
with sweating, nausea, and shortness of breath. On examination, his BP is 160/100 mmHg, pulse is
110/min, regular and basal crepitations are heard bilaterally.
ANSWER
A
Most Likely Diagnosis:
ACS (STEMI with left ventricular failure)
B
MANAGEMENT
1- Quick History & Physical:
• History:
o Pain, duration of symptoms
o Previous h/o cardiac disease.
o Cardiac risk factors: Smoking, DM, dyslipidaemia, HTN, family history of MI in < 55
years (male) and < 65 years (females).
o Ask about use of sildenafil use in last 24 hours.
o Contraindications to use of thrombolytics.
• Physical examination:
o GCS, vital signs (low BP, tachycardia)
o JVP
o Auscultation of bases of lungs
o Hepatojugular reflex
o Sacral and pedal edema
2- Investigations;
CBC, FBS, TSH, Urea creatinine, EKG
3- Further Treatment/Action:
• ASA non-enteric 160 to 325 mg chew and swallow.
• Clopidogrel 300 mg
• NTG SL unless BP < 90 mmHg
• IV access
• Morphine 5 -10 mg IV at 1–2 mg/minute
• O2 by nasal canula if SaO2 < 90%.
• ECG stat, troponin I or T stat
• Classify: Unstable angina, STEMI, NSTEMI.
• Inj. Lasix 40 mg IV as patient is displaying signs of LVF.
• Refer to emergency department of tertiary care hospital with PCI capability (in case of STEMI
or for fibrinolysis))
• The goal is immediate fibrinolysis within 30 minutes of onset of symptoms or PCI within 90
minutes of onset of symptoms.
• If transfer is expected to be delayed for more than 2 hours, discuss and involve ED physician or
SIGNIFICANT TOPICS WITH SAQs 150
89.
A 50 year old businessman presents for a general check up. He has following laboratory findings: Total
Cholesterol = 278 mg/dl, S. triglycerides =-260 mg/dl. High density lipoprotein (HDL) 36 mg/dL, low
density lipoproteins (LDL) = 193 mg/dL
ANSWER
FIVE STEPS OF MANAGEMENT
1. History:
• H/O diabetes, HTN, hypothyroidism.
• H/O sleep apnea
• Family history of premature CVD (men < 55 years, women < 65 years) Family history of
dyslipidaemia.
• H/O lifestyle preferences.
o H/O smoking/alcohol
o H/O sedentary lifestyle
o H/O food intake.
o Level of physical activity.
• Medication history
• Past medical history
• Socioeconomic history
2. Physical examination
• GPE/Vital Signs:
o Pulse, BP, BMI
o JVP
o Pallor
o Oedema
o Xanthelasma
• Chest: B/L basal crepitations.
• Heart: Heart sounds, heart rate, murmurs, any added sounds.
• Peripheral Vascular Exam
• Neurological Examination
3. Investigations;
• CBC
• FBS
• TSH
• S. creatinine and eGFR
• ECG
4. Nonpharmacological* management;
• Patient education: About his condition, counselling and motivation of patient for
healthy lifestyle modification. Education and information about risks and complications
of dyslipidaema.
• Lifestyle Modification:
o Diet:
SIGNIFICANT TOPICS WITH SAQs 152
▪
Reduce intake of fat to < 30% of total energy intake (saturated fat < 10%)
▪
Reduce cholesterol intake to < 300 g/dL.
▪
Replace saturated fats with monounsaturated and polyunsaturated fats.
▪
If cholesterol level > 5 mmol/L, lowering cholesterol in diet results in
reduction in cholesterol levels of 0.5% in 3 months.
▪ Eat > 5 portions of fruits and vegetables and > 2 portions of fish per week
including one of oily fish.
▪ Stop smoking (if smoker) and stop alcohol.
▪ Weight reduction – lose weight if BMI > 30 kg/m2.
▪ Increase physical activity to 30 minutes moderate exercise e.g. brisk walk >
5 times per week. Encourage aerobic physical activities for optimal weight
+ aim for loss of total body fat + visceral fat.
5. Pharmacological management
• As LDL > 190 mg/dL, Tab. atorvastatin 20 mg OD, education about possible side effects.
• Follow-up: Three months, earlier if any problem.
• Check LFTs at 3 months and 1 year after initiating statin.
SIGNIFICANT TOPICS WITH SAQs 153
90.
A 60 years old domestic servant complains of shortness of breath on walking for past 6 weeks. He has
mild cough, mostly dry. On examination, his BMI is 27 kg/m2. BP is 140/80 mmHg and pulse is 90/min,
regular. Third heart sound is audible along with bilateral basal crepitations.
ANSWER
A
Most Likely Diagnosis:
LVF
B
Investigations:
• CXR-PA view
• Blood tests: CBC and BNP
• Glucose tolerance test
• Echocardiogram
• RFTs (urea, creatinine)
• LFTs
• Fasting lipids
• Ankle brachial index
• Cardiovascular risk assessment
C
Management Plan:
• Detailed History:
o DM, HTN
o Smoking/alcohol history.
o Family history of sudden cardiac death below age of 40 years, dyslipidaemia,
hypertension, diabetes mellitus.
• Examination:
o GPE/Vital Signs: For tachycardia, obesity, hypertension
o CVS: Raised JVP, laterally displaced apex beat, heart murmurs.
o Respiratory: For tachypnoea, basal crepitations, pleural effusion.
o Abdominal Exam: For any liver enlargement
o Extremities: For peripheral dependent edema
• Investigations: As above
• Treatment Plan:
o Review the patient’s medications and stop/reduce any that may worsen heart
failure.
o Start loop diuretics, e.g., furosemide 20-40 mg daily.
o If symptoms not relieved and/or prior history of MI, refer for specialist advice.
SIGNIFICANT TOPICS WITH SAQs 154
91.
A 60 year old male presents with history of substernal chest pain for the past one hour. It is associated
with sweating, nausea and shortness of breath.
On examination, his BP is 160/100 mmHg, pulse 110/min, regular and basal crepitations are heard
bilaterally.
ANSWER
A
Most Likely Diagnosis:
Acute MI with LVF
B
Investigations:
• ECG
• Finger stick BSR
• TROP I
• Others:
o Send samples for myoglobin and CK-MB
o CHEST XRAY
o CBC
o BUN
o BNP
C
Immediate Management:
Reassure and counsel the patient
ECG stat
Brief history and chest auscultation
BP, HR, SaO2 monitoring
administer supplementary oxygen if sat <94%
maintain IV access
Medications:
STAT Aspirin 300 mg PO,
Clopidogrel 300 mg PO
, SL GTN 0.4MG ,
TRAMADOL 100MG IV and
inj metaclopromide 10mg Iv
Call EMS and arrange transfer to hospital
If transfer is expected to be delayed for more than 2 hours, discuss and involve ED physician
or cardiologist in initiating fibrinolytic therapy.
SIGNIFICANT TOPICS WITH SAQs 155
92.
50 year old Mr. Ali Khan lecturer by profession presents to your clinic for a physical check up. On
examination, his BP is found to be 170/105 mmHg. BMI is 30. In this case:
A. Enlist four important relevant examination steps you will perform in this patient.
B. Enlist four relevant investigations you will order.
C. Enlist four points of your management plan.
ANSWER
A
Relevant Examination Steps
1. GPE/vital signs: Sitting and standing BP,Check pulse , jvp, tremors , eyes for pallor and
puffiness , any yellowish fat deposit on eyes.
2. CVS examination .. look for JVP , carotid bruit ,heart sounds , any murmurs ,pheripheral
pulses n compare
3. Chest: look and auscultate for resp breathing , any basal crypitations , any chest deformity.
4. Abdomen .. any abnormal pulsation in abdomen , abdominal reflex , any orgabomegaly
5. Legs: For oedema
B
Relevant Investigations:
• Chest X-ray
• ECG
• Echo
• Lipids, HbA1c, FBS, urea & electrolytes, TFTs, urine R/E
C
Management:
Detailed History: Including;
• H/O DM
• Lifestyle (smoking/alcohol use, level of activity)
• Cardiovascular risk assessment
• Family history of dyslipidaemias, early cardiac-related death.
Physical Examination:
• Please see A
Investigations:
• Please see B
Treatment:
Non pharmacological
Stop smoking , avoid alcohol
Low salt intake, low fats, avoid oily processed food, increase fluid intake, regular excercise,
balanced diet , BP monitoring either at home using ABPM , twice per day for 1 week n come back
to clinic for review/0
Pharmacological:
This patient is having stage 2 HTN ( more than 160/100)needs treatment, start with lisinopril 5 mg
once per day n come back in 1 month
If any time feels severe headache , blurring of vision ANT altered conciousness
SIGNIFICANT TOPICS WITH SAQs 156
Contact A&E
SIGNIFICANT TOPICS WITH SAQs 157
93.
A young male patient presents with case of sudden onset of severe right scrotal pain. On examination,
right scrotum appears normal in color and extremely tender on touch. Mild relief of pain is observed on
scrotal elevation.
ANSWER
A
Most Likely Diagnosis
Testicular torsion
B
Most Important Test
Scrotal Doppler ultraound
C
Management
History: Enquire about
➢ Pain onset (sudden in onset in testicular torsion)
➢ Duration
➢ Severity (usually severe)
➢ Swelling (usually sudden)
➢ Previous episodes of severe, self limiting pain and swelling.
➢ Associated symptoms including;
➢ Nausea or vomiting (common)
➢ Symptoms of lower urinary tract infection or urethral discharge
➢ Parotid swellings suggest mumps related orchitis
➢ Back pain, breathlessness and weight loss (to rule out carcinoma with mets)
➢ Sexual History
➢ History of trauma/strenuous physical activity
➢ Past history of severe pain on and off.
➢ Previous history of undescended testes.
Examination:
• Position of swelling in relation to the testes (testicular, extratesticular)
• Testicular lie (suspect testicular torsion if high-riding and transverse)
• Size and symmetry of testes
• Tenderness (present in torsion)
• Consistency if swelling is present
• Lymphadenopathy and abdominal mass
• Cremasteric reflex (if acute pain or swelling) suspect testicular torsion if cremasteric reflex
is absent.
• Prehn sign (relief of pain with elevation of testes) does not rule out testicular torsion
• Transillumination (hydrocele)
SIGNIFICANT TOPICS WITH SAQs 158
• Features of inguinal hernia (examine the person, both lying and standing)
• A positive cough impulse
• Palpable bulge in inguinal canal
• Skin changes blue dot sign (inflamed and ischaemic torsed appendage can be seen through
the scrotal skin
Investigation
As in B
Treatment Plan
• Nonpharmacological
Quick review and then expalantion of the urgency for the disease
Call for the emergency transfer to the hospital
• Pharmacological/Referral
If on assessment testicular torsion suspected, immediate surgical referral as irreversible
ischaemic injury can begin within 4 hours of cord occlusion resulting in reduced fertility
and testicular loss.
D
Differential
❖ Epidydimoorchitis (onset of pain gradual)
❖ Strangulated inguinal hernia(cough reflex+inguinal swelling)
SIGNIFICANT TOPICS WITH SAQs 159
94.
A 60 year old man presents with 2 weeks history of increased urinary frequency, urgency and poor
urinary stream
ANSWER
A
Most likely
Benign prostatic hypertrophy
B
Investigations
1- Urinalysis / dipstick
• should be done in all patients with lower urinary tract sx to rule out other pathologies
2- PSA
• It’s levels correlate well with prostate volume which can help guide treatment choice
3- International prostate symptom score IPSS
• It scores symptoms of prostatism (minimal, moderate, severe).
• Through a self-administered patient questionnaire with 8 questions (7 questions on symptoms
and 1 question on quality of life - bother score)
C
Management
1- Establish diagnosis by careful history, directed physical examination and investigations as above
2- Nonpharmacological management
• Patient Education
- Give reassurance, explanation and health edu to the patient regarding the probable diagnosis
and discuss its management
• Watchful waiting;
- if sx mild to moderate without evidence of complications
- if sx are not severely troublesome
• Advise self help/ behavioural modification techniques
- decrease fluid and caffeine intake
- bladder retraining
- avoid constipation
3- Pharmacological management
• for those with troublesome mild to moderate sx
• ALPHA BLOCKERS
- PRAZOSIN 1mg 8hrly may increase dose to 20mg/day/ DOXAZOSIN ( monitor for postural
hypotension)
• 5 ALPHA REDUCTASE inhibitors
- FINASTERIDE ( for pts with bulky prostates. It decreases risk of urinary retention )
• Combination Rx for improved outcome
4- Referral to urologist;
SIGNIFICANT TOPICS WITH SAQs 160
• urgently
- if evidence of complication, like acute retention
- on DRE nodular or firm prostate
- increase level of age specific PSA (this case 4.0 or more)
• soon, in case of severe sx
5- Follow up in 2 weeks
SIGNIFICANT TOPICS WITH SAQs 161
A 6 year old child was brought by her mother with the complaint of bed wetting mostly at night.
ANSWER
A
Further History Points:
➢ Ask bed wetting routine.
▪ How many times per night.
▪ How many times per week (because if > 2 nights/week, then it is nocturnal enuresis)
➢ Ask if same problem in other siblings or family because it tends to run in families (strongly
suggestive)
➢ Any other physical abnormality: UTI, constipation or DM
➢ Any mental stress on the child, although occasionally caused by emotional distress.
➢ Ask if the child has been dry for 6 months prior to onset.
➢ Ask maltreatment like parents blame or punish the child despte advice that it is not child’s
fault.
➢ Ask sleep walking as sleep walking can lead to bed wetting.
➢ Ask any psychological issues; death in the family, sexual abuse, extreme bullying are
established as a cause of secondary enuresis.
➢ Ask neurological-developmental issues, as patients with these issues, handicapped and
intellectually disabled children have a bed wetting rate almost three times higher than non-
handicapped.
➢ Children with ADHD are 2.7 times more likely to have bedwetting issues.
➢ How do you respond to your child’s bed wetting.
B
Investigations
• Urinalysis: (Urine DR and culture and sensitivity)
• It is the most important screening test in a child with enuresis.
▪ Children with cystitis have increased WBC and bacteria.
• Children with overactive bladder, dysfunctional voiding, urethral obstruction,
neurogenic bladder, ectopic ureters or diabetes are predisposed to cystitis.
• If urinanalysis findings suggest cystitis, a clean catch urine specimen should be sent
for culture and sensitivity.
• RBCs in urine (in urethral obstruction)
• Glucose in urine (diabetes mellitus)
• Ultrasonography
• Failure to empty the bladder is a significant risk factor for cystitis.
• Portable bladder ultrasonography is available to assess residual urine.
• Residual volume of urine is normally less than 5 ml.
• Voiding cystourethrography and plain radiograph
• If the bladder wall is thickened or trabeculated on ultrasound or significant residual
volume > 50 ml of urine is noted, voiding cystourethrography is considered.
SIGNIFICANT TOPICS WITH SAQs 162
96.
A 16 years old male comes to your clinic with severe pain in left side of abdomen radiating from back to
front. He was playing cricket in a hot summer day and developed the pain just before the end of match
while fielding at the boundary.
ANSWER
A
Most Likely Diagnosis:
Renal calculi
B
Immediate Management
• Quick assessment by history and exam Reassurance
• IV line
• Analgesia: Diclofenic sodium 75 mgIM
• Antispasmodic: Inj. Buscopan IV
• Antibiotics: If any signs of infection.
Admission: If;
o fever
o pain not settled
o Poor oral intake
o Oliguria
o Uncertain diagnosis
Referral:
To urologist if investigations show decreased renal function. Renal obstruction. Or remaining
stones
97.
A 30 year old man presents to your clinic for advice regarding his recurrent renal stones. He has had 3
episodes of passing stones in urine in the last 6 years. His stone analysis revealed calcium oxalate stone.
ANSWER
A
Detailed History:
➢ I will ask about the reason of this visit
➢ Is there any current complaint like flank pain or dysuria?.
➢ I will ask about family history of renal stones especially in maternal uncles or brother,
cystinuriaor hyperoxaluria in family, any congenital kidney problem ( horse shoe shaped
kidney ormedullary sponge kidney)
➢ Any drug use (steroids , aspirin , loop n thiazides diuretics, allopurinol, calcium n vitamin D
supliment, vit C )
➢ Ask about if suffering from any disease (gout , hypercalemia, parathyroid disorder,
hyperuricemia, hypercalciuria,)
➢ Ask abt water intake, rule out dehydration
➢ Ask about recent surgery
➢ Ask about physical activity ( to rule out immobilization)
➢ Ask about recurrent UTI GPE general n systemic examination
Investigations: First I will review all past reports record available.
• Check current renal status; urea and electrolytes, creatinine
• Serum calcium, phosphate
• eGFR
• urine dipstick n MSU
• X-ray KUB
• Renal tract ulttasound
• Alkaline phosphatase
• Serum albumin
• Uric acid
Pharmacological:
Effective analgesia if pt is having pain now.
Since patient is having calcium oxalate stones advice the patient for urinary alkalization
with pitassium citrate
If patient is suffering from hypercalciuria add thiazides diuretic bendroflomethiazide 2.5 mg
OD Or pyridoxine if hyoeroxaluria.
Red Flags: Advise the patient to go to A&E as soon as possible if;
• symptoms remain persists or
• suffering from fever,
• decreased urine output or
• unable to take his meals.
Advice to the patient:
Increase fluid intake, especially in hot weather.
SIGNIFICANT TOPICS WITH SAQs 166
98.
You are Mr. Jamal’s family physician. He is 26 years old and has had 3 episodes of ureteric colic in the
last 2 years. He had had extensive workup for this, all of which were normal. His calculi are of Ca
oxalate variety. He wants your advice on preventive measures for this problem.
ANSWER
A
➢ Increase fluid intake, especially in hot weather. Recommended intake is 2.5 to 3 L per day.
➢ Lose weight if obese (through a mechanism other than a diet high in animal protein)
➢ Dietary Measures:
o Decrease animal protein
o Increase fruits and vegetables
o Lower salt intake. A dietary intake of < 2 gram per day is recommended.
o Avoid oxalate-rich foods like chocolate, tea, rhubarb, dark green leafy vegetables
(spinach, chard, arugula), nuts, beans, beetroot, strawberries, wheat bran, nuts,
seeds, soy products and citrus fruits
o Increased intake of calcium (as calcium inhibits intestinal oxalate absorption). A diet
having 1200 mg calcium daily with reduced oxalate and salt intake is found to be
highly beneficial in preventing recurrence in random controlled trials (RCT).
➢ Pharmacological:
o Improved diabetes control may help to prevent stone formation in appropriate
patients.
o Potassium citrate for urinary alkalinization, as low urine pH is found to be associated
with higher recurrence rates.
o Bendroflumethiazide 2.5 mg if hypercalciuria
o Allopurinol 100 mg 3 times daily is found beneficial in reducing stone formation.
o Fish oil may be tried as low levels seen in eskimos, but it has yet to be trialled.
o Pyridoxine if hyperoxaluria.
SIGNIFICANT TOPICS WITH SAQs 168
99.
A 42 year old college lecturer comes very concerned with her blood report. Showing a creatinine level of
1.6 mg/dl. Her creatinine level 4 months ago was 1.3 mg/dl. Her father died of renal failure at the early
age of 50. She has no ongoing medical problems. Her blood pressure is 120/75 mmHg.
ANSWER
A
Most Likely Diagnosis:
Chronic Renal Failure (CKD)
B
Common Causes:
• Adult polycystic kidney disease
• Chronic glomerulonephritis
• Chronic pyelonephritis
• Hypertension
• Diabetic nephropathy
•
C
Relevant Questions in History:
➢ Ask about family history ( to rule out polycystic kidneys)
➢ Chronic disease (especially diabetes or HTN) or other autoimmune disease (HIV, SLE, RA)
➢ Drug history (especially NSAIDs)
➢ H/O anaemia (normocytic normochromic anaemia), which becomes evident when GFR < 35
ml/min. If GFR < 60 ml/min, then other causes can be considered.
➢ H/O pain in joints of legs or arms (to rule out low vitamin D and low calcium.
D
Investigations:
• Albumin:creatinine ratio (to check the stage of disease)
• Glomerular filtration rate (GFR) (to check the stage of disease)
• Ultrasound of kidneys (to rule out the cause)
• Biopsy (to rule out the cause)
•
E
Steps of Management:
1. Detailed patient history (for proper diagnosis)
2. Investigation and examination (to rule out the cause)
3. Non-pharmacological Treatment:
a. Lifestyle modification
SIGNIFICANT TOPICS WITH SAQs 169
b. Smoking cessation
c. Increased fluid intake
4. Pharmacological Treatment
a. Manage underlying cause (monitor or treat anaemia or underlying renal bone
disease or DM)
b. Dialysis
c. Transplant
d. Stop or avoid nephrotoxic drugs (NSAIDs, ACE-I, aminoglycosides, etc.)
SIGNIFICANT TOPICS WITH SAQs 170
100.
A lean young man has a dull pain from left loin to the groin extending to the testicle. He also complains
of running a low grade fever for the last few weeks, loss of appetite and weight.
ANSWER
A
1.Tuberculous pyelonephritis or Renal TB
2.Renal carcinoma
3.Renal Abcess
B
Renal TB:
History:
• Points in favour:
a).h/o fever
b).h/o weight loss
c).h/o loss of appetite
d).pain in loin and groin
• Points against:
a).high grade fever
b).absence of dysuria
c)no h/o of pulmonary tuberculosis or sexual contact with tb patient.
Examination:
• Points in favour:
1.temperature raised
2.testicular swelling with tenderness in most cases
3.malnourished and low body weight.thin and lean.
• Points against:
1.afebrile patient
2.palpable mass in flank( May b tumour)
3.absence of testicular or scrotal swelling
Investigations:
• In Favour:
1.CBC,raised WBC count,HB May b low,ESR raised, CRP raised
2.Urine examination:sterile pyuria
3.Urine Culture for AFB is positive mostly
4.tuberculin test: positive
5.PCR:is highly effective
6.KUB,IVP
• Points Against:
1.Absence of sterile pyuria
2.Negative urine culture for AFB
SIGNIFICANT TOPICS WITH SAQs 171
Examination:
• Points in favour:
1:temperature raised
2.BP:is low in abcess
3.tachycardia,tachypnea
4.Palpable mass with tenderness in flank
• Points Against:
1.Low or normal temperature
2.No palpable mass in flank
3.absence of varicocele
Investigations:
• Points in favour:
1.CBC with differentials:raised leucocytes count
2.BUN,S/creatinine are raised
3.Urinalysis:pyuria and proteinuria
4.Urine culture and blood culture are positive mostly
5.USG
6.CT scan is more specific
• Points Against:
1.absence of pyuria and proteinuria
2.Negative urine and blood culture
3.Negative findings on USG and CT scan
SIGNIFICANT TOPICS WITH SAQs 173
101.
A forty-six year old businessman presents with the complaint of severe pain in the right toe. On
examination the big toe is red, swollen and tender.
ANSWER
A
Further Questions In History
1- Onset and progression of symptoms.
2- H/O trauma/ injury / previous joint surgery or spontaneous ?
3- Any h/o fever?
4- Any recent or current GI infection/ conjunctivitis/ rash/ urethritis
5- Any hx of psoriatic rash
6- Any previous similar hx?
7- Any earlier treatment?
8- Any other joint affected?
9- Past medical hx esp. of metabolic syndrome (obesity, hyperlipidemia, hypertension), DM, renal
disease and osteoarthritis ?
10- Medication hx ?
11- Personal hx, esp. diet and hydration?
• increased consumption of red meat, sea food, some vegetables like spinach and mushrooms,
sugar sweetened soft drinks (clear association with gout)
12- Any hx of alcohol (esp. beer) intake/ addiction ? (association with gout)
13- Any hx of IV drug use?
14- Any significant family hx ? (familial preponderance of gout)
B
Differentials
1- Acute podagra due to gout or pseudogout (crystal induced)
2- Septic arthritis
3- Trauma-induced
C
Management:
1- Confirmation of diagnosis by;
• careful history (discussed above) and
• physical examination: GPE, Vital Signs; local exam of affected toe looking for crepitus,
effusions, limited ROM, check contralateral joint, evaluate other joints, look for tophi or
nodules.
• Investigations:
- CBC, ESR, (leucocytosis, raised ESR)
- FBS, Lipid profile, (Screen for chronic disease and cv risk assessment)
- Renal profile (kidney function)
- Serum uric acid (after at least 2 weeks of resolution of sx due to high false positive rate)
SIGNIFICANT TOPICS WITH SAQs 174
102.
A 26 year old house wife presents with pain in the right forearm especially elbow area since 1 month.
The pain is worse when she washes clothes or stirs the meals in the kitchen or when carrying heavy
weights. Examination reveals tenderness over lateral epicondyle.
ANSWER
A
Lateral epicondylitis, tendinopathy from overuse/overload* _Tennis elbow_
B
Management
Reassurance
Detailed hx
• Pain (localized or diffused)
• Swelling/lump/deformities
• Limitation or loss of function
• numbness/paresthesias
Focused physical examination
Nonpharmacological (for at least 6 weeks)
Arm rest
Icing 20 minutes QID and after exercise
Moist heat or ultrasound
avoidance of aggravating/trigger movement
physiotherapy/exercise – Epicondylitis
o Painless passive wrist flexion
o Progressive resisted wrist extension
Use elastic band tied between foot and hand
consider forearm strap
Tennis Elbow counterforce strap
o Dampens force transmitted to elbow from wrist, hand.
o Activity restriction
o Avoid grasping in pronation
o Lift only with wrist in supination.
Pharmacological
Topical or oral NSAIDS –Diclofenac
Lidocaine (Xylocaine) 1%: 1 ml
Local Steroid Inj if severe pain
o methylprednisolone (Depomedrol) (40 mg/ml): 1 ml
o Triamcinolone acetate (kenacort)
Follow up every 6-8 weeks for reassessment.
Referral: To orthopedics if patient not responding in 6 months for surgical debridement and
tendon repair.
SIGNIFICANT TOPICS WITH SAQs 176
103.
A 10 years old boy is brought to your clinic by his mother, with 3 months’ history of joint pains and
swelling.
ANSWER
A
Important hx points
-onset, course, duration, frequency of symptoms.
-Specific location of pain (connective tissue vs. referred pain).
-Joint distribution and the number of joints involved.
-Prolonged morning stiffness, swelling.
-Constitutional symptoms: fever, weight loss and fatigue.
-Alleviating and aggravating factors.
-Extraarticular symptoms: Red eyes, rash
-Family history of psoriasis, connective tissue diseases, rheumatoid arthritis
-Limitation of daily activities
-H/O trauma
B
Important areas of Examination
• GPE: General appearance, weight, height
• Eye: (red eye)
• Reticuloendothelial system: cervical, axillary and inguinal lymph nodes.
• Abdomen: Hepatosplenomegaly
• Skin: Rash
• Musculoskeletal Examination:
o Warmth, swelling, loss of ROM, guarding)
o Muscle mass
o Strength
o Proximal muscle weakness
o Local tenderness
C
Differentials: *DDs of Chronic* first
-Juvenile Rheumatoid/Idiopathic Arthritis
-Infective/Septic
-Trauma
-Malignancy
-SLE
-Reactive arthritis
-HSP
D
SIGNIFICANT TOPICS WITH SAQs 177
Investigations
-CBC
Rule out infection, SLE, malignancies
-ESR
Inflammation
Infection
-ANA
SLE
Vasculitis
JRA
Plain X-ray
Fracture and bony loss.
RA Factor: The higher the level, the higher the possibility of having any one of the following.
Rheumatoid arthritis
Sjogren’s syndrome
SIGNIFICANT TOPICS WITH SAQs 178
104.
35 years old Mrs. Khadija presents with history of discomfort in fingers and toes during cold weather
and when bathing. She has also noticed her finger turn blue. She has also reports that the joints of her
hand and feet have become painful. This has been happening since 6 months.
ANSWER
A
Differential Diagnosis:
Rheumatology Conditions:
• Scleroderma
• SLE
• Rheumatoid arthritis
Haematology
• Leukemia
• Polycythemia
• Thrombocytosis
Medications:
• Beta blockers
Smoking
• Can cause arteriosclerosis
Trauma: For example, use of vibrating tools
B
Investigations
Baseline
• CBC – to evaluate polycythemic disorder or underlying malignancy
• ESR and CRP-
• BUN + S. creatinine
• Blood sugar
• Urinalysis
Specific Investigations:
• Rheumatoid factor
• Anti-CCP
• ANA antibodies
• Anti-ds DNA
• Anti-Scl 70
SIGNIFICANT TOPICS WITH SAQs 179
105.
A 30 years old woman presents with 2 months history of pain and swelling of small joints of both her
hands. She reports morning stiffness of her joints lasting one hour.
ANSWER
A
Provisional Diagnosis:
Rheumatoid arthritis
B
Investigations:
• RA factor: Positive in 80% of rheumatoid arthritis patients.
• ANA: Positive in 30%.
• Anti-CCP level: It is positive in most of patients with suspected RA (may be detectable 10
years preceding development of RA)
• X-ray of hand joints, which are involved: May show perioarticular osteoporosis or soft
tissue swelling. Radiography will show symmetric involvement, articular osteopenia, loss of
articular cartilage.
• CBC: May show mild normochromic normocytic anaemia
• CRP and ESR
C
Treatment Options: Patient with multiple joint involvement is indicative of referral to
rheumatologist, but I will refer the patient after baseline investigations and offering initial
treatment. Treatment options are divided in two.
Non-Pharmacological Treatment:
I will counsel the patient that most probably she is suffering from RA, an inflammatory
condition, which primarily involves joints.
Will address patient’s ideas, concerns and expectations in detail.
Will discuss in detail about disease, treatment and support available
Will advise patient exercise, splint and strapping to keep joints mobile, painfree and to
preserve function.
Vegeterian and Mediterranian diet might be helpful
Will screen for depression (as chronic pain may lead patient to depression)
She might need to get involved with physiotherapy, occupational therapy and psychotherapy.
Ice or heat pad might be helpful
Will assess weight, BMI, fatigue and sleep quality
I will advise medication adherence and medication compliance.
Pharmacological Treatment:
Initial Treatment:
o Tablet Ibuprofen 400 mg TDS or
SIGNIFICANT TOPICS WITH SAQs 180
106.
A 26 years old male comes to you with history of right ankle joint pain for the last 1 hour. He was
playing football when he suddenly twisted his ankle joint. He was able to bear weight and walk
immediately. On examination, patient is able to bear weight and walks 4 steps in the clinic. Ankle joint
is swollen, warm and tender on lateral aspect. There is no tenderness over malleoli.
ANSWER
Management Plan:
History: In addition to information given in scenario, ask for;
• At time of injury, did you feel as if struck violently over back of ankle?
• Experienced a “snap” or “pop” over injured ankle?
• Sudden, sharp, severe pain radiating to ipsilateral calf?
• Cause of injury most likely was “foot landing (ankle sprain) or pushing off (Achilles tendon
rupture)”
• Occupational history – athletic activities. Increased speed work, hill training (to rule out
chronic abuse – Achilles tendinopathy)
• Use of ill fitting, worm and hard-backed foot wear (to rule out calcaneal bursitis)
• Drug history – glucorticoids, fluoroquinolones
• Comorbidities – DM, HTN, psoriasis, ankylosing spondylosis.
Relevant Examination: Compare with uninjured side. Look and check for.
• Ambulation
• Foot architecture – pes planus, pes cavus, heel malalignment
• Defortmity, oedema, discoloration, bruises
• Warmth, tenderness (over Achilles insertion point or 2-6 cm above), sensations
• Palpate Achilles tendon (injured) for thickness, defect in continuity, crepitus
• Palpate tendon while asking the patient to dorsiflex and plantar flex the foot (injured side)
• Movements:
o Assess active and passive movements of ankles, subtalar, midtarsal and toe joints
symmetrically.
o Check ROM of ankle joints.
o Ask patient to lie prone with feet hanging off the table over injured side, elicit
o Thomson Test – Positive in Achilles tendon rupture (i.e. absence of plantarflexion on
squeezing ipsilateral gastrocnemius belly)
Investigations:
• Achilles tendon rupture usually diagnosed clinically
• MRI – best test to confirm.
Treatment Plan:
• Partial rutpures can be given conservative trial (for 3 months) – if failed, sugical
intervention.
• All complete Achilles tendon ruptures merit immediate referral for surgical consultation in
case of unavailability of orthopedic consultation.
• Non-Pharmacological:
SIGNIFICANT TOPICS WITH SAQs 182
o
Avoid aggravating activities
o
Apply ice when symptomatic
o
Support Achilles with a heel lift, arch support, orthotics and running shoes.
o
Regular physiotherapy by consulting a physiotherapist – eccentric exercise with
heavy loads may benefit.
o Immobility using plantar flexion, short leg cast or functional brace with heel lift (6-8
weeks) may also be performed but in specialized care units.
o Can return to sports after 3-6 months.
• Pharmacological:
o Analgesia
▪ Ibuprofen (400 mg) TDS
▪ Topical, e.g. diclofenac gel as needed.
Follow-up: 2 weeks
SIGNIFICANT TOPICS WITH SAQs 183
107.
Mrs. Farhan brings her 18 months old child to you with the complaint that he is not walking yet. Her
previous child has started walking at the age of one year. The child is bottle fed on cow’s milk and has
been weaned at 6 months of age. His weight is 8 kg. His wrists are broadened and he has a wide
fontanelle.
ANSWER
A
Most Likely Reason:
Rickets
Logical Justification: Child has wrist widening, frontal bossing, delayed walking, low weight for
age, using cow’s milk, all pointing towards nutritional rickets.
B
Investigations:
✓ X-ray wrists and other bones
✓ Serum electrolytes (Calcium, phosphate, alkaline phosphatase) and creatinine – to rule out
renal rickets
✓ Serum proteins, albumin
✓ 24-hour urine for Ca and PO4.
C
Management:
After taking complete history and examination and sending labs and reaching a confirmed
diagnosis, then start management according to diagnosis.
Non-Pharmacological:
o Sunlight exposure
o Use fortified milk butter, cheese, etc.
Pharmacological:
o If child has tetany – treat with Ca gluconate 2 ml/kg IV slowly. Repeat after 6 hours
interval.
o Give Vitamin D – 2000 to 3000 IU daily for six weeks, followed by maintenace,
followed by maintenance 600 to 800 IU per day. Healing generally occurs in 2-4
weeks.
o Alternatively, 200,000 units of vitamine D (IM) single injection can be given.
o Review after 2 weeks and look for x-ray evidence of healing.
o If healing has not initiated, then give another injection of vitamin D3. If after two
injections healing does not occur, then vitamin D resistant rickets should be
labelled.
o If healing has initiated, continue 400 IU daily as maintenance dose.
SIGNIFICANT TOPICS WITH SAQs 184
108.
An 8 year old boy is brought by his mother complaining that the child suddenly started to have walking
difficulty for the last two days. He complains of pain in the anterior thigh region.
ANSWER
A
Important Differentials:
1. Perthes’ disease
2. Growing pain
3. Trauma
4. Muscular dystrophy
5. Arthritis
B
Questions to Ask From Mother:
1. Onset and progression of the pain, if the pain aggravated since start or has stayed the
same.
2. Any particular time of pain or is it constant.
3. Association of pain with activity or rest.
4. Any preceding trauma
5. Any signs of infection, i.e., fever.
6. Aggravating and relieving factors
7. Any previous attacks of similar nature
8. Any medication tried and its affect.
9. Any first-degree relatives with muscular dystrophy
C
Important Points on Examination:
1. General Physical Exam: General appearance, anaemia, gait examination, etc.
2. Vital Signs: Temp, BP, pulse, R/R, height and weight of the child
3. Thigh Exam: Size, symmetry, any tenderness, warmth, swelling, tone, power, muscle bulk,
range of movements.
4. Hip & Knee Exam: Check any tenderness, warmth, swelling, range of movements (will be
decreased in Perthes’ disease). Internal rotation and abduction movement at the hip joint.
SIGNIFICANT TOPICS WITH SAQs 185
109.
A 56 years old lady presents to you with the complaint of pain and stiffness of the right shoulder.
ANSWER
A
Further Questions on History:
➢ history of trauma
➢ history of diabetes
➢ fever
➢ pain and stiffness on top of shoulder(acromioclavicular)
➢ any deformity swelling.winging of scapula
➢ history of fall on the shoulder
➢ history of recent bacterial infection lung,MI
➢ pain worse at what time ( adhesive capsulitis more at night)
➢ does pain radiates from any where or to anywhere
➢ LOSS of function .difficulty reaching behind back
B
Steps of Examination:
• Look... look for posture , symmetry , muscle swelling, muscle wasting
• Feel... palpate for tenderness, warmth , crepitus
• Movements. ..active and passive movements .compare sides.range of movement.power.
External rotation might be markedly affected in adhesive capsulitis
C
Differential Diagnosis:
❖ Adhesive capsulitis (needs USS/MRI to confirm)
❖ OA. More common in old age. Need x Ray
❖ Inflammatory arthritis RA/psoriatic
❖ Rotator cuff
SIGNIFICANT TOPICS WITH SAQs 186
110.
A 35 years old woman presents to your clinic with recurrent history of vaginal discharge since last 2
years.
ANSWER
A
Questions To Be Asked: I will ask about symptoms
➢ discharge is thick or clear, sticky, offensive, itchy, colour, quantity, association with menstrual
cycle,
➢ vaginal soreness n irritation, redness, vulval pain
➢ menstrual : heavy periods, dysmenorrhea, intermenstrual bleeding,
➢ sexual : dyspareunia, post coital bleeding, multiple partners or new partner, symptoms in
partner, h/o STI in pt n partner
➢ Medical: fever, pregnancy, abortion or instrumentation, diabetes, immunocompromise, recent
antibiotics ,
➢ contraception if used, IUCD
➢ attempts at self medication
B
Investigations:
• chk pH of secretion with narrow range pH paper, if >4.5, BV or TV is likely, if pH is <_ 4.5 with
physiological discharge or candida inf.
• chk for pregnancy
• high vaginal swab for M , C & S, if suggestion infective cause, pregnancy, posr partum, after
miscarriage or termination, after instrumentation, recurrent infection or treatment failure in
past
• endocervical swabs for gonorrhea and Chlamydia
• viral swabs for herpes
SIGNIFICANT TOPICS WITH SAQs 187
111.
A married woman 26 years of age with 2 children attends your clinic with a request for a prescription of
a contraceptive pill.
A. What further questions will you ask her before considering her request?
B. What relevant examination will you carry out?
C. What will you say to her regarding the possible side effects of the two common types of pills?
ANSWER
A
Further Questions To Be Asked
➢ History of migraine
➢ History of liver disease
➢ History of clotting disorder
➢ History of DVT
➢ History of uncontrolled BP
➢ H/O breastfeeding
➢ History of breast or ovarian cancer personal or in family
➢ Ask about smoking
➢ Any medication if she is taking currently.
➢ Does she have any anxiety
➢ Weight gain
➢ Elevated lipid profile
➢ Family H/o MI
B
Relevant Examination
• BP
• Weight
• Breast Examination
• Pelvic exam including Pap smear swab
C
Side effects Regarding OCP Choices: I will advise her regarding the two choices’ side effects as
under;
Norethisterrion and levonorgestril pills: May cause;
Bleeding
Acne
Headache
Weight gain
Amenorhea
Breast pain
Increased vaginal discharge.
-COC increases risk of breast cancer,ovarian cancer and thromboembolism.
Progestin only pill: May cause;
Menstrual changes.
Amennorrhea.
SIGNIFICANT TOPICS WITH SAQs 188
SIGNIFICANT TOPICS WITH SAQs 189
112.
Mrs. Altaf is a 35 years old bank employee. She presents with a 2 weeks history of vaginal discharge.
The discharge is whitish-yellow along with associated itching in the vagina. This is her second episode of
vaginal discharge.
ANSWER
A
Most Likely Diagnosis:
Vaginal Candidiasis
Logical Justification: 2 weeks history of whitish yellow vaginal discharge with itching points to
vaginal candidiasis.
B
Investigations:
• Test pH of vaginal secretions with narrow range pH paper (pH < 4.5 = vaginal candidiasis/
physiological). If pH > 4.5, BV or TV. usually if no other risk factor( lie
pregnancy,postpartum, instrumentation, terminaton, recurrent infections, treatment
failure) and abnormal finding is there, and diagnosis of vaginal candidiasis is made then no
need to do further invesigations.
• Other investigations to concider are:
o High vaginal swab for M, C &S
o Endocervical swabs for gonorrhea & chlamydia..
o For partner: Send urethral swab or first catch urine sample forculture and nucleic
acid amplification testing (NAAT)
C
Management:
History:
➢ vaginal discharge( itchy, offensive, colour), vulval soreness and irritation, lower abd pain n
dyspareunia, heavy periods, intermenstrual or post coital bleeding, fever, vulval pain,
method of contraception, IUCD or IUS
➢ Sexual history: recent new partner or multiple partners,STI symptoms orh/o STI in partner
➢ Medical history: Diabetes,immunocompromise, Cushing’s or Addison’s disease, h/o radio
or chemotherapy, recent antibiotic or steroid use
➢ Attempts to treat current symptoms
Examination: If diagnosis is unclear or any risk factor is found; perform abdominal, bimanual
pelvic and vaginal speculum examination, look for;
✓ Tenderness inlower abdomen
✓ Cervical erosion
✓ Contact bleeding
✓ Discharge
✓ Foreign body
✓ Ulcer
SIGNIFICANT TOPICS WITH SAQs 190
113.
A 30 year old married lady comes to your clinic complaining of excessive discharge from her vagina,
which is quite concerning for her.
A. Enlist 6 relevant points you would ask in the history to evaluate her condition.
B. Enlist 4 causes you would consider in this patient.
C. Enlist 2 office based tests which are performed in this condition.
D. Outline drug treatment for three differentials listed in “Answer B”.
ANSWER
A
Six Relevant Points in History:
1) Since how long having this problem? Nature of discharge, profuse, scanty, foul smelling.
2) Associated symptoms like itching, pain, soreness, burning.
3) History of postcoital bleeding/dyspareunia.
4) Consistency of discharge
5) Previous/past history of STI
6) Other associated symptoms, fever, nausea, vomiting, lower back pain, related to UTI?
7) History of contraception/IUCD use.
B
Four causes to Consider:
1) Chlamydia
2) Bacterial vaginosis
3) HVS (genital herpes)
4) Giardia
5) Candida
Clues for D/D of vaginal discharge:
➢ Sexually active/vaginal discharge/deep dyspareunia smear for gonococcus –ve. Motile
flagellated organisms will be seen (protozoa).
o Diagnosis: Chlamydia trachomatis + PID
o Treatment: Tetracycline
➢ Burning dysuria, thin discharge + multiple small ulcers
o Diagnosis: Genital Herpes Simplex
➢ Foul smelling discharge with smear demonstrating squamous cells + clue cells (clue cells are
irregularly bordered squamous epithelial cells whose cell outlines are oblitereated by
sheets of small bacteria)
o Diagnosis: Bacterial vaginosis
C
Office-Based Tests to be Performed:
1) Speculum evaluation
2) Vaginal smear
D
Drug Treatment for Differentials
Vulvaginal Candidiasis (any one):
1) Clotrimazole 1% cream 5 g intravaginally for 7-14 days
SIGNIFICANT TOPICS WITH SAQs 192
114.
A 30 year old married woman who has a 5 year old child presents to your clinic complaining of left
breast nipple discharge which she noticed while bathing 1 day ago.
ANSWER
A
Questions in History:
➢ Ask about history of amenorrhoea and LMP (to rule out pregnancy/amenorrhoea because
of hormone imbalance.
➢ Duration/colour of discharge.
➢ Pain or tenderness in breast – related to menstrual cycle or not.
➢ Drug history (as metoclompramide, domperidone, phenothiazines, haloperidol, SSRI and
opions increase prolactin)
➢ Family history of breast cancer or any other cancer (to rule out pituitary adenoma)
➢ Autoimmune disease (hypothyroidism leads to hyperprolactinaemia)
➢ H/O stress/anxiety
B
Clinical Findings
1) Galactorrhoea
2) Anovulatory amenorrhoea
3) Headache and impaired vision (prolactinoma)
4) Vaginal dryness and brittle bones (old age)
C
Investigation:
1. Serum prolactin (might be increased)
2. Thyrotropin releasing hormone
3. MRI brain (to rule out prolactinoma)
4. TSH
5. Mammography
SIGNIFICANT TOPICS WITH SAQs 194
115.
Mrs. Majeed, a 35 year old lady, presents to your clinic with lower abdominal pain for the last 2-3
months. It is a dull continuous pain, but slowly increasing in intensity and worsens during intercourse.
On examination, she is tender in the lower abdomen and both fornices. She has acute pain on
movement of the cervix.
ANSWER
A
Pelvic inflammatory disease
B
I will ask about:
-h/o pain site, severity, character, radiation.
-h/o fever,l ow or high grade
-Menstrual : heavy periods, intermenstrual bleeding,dusmenorrhea.
-Sexual history: post coital bleeding, deep or superficial dyspareunia.No of sexual partners.History
of STD in partner.
-obs history LMP, ectopic pregnancy or abortions, --- h/o vaginal discharge purulent, watery,
white?
-method of contraception, use of IUCD, previous instrumentation.
-H/o dysuria
-Past history of PID n treatment?
-Cervical swabs in past
-drug history
C
Investigation:
- high vaginal swab and endocervical swab for microscopy, culture n sensitivity
-chlamydia/ gonorhea screen
-FBC may show leukocytosis, ESR (raised.)
-CRP (raised)
- pregnancy test
D
Management:
-After taking detailed History as mentioned above I 'll do following examination:
-GPE,and vital signs especially Temperature.
-Lower abdominal or pelvic palpitation for tenderness or mass
-Bimanual pelvic examination for cervical motion tenderness, uterine tenderness.
-Vaginal Speculum examination: to see any discharge, contact bleeding, erosion, any IUCD.
INVESTIGATIONS:as mentioned above.
Non pharmacological treatment:
-Reassurance of patient.
SIGNIFICANT TOPICS WITH SAQs 195
-Explain to her about nature of disease treatment options and consequences of not treated
adequately.
-advice rest and sexual abstinence.
-If IUCD is there, remove it as symptoms are getting worsened and advice realternative
contraceptive.
Pharmacological:
-provide analgesia
-Treast with tab Ofloxacin 400 mg bd and metronidazole 400 mg bd for 14 days.
-Alternative is inj. ceftriaxone 500 mg IM stat then oral doxycycline 100 mg bd and metronidazole
400 mg bd for 14 d.
F.up and Referral:
- if no significant improvement after 48 hrs, admit.
consider Referral to gynaecology if:
-very sick
-Pelvic abscess suspected
-pregnant
-not responding to treatment
-No obvious cause is found.
SIGNIFICANT TOPICS WITH SAQs 196
116.
A 30 year old lady presents with irregular periods for the last 1 year. Her last LMP was 2 weeks ago. She
has associated excessive hair growth on face with a BMI of 29 kg/m2.
ANSWER (PCOS)
A
Detailed history of presenting symptoms.
i) For how long she observed hair growth.
ii) Menstrual history
a. Age of menarche
b. Menstrual flow
c. Irregularity for how long
d. Any associated symptoms like pelvic pain.
iii) Any changes in voice.
iv) Energy level, sleep pattern
v) Diet and exercise
vi) If patient married for how long she is married, did she conceive before.
vii) Recent weight gain in last few months
viii) h/o acne
ix) family history
B
PCOS
C
Investigations
• FSH/LH ratio
• Ultrasound pelvis
• Serum testosterone level
D
Non-pharmacological
Reduce weight
Excercise
Healthy diet
Avoid food which can aggravate acne like spicy food chocolate
Pharmacological
For Acne
o Benzoyl peroxide + Clindamycin gel up to TID for 4 weeks.
For irregular menses (if patient is not pregnant and not desirous of fertility
o OCP 1 OD every day including 7 days placebo at least for 6 months to regulate cycles.
o Tablet Metformin 500 mg with meals for 6 months
o Follow up in 1 month
SIGNIFICANT TOPICS WITH SAQs 197
If patient married and desires pregnancy, still I will advise her to let cycle regularize first,
then will try fertility pill, i.e. clomiphene citrate.
Tab CaC1000 (Ca+Vit D) 1000 + 400 IU 1 PO OD
SIGNIFICANT TOPICS WITH SAQs 198
117.
A 45 year old lady P4+0 presents with heavy periods for 1-1/2 years. Her LMP was 15 day ago. Duration
of periods is 7 days and length of cycle varies from 15 to 30 days.
A. What are the pertinent features on history that need evaluation? (At least 4)
B. List the steps of examination
C. What are the most likely differentials? (any 2)
D. What are the 4 most relevant investigations?
E. Write the pharmacological management plan.
ANSWER
A
Number of tampons or pads used per day,
-use of double protection to prevent leaks
-flooding and clots to guage bleeding
- how periods affect life and activities
- intermentrual or post coital bleeding
-irregualr bleeding( present in this case)
-dyspareunia
-pelvic pain
- prementrual pain and symptoms
-h/o pcos, bleeding disorder, tamoxifen therapy, endometriosis, PID, thyroid disease, DM, adrenal,
kidney or liver disease,
- contraception method, how long u hv been using it , any change in last yrs
B
- abdominal and bimenual examination of uterus, chi for pappable masses, tenderness, polyp
-general examination: signs of anemia, obesity, routine examination of vital signs n sign of any
systemic or endocrine disorder
C
d/d: -dysfunctional uterine bleeding(50%)
-fibroids
-presence of IUD
- pelvic infection
-endometriosis
Less likely
- congenital bicornuate uterus
- endometrial/ cervical polyp
- endometrial carcinoma or other hormone producing tumors
- bleeding tendency
D
according to history n investigation,
- cervical smear if due
-cbc to chk anemia
-endometrial assesment bc she is 45 yrs old
-u/s pelvis to see anatomy n exclude fibroids
SIGNIFICANT TOPICS WITH SAQs 199
E
First discuss plan for further pregnancies, chk for risk of dvt, thromboembolism n other
contraindication if using hormonal therapy
- treat anemia if present, iron therapy
-if uterine abnormality is ruled out and no other pathology found then fit IUs: progestigen only
intrauterine system (mirena)
- if not significantly improved after six months, add combined oral contraceptive pills
- if no improvement or non hormonal therapy is prefered then trial of mefenamic acid 500mg tds
or tranexamic acid 1g tds, start on day 1 of period, and continue till days of heavy flow, review
after 3 months
-continue heavy bleeding , try norethisterone 15 mg od from day 5-26 of cycle, or inj progesteron
long acting
- referal to specialist if no response to treatment
SIGNIFICANT TOPICS WITH SAQs 200
118.
Under what circumstances will you consider referral of a 41 years old patient with heavy menstrual
periods to a gynecologist?
ANSWER
Criteria for Referral:
➢ Very heavy bleeding with shock
➢ Anaemic secondary to heavy bleeding
➢ Failure of medical treatment in women
➢ Irregular or heavy periods in a woman of any age with structurally abnormal uterus.
➢ Patient has PCO, where anovulatory menstrual abnormalities do not respond to medical
therapy.
➢ Patient over 40 years of age may require referral for irregular periods because it may be
difficult to distinguish between intermenstrual and menstrual loss.
➢ Recurrent heavy bleeding and if family is complete.
➢ Women with fibroids that are palpable abdominally or who have intracavity fibroid and or
whose uterine length as measured at ultrasound or hysteroscopy is greater than 12cm
should be offered immediate referral to specialist.
➢ Urgent two-week referral if;
o Suspicion of underlying cancer (pelvic mass, abnormal cervix or cervical smear,
irregular bleeding, repeated intermenstrual bleeding or postcoital bleeding.
o If despite 3 months of drug treatment, heavy bleeding persists and is interfering with
the quality of life. Failure is best based upon the women’s own assessment.
o If menorrhagia and patient wishes to explore the possibility of surgical intervention
rather than persist with drug treatment.
SIGNIFICANT TOPICS WITH SAQs 201
119.
A 25 year old recently married woman presents to clinic wanting information
A. What tests are done for diagnosis of cervical cancer and how often should this be done?
B. What is the latest approach to prevent cervical cancer and how would you recommend this?
C. What are the risk factors for cervical cancer and what lifestyle modification you would suggest?
ANSWER
A
Tests for Diagnosis of Cervical Cancer:
Cervical smear (Pap smear):
1: Liquid based cytology (LBC):
• Cells r collected from cervix with a brush, head of brush is either broken off or rinsed into a
vial containing preservative fluid n sent to laboratory.
• Timings: Avoid menstruation, ideal mid-cycle
2: HPV screening: Infection with HPV 16, 18, 31+33 is associated with cervical carcinoma. Women
with HPV infection are 70% more likely to develop high grade cervical abnormalities. So, Pap
smear showing borderline or low-grade abnormalities are tested for HPV
• Cells from LBC samples with borderline or low grade abnormalities are tested for HPV.
3. Colonoscopy:
• If LBC is positive for HPV, patient is referred for colonoscopy
• If Pap smear showing moderate or severe dyskaryosis, also referral for colposcopy.
• Screening interval
o 3 yearly from 25-49 yrs
o 5 yrly from 50-64 yrs
B
Prevention of Cervical Carcinoma:
HPV Vaccination:
• Routine vaccination of girls 11-12 years
• 3 doses of HPV2, HPV4 + HPV9 at 0, 2, 6 months.
• All unvaccinated females from 13-26 must be vaccinated
C
Risk Factors for Cervical CA
1) HPV infecton (subtypes 16, 18, 31, 33) 99.7% of cervical cancer are associated with HPV
DNA
Women with HPV are 70 times more likely to develop high grade cervical abnormalities.
2) Smoking (increased risk of squamous CA)
3) Early age of first intercourse.
4) Early age of first pregnancy
5) Multiple serial partners (≥ 6 lifetime partners)
6) Low socioeconomic class
7) H/O STIs
8) Immunocompromised (HIV, organ transplant, poor nutritional status)
9) H/O cancer in patient and family.
10) Method of contraception – 5-year combined hormone contraceptive increases risk
SIGNIFICANT TOPICS WITH SAQs 202
120.
Write short essay (short note) comprising 200 – 300 words on primary dysmenorrhea and its
management.
ANSWER
Primary Dysmenorrhea:
✓ Dysmenorrhea is painful cramping, usually in the lower abdomen occurring shortly before
or during menstruation or both.
✓ It occurs in the absence of identifiable underlying pelvic pathology.
✓ It is thought to be due to production of uterine prostaglandins during menstruation.
✓ Risk factors include age < 30 years, BMI < 20 kg/m2, smoking, menarche before age 12
years, duration of bleeding, longer menstrual cycles, irregularly or heavy menstrual flow,
irregularity or heavy menstrual flow.
✓ Onset after 6 months of menarche.
✓ Assessment of Primary Dysmenorrhea:
o History:
▪ Age at menarche
▪ Duration and interval between menstrual cycles
▪ Onset and duration of cramps
▪ Presence/absence of nausea, vomiting, diarrhea, back pain, dizziness or headache
during menstruation.
▪ Severity of symptoms: Including impact on daily activities.
▪ Medications used for it.
▪ Sexual History
▪ Psychological symptoms including mood swings, nervous tension, or irritability
(when severe, termed PMDD)
▪ Physical - abdominal bloating.
▪ Overweight
▪ Breast tenderness
▪ Headache
▪ Decreased visuospatial and cognitive ability
▪ Increase in accidents.
o Examination:
▪ General Exam: Anaemia
▪ Abdomen: For masses
▪ Pelvic: If sexually active
o Investigations:
▪ For primary dysmenorrhea, no test needed, but if red flag symptoms present, then
investigate accordingly
✓ Management:
o Initial Treatment: 3 months.
o Nonpharmacological:
▪ Keep a diary to establish pattern for > 2 months.
▪ Wear loose clothes
▪ Ensure adequate sleep and regular exercise
SIGNIFICANT TOPICS WITH SAQs 204
▪ Diet: Low fat/low salt, contains plenty of fruits and vegetables and complex
carbohydrates (e.g., bread, pasta, rice and potatoes)
▪ Decrease fluids or eat diuretic foods (strawberries, watermelon, aubergines, prunes,
figs, parsley) to ease fluid retention.
▪ Consider drug therapy or CBT if diet/lifestyle measures fail.
▪ Referral to gynecology/mental health services if symptoms severe or primary care
management is ineffective.
o Pharmacological:
▪ COC – First line combined new generation pills, Yasmin or Cilest – given cyclically or
continuously.
▪ Low dose oestrogens – second line treatment 100 mcg of oestradiol patches. Avoid
giving unopposed oestrogen, combine with dydrogesterone 100 mg from day 17 to
28 or IUS if the uterus is intact.
▪ GnRH: Reserved for specialist care, usually given in combination with add on HRT.
▪ Antidepressants:
• Decreases physical as well as psychological symptoms.
• First-line treatment that can be given continuously or just in the luteal phase (day
15 to 28)
▪ Other drugs.
• Diuretics – spironolactone, effective for bloating, breast tenderness
• NSAIDS – For pain and to reduce menstrual bleeding
SIGNIFICANT TOPICS WITH SAQs 205
121.
A 24 year old woman visits her physician for her routine annual examination. She has no
complaints. She is taking oral contraceptive pills for 3 years and she has regular menstrual
cycles. Her Pap smear shows irregular, hypochromic nuclear enlargement and perinuclear
cytoplasmic vacuoles.
ANSWER
A
Diagnosis:
Cervical dysplasia
B
Causative organism:
Human papilloma virus (HPV)
C
Next Step of Action:
Findings of Pap smear cytology (koilocytosis, cytonuclear changes, e.g. irregular nuclear
contours with hypochromasia) are suggestive of low grade squamous intraepithelial lesions
(LGSIL), 24 year old asymptomatic female with COCP intake, having impression of LGSIL on Pap
cytology qualifies for;
o Repeat Pap smear cytology + Reflex HPV screening – at 6-month interval
o HPV DNA – PCR assay – for HPV typing (16, 18 – precancerous oncogenic).
o If it shows cellular reversion, repeat only Pap smear cytology at interval of another 6
months (to make sure infection clearing due to patient’s cell mediated immunity)
If, however, features of persistence or progression for cytonuclear atypia seen in 3 consecutive
borderline smears of symptomatic patient, she must be referred to specialized care unit for;
o Colposcopy – for visualization of transformationzone and taking direct punch biopsy
o Cervical biopsy – for histology and adequate grading of CIN
Management: Depends on outcome of serial Pap smears
o Suspicious findings – repeat Pap smear 6 monthly until comes
▪ Normal –
• then 3 yearly screening (25-50 years)
• 5 yearly screening (51-64 years)
▪ Conservative watchful monitoring
o Persistence, progression in dyskaryosis and cytonuclear atypia – 2 consecutive smear
results, then referred to specialised gynaecological setup for surgical treatment
option.
▪ Large loop excision of transformation zone (LLET2)
▪ Hysterectomy
SIGNIFICANT TOPICS WITH SAQs 206
122.
A 55 years old woman has presented with post-menopausal bleeding of one month duration. She had
her menopause at the age of 52. She has only one son. She is a known diabetic and hypertensive. On
examination, she is healthy looking and obese. Her abdomino-pelvic examination is normal.
ANSWER
A
Most Likely Diagnosis:
Endometrial CA
B
Most Likely Differentials:
❖ Atrophic vaginitis
❖ Cervical polyp
❖ Endometrial thickening secondary to HRT with breakthrough bleeding
❖ Endometrial polyps
❖ Submucosal leiomyoma
❖ Non-genital differentials
o CA bladder
o Cystitis/(severe)
C
Features in History Pointing to This Condition:
• Postmenopausal lady
• Low parity
• Diabetic
• Obese
• P/V bleeding afer menopause (PMB)
D
Management Plan:
1) Detailed History: In addition to info given in the question, ask about
a) Detailed history of symptoms, severity, mixed with discharge, heavy bleeding.
b) Constitutional Symptoms: Fever, night sweats, any lump on any other site, appetite,
bowel habits.
c) Use of medication like HRT, COCP, anticoagulants, tamoxifen
d) H/O vaginal discharge. Associated symptoms, pain, severity, recent weight loss.
e) Postcoital bleeding/dyspareunia
f) Vulval itching, STIs
g) Sexual partners
h) HPV vaccination or screening done before.
i) Previous history of IU system in situ?
j) Recent start of estrogen + combined progestin regimens for last 6 months.
SIGNIFICANT TOPICS WITH SAQs 207
123.
Amina, a 38 years old wife of a laborer, presents with amenorrhea for 3 months, nausea and breast
tenderness. She has 4 living children (ages 16 years to 2 years)
A. What are the key elements of antenatal care required if you confirm that she is pregnant?
B. What other advice would you give this lady in view of her age and socio-economic background?
ANSWER
A
Key Elements of Antenatal Care:
HISTORY
Menstrual History:
• LMP
• EDD
• Contraception
• Previous pregnancy
Review of Systems/Medical History:
• Pain
• Skin
o Dark striae
o Strias
o Rash or not – vesicular
• Constipation
• Vomiting
• Backache
• Carpal tunnel
• EPD
• GDM
• Previous h/o chickenpox
• Rubella
• h/o epilepsy
• varicose veins
• haemorrhoids
• Past h/o HTN & DM
Past Surgical History:
Past Obstetrical History:
• Anti-D
• Post-partum haemorrhage
EXAMINATION:
• Vital recording – BP, pulse, heart rate, RR
• Height, weight, BMI
• Nutritional status, hydration level
• General look – pallor, signs of self neglect, mental calibre, signs of domestic violence
• Head & Neck – check teeth, gums, thyroid
SIGNIFICANT TOPICS WITH SAQs 209
Safety Netting:
Visits at 16, 24, 36, 38 weeks of gestation
USG:
o At 20 weeks to see birth defects
o At 30 weeks to confirm dates
SIGNIFICANT TOPICS WITH SAQs 211
124.
A 55 year old lady with no comorbids presents with postmenopausal bleeding since one month.
She had her menopause at the age of 51 years.
ANSWER
A
Possible Causes
• Endometrial cancer
• Cervical cancer
• Breakthrough bleeding
• Atrophic vaginitis
B
Questions in History:
• Use of HRT
• Duration of HRT
C
Management:
Detailed history
➢ last periods, menstrual history
➢ gynae/sexual history
➢ Last Pap smear
➢ Any vaginal dryness complaint
Examination
• Abdominal
• Bimanual pelvic and vaginal speculum ( look for dryness , polyp , erosion , abnormal
bleeding , bleed to touch)
Investigations
✓ Pap smear / transvaginal usg along with endometrial biopsy, hysteroscopy
Non pharmacological
Wash with lukewarm water
Avoid soap perfumes n disinfectant
Pharmacological
Apply estrogen vaginal cream BD
Cyclic hormonal therapy
Combined pill for 3 weeks with 2 weeks off
Treat anemia if present.
Excluding surgical issues: If any, referral to gynae for endometrial ablation or TAH & BSO.
SIGNIFICANT TOPICS WITH SAQs 212
125.
A 62 year old woman presents with vaginal bleeding for the last one month.
ANSWER
A
Differential Diagnosis:
❖ Endometrial CA
❖ Atrophic vaginitis
❖ Endometrial hyperplasia of polyp
❖ Cervical polyps, malignancy
❖ Submucosal leiomyoma
❖ Breakthrough bleeding
B
Investigations:
Choice of investigations depends on detaild history and clinical examination.
History: Ask for;
• Onset, severity, association with pain of the of the pv bleeding.
• Menstrual history
• Parity
• Comorbidities – DM, HTN
• Malignancies – breast CA
• Drugs history
o COCP, tamoxifen, antihypertensives, anticoagulants
o HRT history:
▪ Estrogen only pills
▪ Combined estrogen-progestin regimen
• Obesity
• Smoking
Relevant Examination:
• Abdominal Examination – look for ascites, distension, scar marks
• Per speculum – atrophic changes in vagina, vulval or cervical pathology (polyps)
• Bimanual Examination – uterus size, motility, cervical motion, adnexal masses.
Imaging Investigation: Not required if started taking estrogen with continuous progestin therapy
for last 6 months only.
1. TVUS – for endometrial thickness
a. ET > 4 mm
i. Suggestive of endometrial CA
ii. Endocervical sampling for histopathology is indicated.
1. Pipelle biopsy
2. Pap smear
2. Hysteroscopy
SIGNIFICANT TOPICS WITH SAQs 213
a. Indicated if;
i. On TVUS ET < 4 mm
ii. For cervical polyps
iii. For submucosal fibroids
3. Lab Investigations: Pre-op workup where indicated;
a. FBC with platelets
b. Blood grouping and cross matching
c. BSR
C
Treatment: According to diagnosis
1) Atrophic Vaginitis: Estrogen vaginal cream or pessaries.
2) Malignancy (endometrial, cervical) – Referral for surgical consultation (TAH + BSO)
3) Breakthrough Bleeding – Stop HRT and refer for consultation.
SIGNIFICANT TOPICS WITH SAQs 214
126.
Mrs. Arshad is a 33 years old executive. She attends your clinic with history of vaginal discharge
for the last two weeks. The discharge is whitish yellow with itching in the vagina and vulva. This
is her second bout of vaginal discharge in the last three months.
ANSWER
A
Three Likely Causes of Symptoms:
1) Vaginal candidiasis (white discharge and itching)
2) Trichomonisasis (yellow discharge and itching)
3) Chlamydia (discharge and itching)
4) Bacterial vaginosis (less likely)
B
Investigations:
1) High vaginal swabs for
a. pH and whiff test
b. Microscopy (wet film with gram stain)
c. Baterial and fungal culture for specific pathogens
2) Endocervical swabs or first-void urine samples
a. If gonorrhea, chlamydia or mucopurulent cervicitis is suspected for.
3) Endocervical swab for M, C&S.
4) First void urine for PCR to detect chlamydia or Neisseria gonorrhae.
5) Cytological examination with chronic cervicitis
6) Lesion biopy if necessary.
C
Management
Detailed History: LMP, parity
➢ Discharge:
o How does it differ from your normal discharge?
o Colour, consistency, volume, odor
➢ Symptoms Associated with altered discharge
o Vulval discomfort or irritation.
o Vulval itch
o Abdominal bleeding (postcoital or intermenstrual)
o Pelvic pain
o Dyspareunia
o Dysuria
o Genital ulceration
o Systemic symptoms
➢ Sexual History:
o Are you sexually active at the moment
SIGNIFICANT TOPICS WITH SAQs 215
4) Chlamydia
a. Azithromycin 500 mg 1 gram PO single dose
b. Doxycycline 100 mg BID 7 days
5) Gonorrhea:
a. Ceftriaxone 250 mg IM + Azithromycin 1 gram PO stat OR Doxycycline 100 mg BID x 7
days.
SIGNIFICANT TOPICS WITH SAQs 217
127.
A 15 years old unmarried girl presents with acute diffuse central abdominal pain for the last 4 hours.
This is accompanied by nausea and she has vomited once. Temperature is 37.5 C and pulse is 86/min
and BP is 110/70 mmHg. She has diffuse tenderness all over the abdomen.
ANSWER
A
Most Likely Diagnoses:
Acute abdomen can be caused by;
❖ PID
❖ acute appendicitis
❖ dysmenorrhea
❖ ruptured ectopic pregnancy
❖ renal colic
❖ intestinal obstruction
B
Initial Investigations:
1) Dip stick
2) MSU
3) urea and electrolytes
4) CBC and ESR
5) CRP
6) USG abdomen
7) KUB X-Ray
8) Plain X-ray Abdomen, to detect dilated loop of bowels/fluid level erect/supine
C
Management:
• NPO
• History with particular attention to pain, i.e., onset, nature and other features.
• Vomiting/contents of vomitus.
• Examination.
o Sign of peritonitis.ie tenderness, guarding.
o Sign of obstruction, i.e., distention, gut sounds.
o Pelvic n rectal examination.
o Tongue, i.e. furred in most acute abdominal diseases.
• In acute abdomen diclofenic/analgesia contraindicated. Find out the cause and treat it
accordingly. If the diagnosis is uncertain, refer patient to A&E for the appropriate action
• Pharmacological Treatment/Referral:
Maintain IV line
Give IV fluids
If PID, antibiotics/referral
SIGNIFICANT TOPICS WITH SAQs 218
128.
A 36 years old para 5+0 presents with a haemoglobin of 8 gm/dl at 20 weeks’ gestation.
ANSWER
A
Significant Aspects of History:
1) Obstetric and menstrual history: time gap bw consequent pregnancies, when was the last
pregnancy, hb at start of pregnancy, is there any bleeding, excessive vomiting in pregnancy,
abortions, length of cycle, days of mensruation , heavy periods, IUCD, anemia in previous
pregnancies.
2) Drug history: Use of iron, folic acid, multivitamins, cytotoxic drugs
3) General health: Ongoing infections, hypertention, diabeties millitis, hemoglobinopathies in
pt or family, h/o malignancy
B
Additional Investigations:
Investigations could be divided into two groups.
a) To investigate symptoms of pregnant women.
i) First of all CBC with peripheral smear (To check type of anemia).
A) If smear shows microcytic, then order S. ferritin level. Most probably in this case, patient
has iron deficiency anemia.
B) If smear shows microcytic, order vitamin B12 level, folate level
ii) Urinalysis (for protein, infection)
b) To order routine labs necessary 28 week gestation: As it is patient’s first visit, order all routine
labs, i.e.
i) Blood group
ii) OGTT
iii) HBsAg, Anti-HCV
iv) USG pelvis (anomaly scan)
C
Management Options:
1. Iron tablets (if iron deficiency):
• Ferrous sulfate tablets 200 mg TDS.
• Repeat Hb in 2 weeks.
• If there is no response to oral iron, exclude occult infection (e.g. UTI); check
haematinics; consider referral for parenteral iron.
2. Parenteral Iron (If oral is not tolerated, patient is not compliant, woman is approaching
term)
3. Vitamin B12 supplement (if vitamin B12 deficiency):
• Treat with vitamin B12 parenterally (hydroxocobalamin IM—initially 1 mg 3 times
per week for 2 weeks then every 2–3 months).
SIGNIFICANT TOPICS WITH SAQs 220
129.
A 32 years old para 5+0 who has had all her previous deliveries at home presents at 28 weeks pregnancy
with dyspnoea and easy fatigability. She has never had antenatal care. The last delivery was 2 years
ago. On examination, she is pale, has a slight ankle edema. Her blood pressure is 130/90 mmHg. Chest
and heart sounds are normal. The fundal height corresponds to period of gestation. Hemoglobin report
done by a local doctor one month back shows Hb of 8 g/dL.
ANSWER
A
Probable Diagnosis:
Anemia in pregnancy
B
Investigations:
1) Haemoglobin and hematocrity
a. Hb - < 10 gm% (N 11-14 gm%)
b. PCV < 32% (N 32% - 36%)
c. RBC count - < 3.2 million (N-4 to 4.5 million/cm3)
2) Peripheral Smears:
a. Thin smear – RBC morphology
b. Thick smear – useful for identifying parasites/malaria
3) Red cell indices
a. MCV (low)
b. MCHC (raised)
c. RDW (raised)
4) Special Tests
a. Serum Iron and TIBC
b. Serum ferritin
c. Serum transferring receptor
d. Transferrin % saturation
5) Urine for hematuria, pyruia, and proteinuria (becaue of ankle oedema and BP 130/90)
6) Stool exam for occult blood, ova, cysts
7) Routine Pregnancy Investigations:
a. Blood grouping
b. HbsAg + VDRL
c. USG of pregnancy
C
Management Counselling:
Take detailed history, examination according to weight of pregnancy + do investigations for
proper diagnosis.
Non-Pharmacological:
o Address ICE about anaemia in pregnancy
o Explain risk factors (pre-term, low birth weight)
SIGNIFICANT TOPICS WITH SAQs 222
o Diet modification:
▪ Brown rice
▪ Pulses and beans
▪ Nuts and seeds
▪ Meat, fish and tofu
▪ Eggs
▪ Dried fruit
Pharmacological Treatment:
o Tablet iron sulfate 200 mg TDS.
o If not tolerating oral iron then;
▪ Iron Dextran OR Iron sucrose
▪
o Tablet folic acid 5 mg OD
o Add vitamin C tablet (as it increases iron absorption)
o Avoid Tablet Calcium (as it decreases iron absorption)
Follow the patient after 2 weeks.
SIGNIFICANT TOPICS WITH SAQs 223
130.
A 20 years old newly married female presents with history of secondary amenorrhea for two months.
She is complaining of morning sickness and fatigue.
ANSWER (Pregnancy)
A
Further Information: Most common cause of sec amenorrhea is Pregnancy
➢ Pregnancy (Do BHCG to confirm it first) LMP, usual cycle, fertility problems, contraception,
desirability of pregnancy
➢ Previous pregnancies n outcome ( any abortion), breast feed.
➢ Past medical history (DM, HTN, hypothyroidism, polycystic ovary disease, psychiatric
illnesses, drugs , allergies, any pelvic surgery, any prolactinoma symptoms – vision changes)
➢ Family history of DM, HTN, genetic abnormalities
➢ Social history ... smoking, alcohol
➢ Any h/o starting high-intensity exercise
B
Examination
GPE/Vital Signs: Check weight height calculate BMI as obesity is also a cause of missed periods,
anemia, pulse, BP
Lungs Exam
Heart Exam
C
MANAGEMENT
History & Exam: As above
Investigations
CBC, FBS, HBa1c, OGTT (If suspecting gestational diabetes), blood grouping ,rhesus status ,syphilis ,
rubella , urine culture , HbsAg, HIV, Early USS at 10 weeks Anomaly screen18-20 weeks
Treatment:
1) Non-Pharmacological:
a. HEALTH EDUCATION
a. Health promotion
b. Antenatal n parental classes
c. Choice of delivery method n place available
d. Worried about pregnancy or other domestic problems
b. Have a regular follow up with antenatal clinic
c. ROUTINE check up to look at BP HEIGHT WEIGHT oedema, urine for protein , fundal
height , FHS.
2) Pharmacologial:
a. Folic acid 5 mg once daily till 3 months this will Help to prevent the incidence of neural
tube defect.
SIGNIFICANT TOPICS WITH SAQs 224
131.
A 25 year old married female presents with lower abdominal pain on and off for three days. Her last
menstrual period was seven weeks back.
ANSWER
A
Most Likely Diagnosis:
Ectopic pregnancy
B
Differential Diagnosis:
❖ Pelvic inflammatory disease
❖ Vaginosis
❖ Gastroenteritis
❖ Diverticulitis
❖ Missed Abortion
❖ Ovarian cyst
C
Relevant Clinical Examination: Check
• Vitals – look for hypotension, tachycardia, tachypnea
• GPE
• Inspect:
o General overall appearance of discomfort and pain before, during and after
examination.
o Inspect peri-pad amount of bleed if documented on further exploration of history
• Palpate:
o Abdominal examination for rebound tenderness and guarding, masses, abdominal
wall distention. Look for signs of internal haemorrhage.
o Obstetrical examination: Palpate uterine size.
o Check iliopsoas and obturator muscle tests.
o Pelvic Examination: Per speculum examination: Look for P/V bleeding/discharge
o Bimanual Examination: Reassess for cervical dilatation, soft cervical, cervical motion
tenderness, adnexal mass, bulging, cul-de-sac.
• Auscultate: Heart, lungs and bowel sounds.
D
Management:
Detailed History:
Complete Examination + Investigations (labs + radiological – TVUS)
Admit Immediately
Maintain IV line
Take blood sample for relevant investigations
SIGNIFICANT TOPICS WITH SAQs 226
132.
A 25 years old woman, married for 2 years with no issue presents with moderate lower abdominal pain
on & off for 3 days. The pain is associated with nausea and transient fainting attacks. Her last
menstrual period was 3 weeks back.
ANSWER
A
Aspects of History:
1) Ever had the pain before as above?
2) History of menorrhagia?
3) Periods regular or irregular?
4) History of pregnancy or abortion?
5) History of infertility or ovulation induction?
6) Any association of anxiety, craving or depression.
B
Required Clinical Examination:
1) Bimanual examination: Check cervical motion tenderness, adnexal mass.
2) Transvaginal Scan: Refer to be explored by transvaginal examination
3) Ultrasound Examination
C
Differential Diagnosis:
1) Ectopic pregnancy
2) Premenstrual syndrome
3) Endometriosis
4) Fibroids
D
Management:
Give painkiller and treat the cause.
Treatment/Action Plan: According to diagnosis.
Ectopic Pregnancy:
• Give methotrexate to stop the progression of pregnancy.
• Referral for surgical termination of pregnancy
Premenstrual Syndrome:
• Antidepressants (SSRI)
• NSAIDs
• Diuretics
• Hormonal contraceptives
Endometriosis:
• Hormonal therapy: Oral contraceptive pills and progesterone, danazol)
• Pain medications – NSAIDs.
SIGNIFICANT TOPICS WITH SAQs 228
133.
A 32 year old lady presented with the concern of Subfertility. She got married at the age of 28 but
despite trying has not conceived.
ANSWER
A
Relevant Points on History
From Couple:
-Length of time trying to conceive.
-Timing and frequency of intercourse and any difficulties with it (psychological problems), physical
disability.
From Woman:
-Menstrual history
-Complete obstetrical history – previous pregnancies and outcome
-Previous use of contraception
-Pelvic infection
-Medication use
-Occupational exposure
-Substance abuse
-Alcohol intake, tobacco use
-Previous surgeries on reproductive organs
-Preconception screening and vaccination (rubella, STDs, varicella and cervical cancer)
From Man:
-Previous infertility
-Pevlic or inguinal surgeries.
-Systemic disease
-Injury, infection
-Toxin exposure
-H/O mumps, STIs
-Occupation (exposure to pesticides, x-ray)
B
Common Causes of Infertility:
1. Males:
a. Decreased sperm count (26 to 30% of infertility cases)
b. Sexually transmitted disease.
2. Females:
a. Ovulatory dysfunction/imbalances (21 to 25%)
b. Hormonal imbalances, e.g., overproduction of prolactin hormone decreases
ovulation
c. Tubal dysfunction (14 to 20%)
3. Unexplained
SIGNIFICANT TOPICS WITH SAQs 230
C
Investigations:
1) Semen analysis
2) Day 21 progesterone level
3) USG pelvis
4) S. TSH, FSH, LH estradiol levels
5) Refer for HSG to assess tubal patency
SIGNIFICANT TOPICS WITH SAQs 231
134.
A 24 year old recently married woman complains of 6 weeks amenorrhea with irregular bleeding P/V for
2 weeks. On examination, she is pale with a heart rate of 120 beats/min and blood pressure of 80/60
mmHg. Abdomen is tender and rigid.
ANSWER
A
Most likely Diagnosis:
Ruptured ectopic pregnancy
Logical Justification: 6 weeks history of amenorrhea, abdominal pain, bleeding PV and tenderness,
guarding, tachycardia, hypotension, all point to the above-mentioned diagnosis.
B
Investigations:
• Urine for pregnancy test.
• Serial Quantitative: β-HCG levels. In normal pregnancy, B-HCG level doubles in 48-72
hours, but it will be less in ectopic pregnancy but B-HCG level will be above discriminatory
zone at which intrauterine pregnancy is visualized, it is;
o 700 to 1000 mIU/ml – transvaginal USG
o Up to 6000 mIU/ml – abdominal USG
• Transvaginal USG: An empty uterus on transvaginal USG when B-HCG level is above
discriminatory zone is ectopic pregnancy until proven otherwise.
• Blood group + Rh – to identify the patient in need of RhoGAM
• Send blood for crossmatching – as patient may need blood transfusion.
C
Immediate Management:
• Counselling and education of patient and family about condition and need for
hospitalization and patient may need operation.
• Stablize the patient as pulse 120/minute, BP 80/60 mmHg, patient may go into shock.
• Maintain double IV line, as impending shock (low BP and bleeding)
o Start Haemaccel IV stat
o R/L 1000 ml IV stat
• Send blood for blood grouping + Rh + cross match.
• Make arrangement for transfer of patient to tertiary care hospital
• Urgent referral to hospital for possible laparascopy or laparotomy as patient’s abdomen is
tender + rigid and BP 80/60 + 110/minute pulse.
• Follow-Up:
o After surgical excision of ectopic pregnancy, weekly maintenance of B-HCG level is
necessary until level is zero to ensure the treatment is complete.
▪ Patient should be on some form of effective contraception until B-HCG level
returns to non-pregnant level. Average time is 2-3 weeks, but may take up to
6 weeks.
SIGNIFICANT TOPICS WITH SAQs 232
SIGNIFICANT TOPICS WITH SAQs 233
135.
A woman with a 32 weeks pregnancy presents to the office for the 2nd week in a row with a blood
pressure of 140/90, 1+ proteinuria, peripheral edema and normal reflexes.
A. How you will further investigate this patient to arrive at the provisional diagnosis
B. Discuss management.
ANSWER
A
Further Investigation to Arrive at Provisional Diagnosis:
History: In addition to info given in scenario, ask for;
➢ Did you experience;
o Headache, visual disturbances (blurring, photophobia)
o GI symptoms – epigastric, RUQ pain, non-settled heartburn.
o Fatigue, malaise
o Altered mental status
➢ Past Obstetrical History – parity, h/o PIH, similar complaint in previous pregnancies,
outcome and mode of delivery, multiple gestation.
➢ Present Obstetrical History – PIH, twin pregnancy, PV bleeding, any vaginal discharge,
antenatal care/visits, compliance to medications and general instruction (bedrest, salt
restriction, hydration), fetal movements.
➢ Personal History – Essential HTN, DM, renal disease, obesity, any contractions (to rule out
pre-term labor)
➢ Personal Habits – Smoking, alcoholism, recreational drugs
➢ Family History – PIH, HTN, others.
Physical Examination:
• Record vitals, weight, BMI
• Inspection
o General condition, orientation, pallor.
o Check facial, hand and pedal edema
• Abdominopelvic Examination:
o Check fundal height, fetal heart rate and fetal movements
• Palpate Abdomen
o Note any RUQ tenderness, visceromegaly (liver, spleen)
• Percuss gently for liver span
• Systemic Examination:
o Auscultate heart and lungs.
o Eyes – fundoscopy
o Perform Neurological Examination for;
▪ Hyperreflexia
▪ Deep tendon reflexes
▪ Clonus
o Check for lower extremiities pitting oedema
Diagnostic Investigation:
✓ CBC
SIGNIFICANT TOPICS WITH SAQs 234
136.
A woman has come to see you 4 days after a normal home delivery by a Trained Birth Attendant
(TBA), her main complain is fever with rigors.
ANSWER
A
History Questions: Ask for;
• Breast symptoms
o Any tenderness, hardening, swelling and redness.
o If breastfeeding? – any associated symptoms.
o Cracked, sore, traumatized nipples, areola and surrounding skin.
o Newborn’s symptoms of oral thrush.
• Urinary tract symptoms:
o Frequency, urgency, dysuria, burning micturition, haematuria.
o Associated suprapubic, lower abdominal pain.
• Genital Tract Symptoms:
o Excessive post-partum bleeding, malodorous discolored lochia, painful perineum.
• Recent Obstetrical History:
o h/o PROM
o h/o prolonged labour
o h/o instrumentation, excessive vaginal examination, unsterile procedures.
o Placental separation – spontaneous, manual, complete, incomplete.
• Other Infections: Ask for symptoms related respiratory, GI and for other viral/bacterial
infectious diseases (i.e. dengue, malaria, viral hepatitis, tuberculosis, etc.)
• Medical History: Comorbidities + procoagulation disorder
• Drug History: Pre-partum, intrapartum and postpartum.
B
Investigations:
o FBC with TLC and platelets
o MSU for urine microscopy, C/S
o High vaginal swab, endocervical smear
o Blood culture
o USG – to assist in diagnosis of retained products of conception.
C
Management:
Detailed History: As above.
Clinical Examination:
• Vital recording
• GPE
o Look for pallor, jaundice, general ill health, signs of self neglect.
SIGNIFICANT TOPICS WITH SAQs 236
Perineal Infection/Cellulitis
o Flucloxacillin 500 mg to 1 gm QID 10-14 days
Referral: In 24 to 48 hours if no improvement/worsening.
SIGNIFICANT TOPICS WITH SAQs 238
137.
A 23 years old woman who delivered her first child 6 days back at home presents to your clinic with
fever and foul smelling lochia for one day. On examination, her temperature is 101 F, pulse 100/min, BP
120/80 mmHg.
ANSWER
A
Most Likely Diagnosis:
Puerperal sepsis most likely due to postpartum endometritis
Puerperal pyrexa
Endometriosis – offensive lochia with pain + tender uterus.
B
Further Questions to Rule Out Other Conditions:
• Onset, duration and course of symptoms (intenstiy, relieving and aggravating factors)
• Any instrumentation during delivery
• Septic/aseptic way of conducting delivery
• Associated symptoms (fever, dysuria, chills, vomiting)
• Any infection/vaginal discharge before delivery
• Intrapartum manipulation (duration of labour, mode of rutpure of membranes, unnecesary
manipulation)
• Postpartum factors (hygiene, medication, cleaning, any retained pieces of placenta)
• Meconium stained liquor
• Any h/o STIs, ulcers, any tear during delivery.
C
Investigations:
1. CBC with differential
2. Blood and urine culture
3. HVS for C& S + STD
4. Wound swab (in case of tear)
5. USG abdomen
D
Management Plan:
Detailed history: As above
Examination:
• GPE + Systemic exam
• Bimanual pelvic exam
• Vaginal speculum examination
Relevant Investigations: As above
Treatment Plan:
• Non-Pharmacological
SIGNIFICANT TOPICS WITH SAQs 239
o
Fluids
o
Good diet and proper rest
o
Hand washing before and after defecation and urination.
o
Application of sterile pads and/or towels to the pelvic/perineal area using sterile
technique.
o If wound:
▪ Wash wound/perineal area with mild soap and lukewarm water BD and apply
Betadine after cleansing with sterile swab with sterile technique.
▪ Removal of any pus/drainage/discharge
• Pharmacological
o Antipyretics (acetaminophen/ibuprofen) as needed.
o Antibiotics
▪ Augmentin (625 mg TID) + Flagyl (400 mg TID) x 7 days OR
▪ If penicillin allergy:
• Non-Lactating: Doxycycline (100 mg BD) x 7 days
• Lactating: Cephalexin 500 mg QID x 7 days
• Follow-up
o Reevaluation in 24 to 48 hours
• Referral/Admission: If;
o No improvement in 48 hours or worsening in 24 hours with antibiotic use.
SIGNIFICANT TOPICS WITH SAQs 240
138.
A 2 months old infant is brought to your clinic with a history of having developed temperature of 39.9 C
following his first set of immunizations. The child is screaming and irritable. General and systemic
examinations show no abnormality.
A. Which immunization(s) is (are) the most likely cause of fever and irritability in this child. Outline
steps of management.
B. What modifications will you make in the immunization schedule for this child?
ANSWER
A
Cause of fever is pertussis vaccine
Management
History
#symptoms of any other infection like cough . flu . vomiting . diarrhea
#H/o problem with vaccine in other siblings
#H/o fits
#H/o epilepsy/febrile seizures in family
Exam mentioned normal except fever
But local exam of injection site needed for redness and swelling
Investigation mostly not done
Treatment
Non pharmacological
Reassurance
Education
Expose the child
Loosen the clothes
Tepid water sponging
Pharmacological
Paracetamol drops 10mg/kg/ dose stat can be given 6 hourly till fever settled
B
Modification in vaccine schedule next two doses would be D TAP i.e. acellular pertussis
Doses would be at scheduled time as previously mentioned in vaccination card
SIGNIFICANT TOPICS WITH SAQs 241
139.
Mrs. Rehman presents to the clinic very concerned. She lives alone with her child as her husband is
working abroad. She visited a friend who had recently arrived from UK. Now 2 weeks after that visit,
she finds out that the friend had swine flu. Mrs. Rehman feels well, but is worried that she or her child
will get the disease.
ANSWER
A
Management
Detailed History: Patient and her child should be asked about following symptoms related to
H1N1.
➢ Fever
➢ Cough
➢ Sore throat
➢ Runny or stuffy nose
➢ Body aches
➢ Headache
➢ Chills
➢ Fatigue and
➢ diarrhea and vomiting (rare)
➢ history of weight loss and poor growth in the child.
Examination: Patient and her child to be screened for following signs of positive H1N1 infection.
• GPE & Vital Signs: In positive H1N1 cases, fever of 100 to 104F may be seen along with
tachycardia. Also fatigued appearance.
• Pharyngitis: Might be present
• Eyes: Red and watery
• Lung Exam: Clear lungs/focal wheezing/rhonchi in positive cases.
Confirmation of Diagnosis: Usually based on clinical suspicion, rapid diagnostic tests may be used
but not 100% exclusive if negative.
Non-Pharmacological:
Inform concerned health authorities/agencies.
Patient education regarding signs and symptoms of H1N1 and addressing ICE.
Patient reassurance
Use of facial masks as it is a droplet spread infection, spread by coughing or sneezing.
Proper handwashing.
Social distancing (avoiding contact with infected people and large gatherings if more people
infected in community) – 5 days for patient and 10 days for child (if H1N1
suspected/diagnosed)
Patient advised if any signs/symptoms develop (as discussed above) to seek medical advice.
Pharmacological:
Oseltamivir (Tamiflu) might be started for patient and child if one of following situation is
met.
SIGNIFICANT TOPICS WITH SAQs 242
140.
A 7 year old boy is brought by his mother complaining that child is having high grade fever and cough for
last 2 days. On examination, his respiratory rate is 40/min, there are no added sounds on auscultation.
According to the IMCI Guidelines.
A. What three most important questions would you ask the mother?
B. What three important things would you look for in examination?
C. Outline your management plan.
141.
A 3 year old male child has been brought to you in outpatient clinic by his mother, giving history of high
grade fever over last 2 days, otherwise no other complaints. Mother has been very concerned as about
one hour ago, she noticed some jerky movements and rolling of the eyes in the child, lasting for about
one minute.
A. What is the most likely diagnosis and give two differentials you must exclude?
B. List 4 relevant points in the history that you would like to explore.
C. List 4 important areas of examination you must conduct.
D. Outline 4 steps in management in order of priority.
ANSWER
A
Most Likely Diagnosis:
Febrile convulsions
Differentials to Exclude:
❖ Epilepsy
❖ Syncope
❖ Meningitis
❖ Encephalitis
❖ Brain Injury because of trauma
B
Relevant History Points:
1) I will further explore regarding fits. Was the jerking generalized or restricted to one area of
body.
a. Did the child lose consciousness
b. What happened after the attack
2) Ask regarding general health of child
a. Past medical history
b. Any serious illness
c. Neurological and developmental problems, heart problem.
3) Birth history
a. Problems during pregnancy
b. H/O birth trauma
4) Family history of epilepsy
C
Important Examination Areas:
1) Full general physical examination
a. Check temperature
b. Assess respiratory/heart rate
c. Capillary return
d. Level of consciousness
2) Examine for source of infection doing
a. ENT examination
b. Neurological examination.
3) Developmental milestones according to age, e.g.
SIGNIFICANT TOPICS WITH SAQs 246
142.
A mother brings in her 9 months old child with a two day history of loose watery stools, 4-5 a day. Child
continues to be breast fed.
ANSWER
A
Important Points in History:
➢ Is there any history of fever ?
➢ Is there any history of vomiting and abdominal pain with this loose motion ?
➢ Is same history of loose motion in other members of family like other siblings?
➢ Any history of eruption of teeth nowadays ?
➢ Any history of recent antibiotic use?
➢ Is child lethargic, irritable
➢ Response to surrounding
B
Relevant Examination Findings:
o Signs of dehydration, sunken eye.
o Tachycardia
o Tachypnea
o Capillary refilling time
o Drowsy
o Acute viral enteritis
o Mild dehydration.
C
Initial Steps in Management
1- Investigations: CBC and electrolytes
2- Treatment
A- Non-pharmacological Treatment:
Continue feeding
Start KUB
B- Pharmacological Treatment:
Fluid replacement, i.e., oral fluids 50ml/kg for first 4 hours, then maintainable ORS.
Check red flags for severe dehydration
Continue breast feed.
Give simple water between feeds.
Avoid fruit juices and carbonated drinks
SIGNIFICANT TOPICS WITH SAQs 249
143.
A 4 year old child presents to your clinic with a history of fall from the stairs.
A. List 3 indications for admitting such a child with a history of head injury?
B. What relevant advice would you give to the parents if such a child is sent home? List 4 points for
safety netting.
ANSWER
A
Indications for Admission
Refer to A&E for further assessment and admission if
• Glasgow Coma Scale <15 at any time since injury
• Loss of consciousness
• Focal neurological deficit since injury—problems speaking, understanding, reading, writing, d
sensation, loss of balance, weakness, visual changes, abnormal reflexes, problems walking,
irritability, or altered behaviour especially in young children
• Any suspicion of skull fracture; penetrating head injury; blood or CSF in the nose, ear, or wound;
serious scalp laceration or haematoma
• Amnesia for events before or after injury
• Persistent headache
• Vomiting
• Seizure
• Any previous cranial neurosurgical interventions
• High-energy head injury (e.g. pedestrian hit by motor vehicle, fall >1m or >5 stairs)
• History bleeding or clotting disorder or on anticoagulant therapy
• Difficulty in assessing the patient (e.g. very young or epileptic) or concern about diagnosis
• Suspicion of non-accidental injury
• Inadequate supervision at home .--- If Glasgow Coma Scale is <15, neck pain/tenderness, focal
neurological deficit, paraesthesiae in the extremities or any other clinical suspicion of cervical
spine injury, immobilize the neck before refering to A&E
B
Relevant Advice: If examination is normal
• Warn the parents that child may suffer mild headaches, tiredness, dizziness, tinnitus, poor
concentration, and poor memory for the next few days
• Advise rest and paracetamol (but not codeine-based analgesics) for the headache
• —sleepiness is common and not a worrying sign as long as the child is rousable
• Give written head injury information regarding warning signs to trigger reconsultation—
drowsiness, severe headache, persistent vomiting, visual disturbance, and/or unusual behaviour
SIGNIFICANT TOPICS WITH SAQs 250
144.
A 5 year old boy with a height of 28 inches with reddish brown hair, pot belly and swelling of feet has
been brought to you by his mother with history of chronic cough and occasional low-grade fever
ANSWER
A
Questions from Mother:
1) Ask about immunization history? (complete/defaulter)
2) Ask about feeding and nutritional history (weaning when started)
3) Ask about regular follow-up in well-baby clinic
4) Ask about any chronic disease or any congenital disease (high risk child – A or B or C)
5) Ask about developmental delays
6) H/O epilepsy or febrile fits (convulsions)
7) Socioeconomic history of parents
8) Family history (joint family system or separate, how many brothers and sisters or twin
deliveries)
9) H/O hair color change
10) H/O cough (dry or productive) + fever (low/high)
11) Weight loss history
B
Essential Clinical Examination:
1) GPE (to check hydration, oedema, anaemia).
2) Systemic examination (to check for hepatomegaly or muscle wasting)
3) Check extremities (oedema)
4) Check height, weight, head circumference (growth retardation)
5) Check for skin depigmentation, nail and hair changes and moon face (for autoimmune
disease)
C
Important Routine Investigations:
1) Full blood count (anaemia)
2) Blood glucose profile (hypoglycaemia)
3) Septic screening (infections)
4) Electrolytes, Ca, Phosphorus and ALP, serum proteins (hypocalcemia)
5) CXR and Mantoux (TB)
6) Stool & urine for parasites and germs
7) Exclude HIV and malabsorption
D
Essential Points in Management:
Detailed History
SIGNIFICANT TOPICS WITH SAQs 251
145.
A 6-year-old girl is brought to your clinic by her mother with complaint of stuffy and running nose for
several months along with excessive sneezing. She has suffered from mild eczema in the past. On
examination, you find bilateral clear nasal secretions along with pale mucosa and hypertrophied inferior
turbinate. In this case:
ANSWER
A
Provisional Diagnosis:
Rhinitis
B
Management Plan:
Clinical evaluation with complete history and clinical examination
• I will ask regarding potential allergens, pollens, animals
• Inquire regarding symptoms may be seasonal perennial or intermittent
• Assessment of severity of symptoms.
• Abnormal sleep?
• Impairment of daily activities
• I will do general physical examination and ENT examination
o I will see:
▪ Nasal crease
▪ deviation or perforation of nasal septum
▪ Retraction and abnormal flexibility of the tympanic membrane
▪ swelling of the palpebral conjunctivae with excess tear production.
o I will see for signs of complications like
▪ Acute or chronic sinusitis
▪ Otitis media
▪ Sleep disturbance or apnea
▪ Dental problems (overbite): Caused by excessive breathing through the
mouth
▪ History of atopy, asthma, Hay fever.
▪ Family history of Atopic dermatitis or similar complains.
Non-Pharmacological Treatment
General measures include decrease in allergen exposure
Nasal douching with saline nose drops
Steam inhalation
Pharmacological Treatment
Oral anti histamines
Nasal decongestant drops
Topical chromones, e.g. sodium cromoglycate
If chronic symptoms then intra nasal corticosteroid sprays can be advised.
Referral: If symptoms are intrusive and difficult to control, refer for specialized tests like
SIGNIFICANT TOPICS WITH SAQs 253
146.
An 8 month old child who has vomiting and diarrhea for two days is brought to your clinic. There is no
blood in the stools and the patient has not passed urine in the last 2 hours. This child, who weighed 8 kg
last week, now weighs 7.5 kg. In this case:
ANSWER
A
Signs of Dehydration
▪ Skin turgor/sunken eyes
▪ Depressed anerior fontanel
▪ Dry mucosal membranes
▪ No tears
▪ Low urine output
▪ Rashes
A
Management
Complete History:
➢ Bottle feeding + sterilization technique
➢ History of diarrhea – number of wet diapers + vomiting
➢ History of immunization
➢ History of similar symptoms of other family members.
➢ Food intake history + water resource.
➢ Socioeconomic status
➢ Any history of travel of parents
➢ Animal exposure- cats (Campylobacter), turtles (Salmonella)
➢ Any antibiotic history, malaria
Examination:
o Temperature,pulse,BP,General look n pallor.
o Chest:for R/R,auscultation of heart and lungs.
o Abdomen for distension,rigidity,palpable mass and bowl sounds.
o CNS examination.
Investigations: Generally not done. Following Investigations may be done if needed.
✓ Stool microscopy/culture
✓ serum electrolytes
✓ FBC,to rule out infection
Treatment: Assessment is done for state of dehydration
Non-Pharmacological:
o Resassurance and education of mother.
o For mild to moderate dehydration, oral rehydration is done with ORS. After every bowel
movement, give 8-10 spoons of ORS.
o Advise mother to continue breast feeding at increased frequency.
Pharmacological:
SIGNIFICANT TOPICS WITH SAQs 255
If there is some vomiting give oral antiemetic. If not responding, give intramuscular
antiemetic.
Oral antipyretic if there is fever.
Referral: Immediately refer to paediatriation if there is;
o high grade fever
o persistent vomiting
o severe dehydration not responding to oral rehydration.
SIGNIFICANT TOPICS WITH SAQs 256
147.
A formula fed, underweight 6 month old child with a history of recurrent acute respiratory infections
and diarrhea for the past 2 months, presents with normochromic anemia, pedal edema, hair loss and
scaling red lesions about the mouth and diaper areas.
A. What relevant questions should you ask in the history to complete your assessment?
B. Describe the dietary management for this child.
ANSWER
A
Relevant Questions in History:
➢ Ask about vaccination history (vaccinated/defaulter)
➢ Ask about feeding/nutritional history (breast milk or use of dilute animal milk or artificial
formula. If artificial formula, then ask about routine artificial or special formula)
➢ Developmental milestones.
➢ Any physical defect, e.g., cleft lip or cleft palate or cardiac abnormalities (high risk child)
➢ Ask about weight loss (marasmus)
➢ Ask about edema anywhere in the body (kwashiorkor)
➢ Socioeconomic history of parents.
➢ Family history (joint family system or separate. Mother working or not. How many
brothers and sisters. Twin pregnancy.
➢ Ask about cold and cough (recurrent)
➢ Ask about diarrhoea (watery or mucoid, relation of diarrhoea to food, color of stool)
➢ Ask about associated vomiting
B
Dietary Management:
Give high-quality protein and adequate carbohydrates in form of milk formula (first
skimmed milk then later whole milk). Give the infant breastfeeding if still on mother’s milk.
Start with liquid food, then semisolid, other chewable food as eggs, beans and meat
(according to age).
Feed the infant orally, either by breast, bottle or spoon. If it is impossible feed him through
nasogastric tube.
Give feeding slowly (as child is irritable).
Avoid interruption of feeding (such as labs or radiology)
Weight daily + avoid body temperature alterations.
Use hot water bottles
Feeding equipment must be sterile.
Other Advice:
➢ Place the infant in a warm, humidified incubator.
➢ Dress infant with extra clothes.
➢ Monitor temperature hourly in unstable infants.
➢ Observe for signs of collapse (temperature drop, coldness of extremities, slow pulse,
cyanosis, or grey white color of skin.
➢ Clean skin with clear water and apply oil
SIGNIFICANT TOPICS WITH SAQs 257
148.
A full term infant weighing 3.5 kg develops jaundice on the fourth day of life. He is being breast fed and
is feeding well. On examination, he is afebrile, not lethargic or irritable. The mother has blood group A
Rh+ve. The baby is also group A RH+ve. His hemoglobin is 16 gm/dl. Coomb’s test is negative. The
total serum bilirubin is 15 mg/dl with bilirubin at 14.15 mg/dl.
ANSWER
A
Provisional Diagnosis
Physiological Jaundice.
Why?
In newborn babies a degree of jaundice is normal. It is due to the breakdown of red blood cells
(which release bilirubin into the blood) and to the immaturity of the newborn's liver (which cannot
effectively metabolize the bilirubin and prepare it for excretion into the urine).
B
Management
Reassuring mother about the relatively benign nature of breast milk jaundice.
Additional Investigations: U/E, CBC, thyroid function.
Increase breastfeeding to 8-12 times per day.
Recheck the serum bilirubin level in 12-24 hours.
Continue breastfeeding and supplement with formula.
(This management is for healthy term infants with breast milk or breastfeeding jaundice
and with bilirubin levels of 12 mg/dL (170 µmol/L) to 17 mg/dL)
Further Relevant Points
For infants with serum bilirubin levels from 17-25 mg/dL (294-430 µmol/L), add
phototherapy.
Temporary interruption of breastfeeding if serum bilirubin levels reach 20 mg/dL (340
µmol/L).
It can persist for 3-12 weeks.
Sunlight therapy might be helpful.
SIGNIFICANT TOPICS WITH SAQs 259
149.
3 years old Saleem is brought to your clinic by his mother with history of 102 F temperature for last 3 to
4 days with watery eyes, runny nose and cough. Today mother has noticed some rash on the face and
neck. Saleem has not received any of the childhood vaccinations.
On examination, child has red and watery eyes with nasal congestion. Chest is clear. Rash is
maculopapular.
ANSWER
A
Most Likely Diagnosis:
Measles
B
Complications of Measles
▪ Acute otitis media (10% of cases)
▪ Pneumonia (5%)
▪ Diarrhea/vomiting
▪ Febrile seizures
▪ Sclerosing panencephalitis (5%)
C
Management
Detailed History: To assess severity level. Ask;
➢ Is child taking feeds
➢ Any h/o seizure (febrile)
➢ Any h/o shortness of breath
➢ Any ear infection
➢ Any h/o cough with high grade fever
➢ Any h/o drowsiness, convulsions.
➢ Any h/o contact with a person with similar condition.
➢ Any immunosuppresed contact at home.
➢ Immunization history
➢ Birth +/- developmental + nutritional history.
Examination:
• Vital Signs: Pulse, RR, temperature.
• GPE: Pallor, jaundice, assessment of hydration level to rule out dehydration. Is child active
or lethargic, sleepy or irritable.
• Mouth Exam: Koplick spots in oral cavity.
• Examination of rash – generalized maculopapular
• HEENT examination
• Chest: breath sounds, any signs of consolidation, chest indrawing
Laboratory Testing
✓ Measles IgM antibodies & IgG serology (ELISA) (serum or oral fluid samples) OR
SIGNIFICANT TOPICS WITH SAQs 260
150.
Briefly outline management plan for a male child less than five years of age with confirmed urinary tract
infection.
ANSWER
A
Management Plan:
As the diagnosis is confirmed so no need for any investigations. Let us directly come towards the
treatment plan
Non-Pharmacological Treatment:
Discuss the condition of the child with the mother, tell her the disease and risk factors and
home remedies + ICE of UTI.
Unsweetened cranberry or blueberry juice: It lessens the adherence of infective bacteria to
the bladder wall and thus allow them to be expelled in urine.
Pineapple: contains a chemicl that reportedly possesses antiinflammatory properties and
thus reduces UTI symptoms.
Yoghurt also decreases UTI symptoms.
Increase in overall fluid intake: It dilutes bacteria concentration and allows more effective
flushing out of HARMFUL BACTERIA
Hygiene: Gentle cleansing of genital parts with uncircumcised males, mild and gentle
traction of the foreskin helps to expose the urethral opening.
Complete Bladder Emptying: Encourage double voiding. Children should be encouraged to
urinate approximately every 2-3 hours.
Avoid the 4C:
o Carbonated drinks
o High amounts of citrus.
o Caffeine
o Chocolate
Avoid bubble baths
Prophylactic antibiotics: Daily low dose antibiotics under a doctor’s supervision may be
used in children with recurrent UTIs (Discuss with doctor when to start prophylaxis)
Pharmacological Treatment: Antibiotic therapy for UTI is based upon the sensitivity profile
obtaiend from urine culture results.
Cystitis: Infection limited to the bladder should respond quickly to routine ORAL
ANTIBIOTICS
Pyelonephritis: may need HOSPITALIZATION for IV administration of antibiotics along with
fluid therapy.
151.
A mother brings her one year old daughter to your clinic. History reveals frequent respiratory
and gastrointestinal infections. She has passed two semiformed stools in one day and has mild
cough. On examination, the child is sleeping in the mother’s lap. Her temperature is 37.7 C.
Heart rate is 90/min. Respiratory rate is 36/min. The anterior fontanelle is flat, oral mucosa is
wet. She looks pale. There is no chest indrawing and you can hear conducted sounds on
auscultation. Cardiovascular and abdominal examination is unremarkable. The child’s weight is
7 kg and she has not been immunized.
ANSWER
A
Assessment:
Cystic Fibrosis
B
Reason for Recurrent Infections:
Abnormalities in a membran chloride channel (CFTR) results in altered chloride transport and
water flux across apical surfaces of epithelial cells. Almost all exocrine glands produce abnormal
thick, viscous mucus obstructing glands and ducts and then getting prone for repeated infective
illnesses of respiratory and GI systems (pancreas, liver, intestine).
C
Management:
Detailed History and Examination:
Investigations:
✓ Ask about child’s diagnostic test results if performed in past, e.g.,
o Quantitative pilocarpine iontophoresis sweat test (> 60 mEq of chloride level and
increase sodium level too) – most common investigation
o Blood spot immunoreactive trypsin – most commonly advised in neonates.
o Nasal potential diference testing
o Others
▪ ABGs
▪ CXR – hyperinflation, peribronchial cuffing, ring shadows, parenchymal cysts
▪ Plain abdominal radiographs – showing faecal body or meconeum ileus signs
▪ USG abdomen
▪ Send immediate sputum/phlegm or nasal secretion for microscopy and C&S
Non-Pharmacological (Advices to Mother):
Exploring and addressing mother’s ideas, concerns and expectations.
Educating her about need of multidiscipilinary treatment approach to her child’s disease
(involving local cystic fibrosis organization, paediatrician and physiotherapist).
Screening of family members and pre-natal genetic counselling
Strict compliance to treatment plan designed by multidiscipilinary management team.
1200 to 1900 calaories per day to be given for prevention of weight loss.
SIGNIFICANT TOPICS WITH SAQs 264
Maintaining child’s good diet, e.g., cheese, full cream milk, poultry, dairy products, fish
(more calories, more fat diet) and good hydration.
Maintaining child’s good hygiene, hands, oral and general
Maintaining good hygiene of mother and care givers particularly hand hygiene.
Provision of safe, clean water and hygienically prepared food.
Regular vaccinations of the child (pneumococcal and yearly influenza vaccines)
Pharmacological:
Inhalation of hypertonic saline 7% stat.
Inhalation of mucolytics granules with neubulizer.
Salbutamol inhalation 200 mcg prn 4-6 hourly with spacer.
Fluticasone inhalation 250 mcg prn 12 hourly with spacer.
Liquid multivitamin supplementation (A,D,E,and K)
Pancreatic enzyme granules according to weight – to be given mixed in an acidic food, e.g.,
small quantity of apple sauce.
Referral: Refer the patient to multidiscipilinary specialized care are for general management plan
+ physiotherapy assisted home technique demos of chest postural drainage and percussion
maneuvers.
Follow-up: As designed by cystic fibrosis care centers (every 8 weeks until age of 5)
SIGNIFICANT TOPICS WITH SAQs 265
152.
Parents brought their 6 years old child with history of dog bite in the right leg while playing in the street.
Child is unimmunized. On examination, child weighs 20 kg and has a deep bleeding wound in the right
leg
ANSWER
Managment Plan for Dog Bites
Detailed History:
➢ History of incident – how and when it happened, simple incident or provoked the dog.
➢ History of dog
o Stray/street dog?
o Wild?
o Any information about dog’s behaviour?
o Any diseases (skin disease to dog)
➢ History of child
o History of immunization
o Any previous h/o dog/animal bite
o Any previous immunization against rabies/tetatnus
o History of allergy
o History of area of bite
o How much blood loss
o Any deformity (fracture?)
o Any immune disorder or on steroid/immunosuppressant regime.
Examination -
o GPE/Vital Signs:
o look at the child as if he is not looking well -site of wound
o Vital signs
o Signs of heavy blood loss
o Signs of struggle during attack
o Local Examination:
o Number of wounds (one or multiple)
o Site/location of wound
o Either clear or contaminated wound with dirt
o Extent of wound, depth of wound
o Still bleeding?
o Any bandage done or not yet?
Treatment:
Reassurance to parents.
Calm the patient
Explain the procedure and immunization and its importance.
Wash the wound with soap 2% benzalkonium chloride and water thoroughly so as no dirt or
debris remains.
Do not sture unless cosmetically essential and there is no deep damage.
SIGNIFICANT TOPICS WITH SAQs 266
153.
Parents brought their 6 years old child with history of dog bite in the right leg while playing the street.
Child is immunized. On examination child weighs 20 kg and has a deep bleeding wound in the right leg.
ANSWER
A
Managment
History: Ask;
➢ If the dog a pet or street dog
➢ Where is the dog?
o Healthy Dog (animal normal)
o Infected Dog (conformed rabied dog)
Rabies Protocol
Wound care: Wash wound with water and soap thoroughly or with normal saline +
dressing with water pressure, “do not stitch the wound”.
Gives rabies immunoglobulin
o According to the weight of the person
o Give half in wound, half in deltoid muscle.
o 20 units/kg. If patient 90 kg then 1800 units.
If the dog is healthy, not dead or ran away, or pet dog, confine the dog in a place to observe
for a least 10 days. If dog rabid, dies in 10 days.
Start rabies vaccine in all cases.
o In 10 days patient will receive 3 doses of vaccine.
o 1st day, 3rd day, 7th day (3 doses in 10 days)
o If animal (dog) normal – stop the vaccination by WHO
o If rabid dog – complete full vaccination protocol (days 0, 3, 7, 14, 28 – 5 doses)
o Dose of vaccine: 1.0 or 0.5 ml in deltoid muscle + immunoglobulin (RIG)
o (keep vaccine in fridge at 8 degrees – to be used within 4 hours out of fridge)
Give ATT IM stat (tetanus toxoid +/- tetanus immunglobulin
Antibiotics
o Aerobes of mouth - Gram +ve (Cloxacillin or Augmentin)
o Anaerobes (metronidazole)
Analgesics
Prescription:
o Tab. Augmentin 625 mg TDS – 7 days
o Tab. Metronidazole 400 mg TDS – 7 days
o Tab. Paracetamol 2 TDS (as needed)
Follow-up: Afer 2 days for 3rd day vaccination.
Safety Netting: Take care if behaviour change, water phobia, etc.
SIGNIFICANT TOPICS WITH SAQs 268
154.
A 62 years old man presented with history of developing jerking in his right arm and leg that lasted 3-5
minutes. He also lost consciousness for a brief period. He gives history of smoking more than 20
cigarettes daily for 40 years. He is normal on examination.
ANSWER
A
Provisional Diagnosis:
Complex partial seizures
B
Underlying Cause:
• Space occupying lesion secondary to CA lung
• Temporal lobe epilepsy
• Stroke /post infarct fit
• Metabolic encaph
C
Management Plan:
Detailed hx
➢ Duration of fit
➢ Any witnessed loss of consciousness, tongue bite, urinary or fecal incontinence during attack
➢ Pre/post ictal symptoms like headache, confusion, irritability.
➢ General symptoms: Any hx of fever, cough, chestpain, shortness of breath, headache, weight
loss, loss of appitite, bowel habits, depressed mood, lethargy.
➢ Medical hx: DM, HTN, previous history of seizures, previous surgery.
➢ Family hx: Hx of cancer, epilepsy, stroke, dyslipidaemia
➢ Social hx: Use of drugs/alcohol.
➢ As patient is a smoker, ask about smoking, for how long, how many packs per day, ever tried to
quit, etc.
➢ Ask about job,any stressor in job or at home.
After complete history and examination,before ordering blood workup, Immediate treatment
Check vitals, check pt is he seizure free,check airway,breathing and circulation, as hx suggestive of
short duration fitsb so will proceed with labs
Investigations
a. Blood tests: serum glucose with glucometer ,CBC,Serum electrolytes,Ca,Mg,urea ,creatinine
b. ECG: baseline
c. X-ray Chest: for any lung nodule
d. CT scan brain: to rule out stroke
e. MRI BRAIN,metastatic brain lesion can be seen as multiple hyperdense area
Non-Pharmacological Treatment:
Address ICE
SIGNIFICANT TOPICS WITH SAQs 269
Let pt about his condition and discuss with him treatment modalities,he may need to be refer
for further management and treatment option,
Written referral with hx and lab reports to neurophysican/neurosurgeon
a. Stop smoking
b. Taking well balanced diet
c. Reduce sodium intake
d. Walk as tolerated
e. Do not drink alcohol
f. Losing weight and bring body weight to normal.
g. Getting at least 7 hours good quality continuous sleep every day
h. Educate pt and care giver about prevention and management of seizure(will be discuss further
in saq)
SIGNIFICANT TOPICS WITH SAQs 270
155.
A 70 year old male is brought to your clinic by his daughter, as he collapsed in the bathroom in the
morning. He remembers feeling faint on getting up from the commode, and then he collapsed/blacked
out. His daughter heard the bang and rushed in. He is feeling fine, except for a bruising on his right
forearm.
ANSWER
A
Important Questions in History:
➢ Hx of co morbidities like HTN, diabetes, CKD.
➢ hx of intake of any sedatives
➢ hx of such episodes before
➢ hx of any family members who had such attacks
➢ Hx of vomiting diarrhea
➢ hx of any addiction
B
Important Aspects of Examination:
• VITALS including BSR,standing and lying BP
• Cardiac murmurs
• Carotid bruit
• Swelling of lower extremeties
• Tongue bite
• Head exam for any laceration
• Neurological Exam
C
Important Aspects in Management
After complete hx and examination,
Investigations: CBC, BSR, TFTs, urea, electrolytes, ECG, EEG, echo, carotid Doppler, chest x-
ray, DEXA scan.
Treatment: Maintain IV line, observe for any further attack, painkiller for any pains. If
cerebrovascular event after PT INR start anticoagulation, serial ECGs if suspected
arrhythmia esp., AF leading to TIA, amiodarone or digoxin
SIGNIFICANT TOPICS WITH SAQs 271
156.
A 32 years old Mrs. Ahmed presents to your clinic with right-sided headache for the last 2-3 years. Now
these episodes are occurring 1 to 2 times per week and she is unable to do her daily activities.
ANSWER
A
History Questions
• Characteristic of headache
o location
o duration
o quality (pulsatile/throbbing/aching)
o severity (disabling/distressing) especially worsening from previous headaches
o radiation
o aggravating and relieving factors (physical exertion/ specific food, )
• assoc symptoms (nausea/vomiting, photophobia or phonophobia, autonomic or sensory
aura, dizziness, eye pain, vision impairment, seizures, body weakness, facial pain, any
fever, neck symptoms)
• hx of head trauma
• Drug hx
o hx of freq use of abortive therapy or OTC painkiller meds
o contraceptive use
• known comorbids
o relevant family hx
B
Examination
• General , vitals, BMI
• HEENT
o facial asymmetry
o Palpation of head and neck for tenderness
o Ear exam
o Dental
• Neck exam
o range of neck movements
o vascular
• Focused neurological examination
o gait and imbalance
o Fundoscopy
o motor and sensory
o reflexes
C
Management
SIGNIFICANT TOPICS WITH SAQs 272
157.
A 65 year old man presents to your clinic after suffering a seizure. An hour back he developed jerking in
his right arm and leg, which lasted 5 minutes. He also lost consciousness for a brief period. He has
smoked > 40 cigarettes daily for over 30 years. He is normal on examination.
In this case:
A. What is your provisional diagnosis?
B. What is the most likely underlying cause for the seizure?
C. How will you investigate and manage?
ANSWER
A
Provisional Diagnosis:
Complex Partial Seizure
B
Most Likely Underlying Cause:
Previously solved
C
Investigations
• FBC
• C-rective protein
• Renal function
• Electrolytes
• Serum calcium and magnesium to rule out metabolic cause.
• RBS
• CXR
• ECG
• CSF examination.
• Serology for syphillis to rule out inflammatory or infective cause.
• Brain scan
o MRI is more sensitive than CT in detecting relevant anatomical abnormalities_
• EEG
o PS: Routine scalp EEG is not sensitive or specific for the diagnosis in *elderly*
people, nor does the absence of epileptiform waves rule out epilepsy
Management
• detailed hx
- previous hx of seizures /treatment
- hx of CVA/stroke/ CNS infection
- drugs
- other comorbids
- trauma
• physical exam
- general
- neurological
- cvs
SIGNIFICANT TOPICS WITH SAQs 274
• Treatment
- Nonpharmacological*
▪ Reassurance
▪ extensive health education
▪ discuss and implement smoking cessation plan with the patient
▪ discuss and involve patient in the management of his disease, esp inform
benefits and risks of starting Anti-Epileptic Drugs
- Pharmacological
▪ Antiepileptics, Consider;
• CARBAMAZEPINE gold standard but in elderly can cause bone loss and
parkinsonism
• LAMOTRIGINE
• GABAPENTIN
• Referral to epilepsy clinic, and smoking cessation program.
SIGNIFICANT TOPICS WITH SAQs 275
158.
A 65 year old retired banker comes to your clinic with history of weakness of his left side of the body
since 5 days. He had noticed that he was unable to use his left arm, and his left leg was weak. This
weakness resolved completely the next day. His weight is 72 kg, height is 162 cm and blood pressure is
145/92 mmHg.
A. What is the exact term given to this type of acute stroke and justify why you would like to consider
that?
B. List 4 modifiable risk factors for cerebrovascular events that you would like to assess in this case in
detail history and physical examination.
C. What investigation you would order at this point? List any 3.
D. Outline 3 non pharmacological measures and 3 pharmacological measures for this patient. Give
reason for choosing those pharmacological measures.
ANSWER
A
Transient Ischemic Attack(TIA).
Justification: It is TIA because recovery takes place within 24hrs (next day)
B
Modifiable Risk Factors:
1.Hypertension:History of hypertension in past and any medication for treatment as B.P is raised
on examination(145/92mmhg)
2.DM
3.Obesity:by calculating BMI
4:Smoking
5.Alcohol
C
Investigations:
-CBC,ESR
-U&E
-Blood glucose level
-Lipid profile
-Se Cr.
ECG&CXR
-Clotting screen if FH of thrombosis.
D
In addition to reassurance, providing information/support to patient I 'll advice him for
1. Weight reduction
2. Stop smoking(if smoker)
3. Salt restriction
4. Regular exercise
5. Calculate ABCD2 score for future risk of stroke.
PHARMACOLOGICAL TREATMENT:
1. Start Aspirin:(antiplatlet) 75 mg od.to prevent further attack
2. Antihypertensive to control BP because patient’s BP is high.
SIGNIFICANT TOPICS WITH SAQs 276
3. Start Statin (atorvastatin 20 mg OD) and Dipyridamole S/R 200mg bd. for primary prevention of
stroke as patient is medium risk of stroke according to ABCD2 criteria.
SIGNIFICANT TOPICS WITH SAQs 277
159.
A 45 year old gentleman presents with a history of headache since 3 weeks. His past history reveals a
migraine headache since 20 years and a fall 3 weeks ago.
ANSWER
A
Further Questions in History:
➢ h/o trauma or any head injury (SAH)
➢ h/o associated fever (meningitis/encephalitis)
➢ Predisposing or relieving factors
➢ Family history of headache (migraine)
➢ Drug history (medication-induced headache)
➢ Chronic disease history (HTN)
➢ h/o pain or fever + cold and cough (sinusitis)
➢ Anxieties + h/o type of job (tension headache)
➢ h/o HIV + cancer (autoimmune diseases)
B
Likely Differentials
❖ Subdural haematoma
❖ Space Occupying Lesion
❖ Head injury
❖ Meningitis/encephalitis
C
Management
Detailed History: As above.
Examination:
o Vitals (especially fever)
o GPE + Systemic Examination (rashes, stiff neck, photophobia), check level of consciousness
Investigations:
✓ To rule out infection:
o CBC
o CRP
o ESR
✓ CT scan head – (SAH)
Non-Pharmacological:
Address ICE about disease
Avoid triggers of migraine
Pharmacological:
Immediate admission
IV or IM penicillin V
Tab. Paracetamol/NSAIDs TDS
SIGNIFICANT TOPICS WITH SAQs 278
Neurosurgical Opinion:
Postoperative nimodipine (e.g., 60 mg/4 hourly if BP allows) has been shown to reduce the
severity of neurosurgical defecitis but does not reduce the severity of neurosurgical deficits
but does not redcuce rebleeding (if SAH)
SIGNIFICANT TOPICS WITH SAQs 279
160.
A 75 year old patient presents with a case of sudden onset of acute confusion and generalized
weakness. There is no history of fever, no shortness of breath and no history of fall associated with it.
A. What are the possible causes of his complaint in this case? (name any 5)
B. What are the possible investigations you would like to offer in this case? (Name any 5).
ANSWER
A
electrolyte imbalance
Stroke
Drug intoxication
Hypothyroidism
Infections like UTI/pneumonia/biliary disorder
Elderly present differently not like the young adults
A. Drug intoxication
Electrolyte imbalance
Hypoglycemia
Stroke
Hypothyroidism
Systemic infections
(Uti . biliary disorder. Pneumonia)
B
# CBC
# BSR
# s/electrolyte
#s/TSH
# urine RE
# CXR
CT BRAIN noncontrast
SIGNIFICANT TOPICS WITH SAQs 280
161.
A 30 year old patient has frequent bouts of headache with a feeling of tight band around his head. He
has lot of stress in work and the headaches started 2 years ago with the start of his present job.
Write in no more than 10 lines, how will you manage such a patient?
ANSWER
A
I 'll take detailed history:ask about
-nature and severity of pain,bilateral or unilateral, duration and frequency, does it affects daily
activities?
-h/o vomiting,visual changes like photophobia, lacrimatiin.does it awakens pt from sleep,aura(
flashing lights,numbness)
,-ask about psychological problem, relationship with boss colleagues,work load.
-medicine use: nifedipine, cimetidine.
-drugs used for headache,h/o head injury.
Examination:
- BP,neurological examination including fundi, visual acuity, palpation of temporal region and
sinuses, examin neck
Investigation, no need in this pt
Managment: it is most likely a case of tension headache so managment includes:
non pharmacological:
Reassure that there is no underlying pathology
HEADACHE DIARY
Try measures to alleviate stress-like relaxation, massage, yoga, excercise, physiotherapy, advice to
consider job change
if possible
Drug therapy:
Acute: simple analgesia like paracetamol, inuprofen
Prophylaxis: acupuncture- up to 10 sessions over 5-8 wk
SIGNIFICANT TOPICS WITH SAQs 281
162.
A 26 years old lady presents with sudden onset of right-sided “Bell’s palsy” for 1 day.
ANSWER
A
Examination of Concerned Nerve (Cranial Nerve VII)
o Watch for ipsilateral facial muscle weakness and droop.
o Forehead:
o Ask the patient to raise the eye brows – flattening of forehead skin crease on
affected side.
o Ears:
o Examine ipsilateral external auditory canal for vesicles, erythema, infections and
signs of trauma.
o Check for reduced pinprick over ipsilateral posterior auricular area.
o Check for hyperacusis (with help of stethoscope/tuning fork) – loud sound will be
lateralized to paralyzed stapedius muscle side.
o Eyes:
o Look for patient’s inability to close affected side’s eye, reduced tear film leading to
dryness of ipsilateral eye.
o Ask for attempted/forced eye closure – look for affected eye getting rolled upwards
and inwards.
o Loss of corneal reflex on ipsilateral side, sparing the contralateral side may also be
noted.
o Nose with perioral muscles:
o Loss of nasolabial fold due to facial muscle atony
o Inability to obey commands like;
▪ Show your teeth
▪ Puff out cheeks
o Oral Cavity: Reduced taste sensation with decreased salivation
B
Management:
Detailed history with Examination
Investigations: Mostly not required, diagnosed clinicallyWatch for ipsilateral facial muscle
weakness and droop.
Non-Pharmacological:
Exploring and addressing patient’s ICE
Patient reassurance with health education.
Importance of adhering to treatment though many cases resolve spontaneously yet
complications and residual symptoms can be avoided with early diagnosis and prompt
treatment.
SIGNIFICANT TOPICS WITH SAQs 282
Rule out conditions (before initiation of treatment) such as pregnancy, active infections and
immunosuppressed states.
Manage with facial exercises during recovery shortnes the phase
Eye care is a must.
Avoiding myths like blood application over affected area.
Majority of patients recover complete4ly, but require 3 weeks’ duration on average.
Pharmacological:
Tablet Prenisolone 25 mg PO BD for initial 5 days.
Taper off over another 5 days (total 10 days treatment plan).
Use of artificial tears during working hours.
Eye lubricants during sleeping hours.
For protecting eye cornea, lip tapes and pads in night and glasses in the day.
Follow-up & Referral:
Follow-up in 5 days.
Referral to specialist care if no improvement seen.
SIGNIFICANT TOPICS WITH SAQs 283
163.
A 35 years old lady presents to your clinic complaining of headaches with increasing frequency. They
now occur two to three times a week. The headaches are throbbing in nature and usually limited to the
right side of her head and have been present for 3 years. On inquiry, they last for 4 to 6 hours and are
relieved by strong painkillers and sleep. She says because they are so frequent, she is not able to cope
with her family and home responsibilities. Her husband lost his job 3 months ago and has been unable
to get another.
ANSWER
A
Most Likely Diagnoses:
❖ Migraines
❖ TENSION headache
❖ Medication induced
❖ Cluster headache
B
History Questions: Ask for;
➢ Onset, course, duration
➢ Pain location, quality, radiation, severity (from 0 to 10 scale)
➢ Triggers: lack of sleep, over sleep, chocolate, cheeze, hunger, caffeine, stress, heat.
➢ Aggravating Factors: Pain increased by activity, coughing, sneezing.
➢ Relieving Factors: medication + sleep.
➢ Associated symptoms: Photophobia, phonophobia, nausea, vomiting, loss of vision
➢ h/o similar pain. If yes, frequency
➢ Any focal neurological deficit
➢ h/o fall, relationship to periods.
➢ Past medical history: HTN, DM
➢ Family History: Migraines
➢ Personal History: h/o smoking, h/o alcohol use.
➢ Past Medications: Analgesic, NSAID, OCP use
Physical Examination:
✓ GCS
✓ Vital signs + GPE – check for hypertension and tachycardia
✓ BMI
✓ HEENT examination
o Cranial/cervical muscle tenderness ipsilateral Horner syndrome (relative meiosis, 1-2
mm of ptosis)
o Conjunctivae
o Adie-type pupil (poor light reactivity)
✓ Detailed Neurological Examination:
o Gait and balance issues
SIGNIFICANT TOPICS WITH SAQs 284
164.
A 16 years old boy has headache for the last two years.
ANSWER
A
Diagnosis:
1) Tension type headache
a. based around the head
b. stress
c. low mood
d. relieve by NSAID
2) Cervicogenic headache
a. Unilateral or bilateral
b. Band from neck to forehead
c. Scalp tenderness
3) Migraine
a. Aura
b. Nausea, vomiting
c. Trigger
d. One or both sides of head
e. throbbing sensation
f. Visual disturbances
4) Medication overuse
a. Rebound headache on stopping medication
5) Increased intracranial pressure – worse on awakening, sneezing, bradycardia, hypertension,
neurological signs.
B
Clinical Examination:
o General appearance of patient, comfortable, well looking or depressed, anxious +
stressedDuring the physical exam
o Vital Signs: If raised temperature, look for a rash.
o Brief Neurological Examination:
o Fundi
o Visual acuity
o Gait
o Head movements.
o Sinuses for tenderness
o Examination of neck
o Brief ENT examination
C
Investigations:
SIGNIFICANT TOPICS WITH SAQs 286
165.
A seventy year old man presented with the history of an attack, which started with twitching of
the left thumb and spreading to involve the whole hand, then the left arm, left foot and leg. He
became unconscious with foamings at the mouth.
He awoke 10 minutes later, but could not remember the attack.
He had suffered no previous attacks, was taking no drugs and was in good general health. There
was no history of any injury.
A. What was the attack described above. Suggest the most likely cause and two other
possible causes.
B. What further investigations will you carry out giving justifications?
C. Describe the initial management of this patient with justification of your steps.
ANSWER
A
This was an attack of complex partial seizure .
Most likely cause
1) SOL
2) Temporal Lobe Epilepsy
3) Stroke/post infarct fit
1) Metabolic encephalopathy
B
Investigation with Justification:
✓ Blood workup
o CBC (baseline) to rule out any infectious cause.
o Metabolic Panel (to rule out metabolic cause)
o Renal Function Tests
o Serum Ca and Mg
✓ Radiology
o MRI (for space occupying lesion)
o CT (for any intracerebral bleed)
✓ Other:
o EEG (to rule out any epilepsy)
o ECG (baseline + arrhythmia)CT /MRI ( any structural lesion.)
Urea ,Electrolyte,LFt,blood glucose,serum calcium and magnesium.(To rule out metabolic cause)
C
Management: As patient is eventually fit free, I will proceed with;
Detailed History:
➢ History of presenting complaint
o Did any one observe fits or patient was alone (witness)
o Did start suddently or gradually increased
o Any feeling of lightheadedness, headache or abnormal sensation before fits.
o Duration of fits
o Did lose consciousness (as mentioned in history)
o Any tongue bite
SIGNIFICANT TOPICS WITH SAQs 288
▪ Tinnitus
▪ Tremor
▪ Weight gain
SIGNIFICANT TOPICS WITH SAQs 290
166.
60 years old housewife presents with a six weeks history of insomnia, crying spells and low mood. She
has a son and a daughter both of whom are abroad. She lives with her retired husband. She is normal
on physical examination.
ANSWER
A
Most Likely Diagnosis:
Depression/MDD (major depressive disorder)
B
Further Points on History: Ask for;
➢ Symptoms of general low mood
o Low self esteem
o Guilt
o Pessimism
o Helplessness/hopelessness, loneliness
o Apathy, lack of interest in social life
o Loss of interest in usual routine
o Loss of concentration
o Paranoia, anhedonia
➢ Somatic features:
o Loss of appetite/increased appetite
o Loss of weight, loss of energy
o Easy fatigability
o Constipation
o Insomnia/hypersmonia
o Amenorrhoea
o Low libido
o Psychomotor agitation
▪ Derogatory auditory hallucinations
▪ Delusions of worthlessness
➢ Anxiety symptoms:
o Tension
o Apprehension
o Suicidal thoughts
o Homicidal thoughts.
➢ Personal preferences (previous, current, or both)
o Smoking
o Alcoholism
o Use of illicit drugs
o Use of corticosteroids
SIGNIFICANT TOPICS WITH SAQs 291
167.
Mrs. Atiya, a 58 years old lady presents with a history of crying spells for 6 weeks. She also complains of
excessive tiredness, with loss of interest in day to day pleasure of life. She is a known hypertensive and
her husband gets upset about her loss of interest in sexual activity. She has two adult sons living in USA.
ANSWER
A
Further Points in History: For making diagnosis, the patient should be asked about these
symptoms.
✓ Sleep (hypersomnia – insomnia)
✓ Interest – (lack of interest in life in general)
✓ Guilt or hopelessness
✓ Energy/fatigue
✓ Mood alterations/depressed, sad
✓ Concentration – lack of concentration or ability to focus
✓ Appetite increased or decreased with relative weight loss or weight gain.
✓ Psychomotor (retardation, agitation)
✓ Suicidal ideation
✓ Social or occupational impairment
✓ Any chronic disease
✓ Any medication related to depressive mood disorder or previous history of drug abuse
✓ Any delusions or hallucinations.
✓ Family and environmental factors ocial support
✓ familial and envoirmental factor.
B
Social Issues Raised
i) The behaviour and mood of depressed person affect the whole family,the irritablity which sets
of conflicts and derails family dynamics this will put poor huge impact on family.
ii) With this depressive disorder ,people usually avoid those persons because of behaviour and
lack of intimate relationship ,hence poor socialisation so at great risk of depresion to arise and
be prepatuated.
C
Management:
CBT. In case of severe symptoms regularly.
Patient needs to know that these spells are due to illness and discuss her condition in light
ways.
Stop alcohol or drug abuse.
Lifestyle management with regular walk and exercise.
Safe and secure neighbourhood
Meeting old friends.
Regular follow-up for pyschotherapy
SIGNIFICANT TOPICS WITH SAQs 293
Start Medication:
o SSRI – but as decreased sexual activity, think TCAs.
o MAOIs avoided as patient is hypertensive.
SIGNIFICANT TOPICS WITH SAQs 294
168.
A 65 years old man retired bank officer, presents with 8 weeks history of insomnia, low mood, crying
spells and irritability. He reports lack of pleasure in life, and says life is not worth living. He is normal on
examination, except that he cried during the interview.
ANSWER
A
Provisional Diagnosis:
Depression
B
Evaluation Prior to Discharge Home:
I will exclude following points before sending patient home.
1) Related to Diagnosis:
a. I will ask detailed history regarding symptoms
i. Does mood swing or it remains same (low mood)
ii. Any feeling of guilt
iii. How is his concentration level
iv. What he feels about life, is it worthless to him?
v. Any suicidal ideation
vi. How is energy level
vii. How is appetite
viii. Interest in intimacy with opposite sex
b. Also will evaluate more about family history – any suicide in family
c. Drug History – any drug abuse
d. Personal history – social support
e. Previous hisory of similar symptoms
f. Any previous history of antipsychotic medications
g. Most importantly will ask about suicidal ideation – if positive, patient needs to be
admitted.
C
Treatment Plan:
History: As discussed above plus also need to exclude differential with history.
➢ Any recent stress in life
➢ Recent weight loss/gain
➢ Sleep hygiene/sleep pattern, early awakening
➢ Any skin change/change in bowel habits (to rule out hypo)
Investigations:
✓ CBC
✓ Vitamin B12
✓ TSH
Non-Pharmacological Treatment:
SIGNIFICANT TOPICS WITH SAQs 295
Educate patient about condition and explain to him that it is a treatable problem
Discuss with patient his ideas, concerns and expectations.
If patient is in poor health and increased suicidal risk, then patient needs to be referred to
psychiatrist.
If patient is low risk, then will manage it in clinic
Arrange frequent visits.
Basic psychological treatment such as advice on lifestyle changes, problem solving, guided
to self help
Structured supervised exercise and supportive counselling.
Offer;
o Cognitive behavioural therapy
o Advise medication adherence and compliance
o Explain side effects of drug treatment.
Pharmacological Treatment:
Tab. Fluoxetine 20 mg OD PO. I will explain to patient that he may take 1 to 2 weeks to see
if any improvement in symptoms.
If symptoms worsen or develops suicidal ideation, he can visit emergency department
Follow-Up: In two weeks.
SIGNIFICANT TOPICS WITH SAQs 296
169.
A 33 year old female, mother of 2 children complained of low mood and crying spells. She has lack of
interests in her activities. She has been having these symptoms for the past 2 months.
A. Enlist four relevant questions you will ask from this patient.
B. What relevant investigation will you perform in this patient?
C. Enlist four steps of your management plan.
ANSWER
A
Four Relevant Questions:
➢ During the last month , have u often been bothered by feeling down , depressed , and
hopeless
➢ During the last month , have u often been bothered by having little interest or pleasure in
doing things
➢ Have u noticed any change in ur appetite n weight n sleep
➢ Do u think that u r worthless n feeling guilt ?do u have recurrent thoughts of death/ suicide
B
Relevant Investigations: Mostly the diagnosis is by history n clinically but we have to do labs to
rule out other DDs
✓ CBC
✓ ferritin ( anemia)
✓ TFT ( hypothyroidism)
✓ Beta HCG ( pregnancy )
C
Management Plan: If we rule out other causes and it's confirmed that patient is having depression
then
History: Given above.
Detailed General Physical Examination
o Assess the patient how she looks/rule out self neglect , any smell of alcohol , is she making
proper eye contact, looks depressed or tired , tearful ,anxious , poor concentration , any
hallucination
Non-Pharmacological
Discuss sleep hygiene, establish regular sleep and awake times
Avoid excess eating
Avoid smoking and alcohol before sleep
Take regular excercise.
Drink warm glass of milk before going to bed
CBT: If patient is having mild to moderate symptoms offer her CBT. This includes sessions
with an expert who has the ability to take out negative thinking from patient's mind n insert
positive thinking in patient mind , by using art of communication/ talking
Pharmacological: If patient's symptoms are moderate/severe or CBT doesn't work, then;
Patient can use SSRI, e.g., citalopram 20-40 mg once per day.
Follow-up:
SIGNIFICANT TOPICS WITH SAQs 297
Patient is advised to have a regular follow-up in first month, especially the young ones
should have follow up in first week as it increases the anxiety and suicidal thinking in first
two weeks.
It also increases GI bleeding risks so always prescribe PPI like omeprazole 20-40 mg daily, to
elderly or if any risk of GI bleeding
Safety Netting:
Give advice and safety netting to patient n family / carers about what to do if patient's
mood deteriorates,or any symptoms of self harm / suicidal thinking or any psychotic
symptoms, and to seek psychiatrist help as soon as possible in this case.
SIGNIFICANT TOPICS WITH SAQs 298
170.
Your next patient is a 34 year old sewing machinist at a garment factory. She bursts into tears as soon
as she sits down and says that she cannot cope with life any longer.
ANSWER
A
Positive Points in History:
I will screen through patient health questionnaire (PHQ-9)
➢ During the last month, have you been bothered by feeling down, depressed or hopeless?
➢ Have little interest or pleasure in doing things.
➢ Trouble falling asleep or sleeping too much
➢ Feeling tired/little energy
➢ Low appetite or over-eating.
➢ Trouble concentrating?
➢ Being fidgety or restless
➢ Any suicidal thoughts
➢ Any past history of pyschiatric disorders, anxiety, alcohol abuse and substance abuse,
eating disorder
➢ Physical disorder: MS, dementia, thyroid disorder, chronic infection, CVD disease.
➢ Drugs: Beta-blockers, calcium channel blockers, OCPs
➢ Biopsychosocial Assessment (BPD):
o Discuss symptoms (nature, onset, duration, severity)
o Past history of depression/mood elevation (bipolar)
o Family history of mental illness
o Quality of relationships
o Living condition
o Social support
o Employment
o Past response to treatment.
C
Management Plan: Discuss patient’s ideas, concerns, expectations.
History: As discussed above.
Examination:
o General Appearance: Self neglect, weight.
o Assessment of Mood: Looks depressed/fixed speed montonous, avoids eye contact,
tearful, anxious, or jumpy, poor concentration.
o Assess severity of depression according to PHQ-9 subthreshold < 5, mild 5-9, moderate 10-
14, moderately severe 15-19, severe ≥ 20 +/- psychotic symptoms markedly interfering with
function.
Investigations:
✓ CBC
✓ Vitamin B12
SIGNIFICANT TOPICS WITH SAQs 299
✓ TSH
Management: I will use a stepped care approach
Step 1: Provide education about depression, support available, discuss treatment options.
o If sleep is a problem:
▪ Discuss sleep hygiene
▪ Do regular exercise.
▪ Avoid excess eating or smoking before sleep.
o Subthreshold or Mild Depression
▪ Active monitoring – watchful waiting for < 2 weeks.
▪ Problem solving: Identify main problem and generate solutions then try out
solutions, feedback and modify
Step 2:
o If persistent subthreshold or mild/moderate depression:
▪ Low-intensity psychosocial interventions.
▪ Individual guided self help
▪ Computerized CBT
▪ Group exercise program
▪ Group based peer support
▪ Drug treatment.
Step 3: If not responding to treatment or moderate/severe depression:
o High-intensity pyschological interventions
o Interpersonal therapy
o Behavioral couple therapy (if relationship difficulties)
o Drug treatment – Fluoxetine 20 mg OD
o Combined treatment: Antidepressant medication + CBT or interpersonal therapy.
Step 4: Refer to psychiatry if;
o Highly suicidal
o Severe self neglect
o Psychotic symptoms or inadequate response to multiple treatments.
Follow-up: In two weeks.
Safety Netting: Clear advice to patient and family on what to do if mood deteriorates, how to
acces urgent support.
SIGNIFICANT TOPICS WITH SAQs 300
171.
A 26 year old banker presents with episodes of sudden onset palpitations, SOB, sweating which usually
occur when patient is under stress. He has visited the emergency room multiple time and all
investigations including cardiac testing have been negative. These attacks are now getting more
frequent and affecting his day to day functioning making him anxious all the time.
ANSWER
A
Likely Diagnosis:
Anxiety disorder with acute panic attacks
B
SSRI is the drug of choice. Priscribe citalopram 20-40 mg per day , follow up in less than 2 weeks
than regular follow up every month. Discuss n explain side effects of SSRI ( already discussed
above )
If patient can't tolerate SSRI than offer TCA to the patient ,,, tell the patient these medicines will
take few weeks to start their action
Do not give benzodiazepines to the patient from the very start because of poor long term
outcome
C
1, CBT ( discussed above )
2, relaxation therapy .. ask patient to sit in a dark room, ask the patient to close the eyes , n focus
on different parts on the body ..starting from the forehead and slowly going down to the toes ..
relax the muscles as much as u can. Breathe in n out slowly n deeply.
3, self help and group support ..( ask the patient to attend the group meetings with other patients
suffering from panic attack , it will help them to discuss their problems n will help them to
overcome )
SIGNIFICANT TOPICS WITH SAQs 301
172.
A 46 year old married female from Quetta was brought by her son in your clinic with symptoms of
irritability, insomnia, nightmares and flashbacks. These symptoms started four months back after the
bomb blast. She is also complaining of low mood and crying spells.
ANSWER
A
Four points in history :
- Frequently having upsetting thoughts or memories about a traumatic event
- Having recurrent nightmares
- Having strong feelings of distress when reminded of the traumatic event
- Actively avoiding people, places, or situations that remind you of the traumatic event
- Having a difficult time falling or staying asleep
- Having a difficult time remembering important parts of the traumatic event
B
Diagnosis
Post traumatic stress disorder
C
Treatment
- Non pharmacological :
- Reassurance the patient
- Explore ideas & concerns
- Cognitive behavior therapy
- Eye movement desensitization and reprocessing
- Interpersonal psychotherapy
- Pharmacological
- Antidepressants : SSRI& SNIR may have some benefit
SIGNIFICANT TOPICS WITH SAQs 302
173.
A 19 year old girl has been rushed into clinic by her family members saying that she became unconscious
suddenly when she was scolded by her father. She did not have incontinence, jerky movements or
physical injuries. Her past medical history is unremarkable.
Her BP is 120/70 mmHg, pulse is 90 BPM and respiratory rate is 20/minute. Neck is soft, pupils are
reactive. She forcefully closes her eyes and mouth when someone tries to open them.
ANSWER
A
Most Likely Diagnosis:
Pure malingering
B
Differential Diagnosis:
❖ Factitious disorder ( should be without purpose but this pt did it as a result of scolding)
❖ Somatization disorder: chronic condition with h/o numerous physical complaints with no
explanation
C
Careful history:
Wat happened, when where, precipitating event?
Does the pt remember whole episode, any jerking of limbs, fall n associated trauma at that time?
Prodromal symptoms like nausea, sweating, blurring, light headedness, dizzines, tinnitis. Anything
else during the attack like tongue bitting, urine incontinance,
Any similar event in past, h/o depression, suicidal ideation, self harm, drug abuse, domestic
violence, parental conflicts, sexuall harassment, academic pressure, possible purpose , f/h of
similar episodes or depression, developmental, medication, diabeties
Residual confusion , amnesia, drowsiness after attack
Examination: general physical, sensory / motor deficit, weakness, muscle tone, up rolling of eyes,
cynosis, abnormal pigmentation on skin, signs of self injury like cuts, signs of violance like finger
marks on body, lash, cigarette burn marks, cardiovascular examination to rule out bradicardia,
arythmia, , carotic bruit, htn or hypotension, murmer,
Investigation: according to history and examination , no need in this case.
Advice: reassurance, explain the condition that she is stable, she is not unconcious all findings r
normal n the likely cause of this event was the scolding from her father and she needs attention
and affection from her parents
D
I’ll avoid any placebo or medication or referal bc it will perpetuate the sumptoms
SIGNIFICANT TOPICS WITH SAQs 303
174.
Mrs. Hakim, a well educated lady of 50 years attends your clinic with symptoms of early morning
awakening, tiredness, and lack of interest in day to day pleasures of life. She has one child, a daughter
of 13 years age. Mrs. Hakim believes that her husband is having an affair with his secretary.
ANSWER
A
Further Points in History:
Take detailed history, especially;
• past history of depression + drug history
• h/o alcohol use or withdrawal
• Caffeine abuse
• Thyrotoxicosis
• Hypoglycaemia
• Temporal lobe epilepsy
• Phaeochromocytoma
• Do screening for depression and anxiety; two questions;
o During the last month, have you often been bothered by feeling down, depressed or
hopeless.
o During the last month, have you often been bothered by having little interest or
pleasure in doing things.
• Assessment: By Hospital Anxiety and Depression (HAD) scale and Patient Health
Questionnaire (PHQ-9)
B
Management: NICE Guidelines suggest a step-wise management
• Step 1: Education about GAD + active monitoring
• Step 2: Low intensity psychological interventions (individual non-facilitated self help or
individual guided self help or psychoeducational groups)
• Step 3: High intensity psychological interventions (cognitive behavioral therapy or applied
relaxation) or drug treatment.
o Do not use routinely drug treatment, but if needed use SSRIs (e.g., sertraline 50-150
mg OD)
o Follow-up 2-4 weeks
o Avoid benzodiazepines except for acute crises.
• Step 4: Highly specialist input, e.g., mutilagency teams.
Safety Netting:
1) Follow up if patients do not attend appointments. Give patient +/- familye carers clear
advice on what to do if the patient’s mood deteriorates.
2) How to assess urgent support both in and out of hours.
SIGNIFICANT TOPICS WITH SAQs 304
175.
You have received the histopathology report of a breast lump of your patient. Her son approaches you
and asks you not to tell the diagnosis to the patient, in case it is cancer.
ANSWER
1: Assuming the mother has decision-making capacity, it is important for me who the patient is:
the mother, not the son.
2: I will invite the son to come in for a discussion with me and assure him for fulfilling all ethical
criteria of breaking bad news (SPIKES).
STEP 1: S—SETTING UP the Interview
STEP 2: P—Assessing the Patient's PERCEPTION
STEP 3: I—Obtaining the Patient's INVITATION
STEP 4: K—Giving KNOWLEDGE and Information to the Patient
STEP 5: E—Addressing the Patient's EMOTIONS with Empathic Responses
STEP 6: S—STRATEGY and SUMMARY
3: I will start off by acknowledging that I understand how much he loves his mother and obviously
he has her best interests in mind.
4: Then, I will explain that as his mother’s physician, I have an ethical responsibility to talk to her
about her disease. It is ethical principle of autonomy, right of patient to know about her
condition.
5: I will also explain him that how this will affect to doctor and patient relationship.
6: I will ask the mother (in the presence of her son) how much detail she likes to hear about her
condition and whether she wants to make her own decision or to delegate this responsibility to
her son.
7: In the unusual event that the son remains in disagreement about the process of information
sharing and insist that his mother not be told about her condition, I will resort to inclusion of
other parties such as hospital ethics committee or an organization that deals with breast
cancer patients.
SIGNIFICANT TOPICS WITH SAQs 305
176.
A man who has had a pre-employment check up is found to be HIV positive. He requests you to change
the report as he is absolutely fit at the moment and has spent a lot of money in acquiring this job and if
he is unable to go, his family will be ruined.
ANSWER
➢ HIV is included in STIs, which have exemption to patient confidentiality and physician has duty
to notify and warn public officials who will then notify the people at risk.
➢ Physician is requested to inform and protect potential victim from harm, like his wife/partner.
➢ Physician cannot hide or change any report including pre-employment check. It is illegal.
SIGNIFICANT TOPICS WITH SAQs 306
177.
A 36 year old HIV positive, son of friend presents to your clinic and asks you to write a medical fitness
certificate for him stating that he is healthy and is HIV negative.
ANSWER
A
A)1: this consultation addresses two reasons where breach of confidentiality may b justified
-publoc health reporting of notifiable disease:
-public interest
HIV is included in STIs which have exemption to patients confidentiality and physician has duty to
notify and warn public officials, who will then notify people at risk , physician is supposed to
inform n protect victims from harm like his wife.
-physician cannot hide or change any report including preemployment check it is illegal
B
Tell him. I respect ur concerns sir. I will not tell to any one. But I m bound to inform ur partners n
GP, employer body
Tell him that it's important not only for him but for his family as well. So that they will not get the
disease. Tell him ( emotional blackmailing) that we know that u love ur family n we want them to b
safe n enjoy healthy life
Tell him he needs treatment for few years and it is not a disease which can bring immediate death.
SIGNIFICANT TOPICS WITH SAQs 307
178.
A 40 years old man has returned to his village in Baluchistan after becoming HIV +ve during his stay
abroad. The community elders are concerned about the health risks he poses to the community.
ANSWER
A
Risk Behaviors:
1) Male to male sexual contact.
2) Vaginal sex without a condom
3) Anal sex without a condom with female
4) Multiple partners
5) Injecting drugs with shared equipment
6) Blood transfusion from infected person.
7) Had repeated sexually transmitted infections.
B
Action Plan Being a Family Physician:
1) Counseling with patient and community members:
As a family physician, I will counsel them regarding HIV spread. I will tell them that;
a) HIV is not spread by casual contact.
b) There is no risk of becoming infected with HIV by working on the same assembly line
using same equipment, sharing locker-rooms or toilet facilities or being in the same
office or community.
c) HIV is not found in vomit, faeces, nasal secretions, tears or urine until these fluids are
visibly contaminated with blood.
d) HIV can pass from one person to another in following ways;
a. Unprotected sexual intercourse
b. By using contaminated needles
c. Via transfusion of infected blood or blood products
d. From infected mother to her infant
e. Via organ transplant from infected donor
2) Inform Higher Authority Departments:
• I should inform relevant department for HIV of infected person.
• Inform ministries of health and AIDS programs.
3) Treatment: Referral to nearest Infectious Diseases/HIV Clinic for antiretroviral therapy
(ART);
a. ART is the use of HIV medications to treat HIV infection.
b. ART is recommended for every one who has HIV
c. ART cannot cure HIV, but HIV medicines help people with HIV live longer, healthier.
d. Because of ART, many people can live for decades with HIV and have a normal life.
4) Good Nutrition Diet Plan:
a. I will advise eating a diet high in vegetables, fruits, whole grains and legumes.
b. Choosing lean, low fat sources of protein
SIGNIFICANT TOPICS WITH SAQs 308
179.
A 70 year old gentleman comes to your clinic with his daughter to discuss his histology report following
colonoscopy for his symptoms of weight loss and altered bowel habits. The histology report you have
reviewed, confirmed the diagnosis of colonic cancer.
ANSWER
A
3 DOs
1, Check the facts first and ensure that you have all the informations about the fact. Ensure
privacy and freedom from interruption
2, ask the patient if he wants to know the fact n to what extent he wants to know abt his
condition. Ask him if he wants some family member / friend to be with him while breaking the
news
3, make proper eye contact , use simple n straight forward language .allows silence , tears and
anger
3 DONTs
1, do not lie or fudge the issue
2 , do not break the news in public
3, do not give the impression of being rushed by interrupting or arguing with patient .. don't give
the impression that u have no sympathy and empathy towards patient.
B
1, try to find out the worries of daughter, that r not letting her to disclose the news to father , and
try to solve or answer her worries
2, convince the daughter that without knowing abt the disease , the father might not b co
operative for seeking medical treatment or chemotherapeutic treatment
3, tell the daughter that being as a doctor we can not hide the facts from a patient if he / she
wants to know abt the disease and it's progression.
SIGNIFICANT TOPICS WITH SAQs 310
180.
A 50 years old man presents to your clinic with complaints of infertility. His semen analysis shows he
has azoospermia. He requests you to tell his wife on next visit his semen analysis report is normal and
that will allow to him to re-marry.
ANSWER
A
A doctor is not ethically obligated to lie with patient’s families or anyone’s on patient’s request.
Doctor wouldn’t provide any wrong information to anyone by making false interpretation of
pateint labs results. Pateint should be encouraged to discuss his lab results with wife.
B
Suggest the pateint to seek medical help, assist him make a necessary referral with concerned
subspecialty. Educate the patient regarding adaptation and encourage him to discuss with his
wife. Also pateint May consider other methods of fertilization such as artificial insemination. Offer
the patient that he can bring his wife on next visit that doctor can discuss in further details
So in this case
We will tell patient that we respect ur request. We will never break dr patient relationship
We will not disclose your illness to anyone until unless u allowed us to do
But it's not something which is not treatable. We can inform ur GP n ur family ( if you allows) so
that they can support u in seeking good treatment
There are so many ppl who r living in the world as a couple but they dnt have kids
So just ignore what u r suffering from. Inform ur wife in a friendly environment n I hope she will
respect u by not hiding things from her. We can inform her that
There are different ways u can enjoy parenthood , u can adopt a child , in this way u will b able to
provide a home shelter for someone who can live with you n u can enjoy the feelings of parent
hood
SIGNIFICANT TOPICS WITH SAQs 311
181.
A female patient of yours with advanced osteoarthritis of both knees has been advised bilateral knee
replacement. She has refused to have surgery. She is able to walk and do all her activities with a
walking stick. Her daughter asks you to tell your patient to have the surgery as it is necessary to save
her life.
ANSWER
A
Ethical Issues:
1) I am a doctor and as a doctor my responsibility is to tell advantages and disadvantages of this
surgery both to the patient.
2) I should not force her to do surgery or tell her that your daughter is insisting me to force you
for the surgery.
3) I tell mother tha osteoarthritis is a progressive disease and it gives her benefits and improve
her lifestyle; otherwise she has to use stick the whole life and bear this painful condition.
4) I tell patient the complication of this surgery, which is mainly pulmonary embolism most
common, and abscess or infection postoperatively.
5) I tell mother there are so many patients go for this bilateral knee replacement and enjoying
their life and no need for any support or help.
6) I tell mother, your age is going to increase and if you do not go for surgery, you have to arrange
some nurse or servant to take care of you.
7) If refuses surgery, I should guide her to take physiotherapy and should take medication for
pain.
A
Counselling of Daughter:
1) Tell daughter that osteoarthritis is not a life-threatening condition and there is no danger to
her life without this surgery.
2) Tell daughter that her mother is sensible lady, able to walk and do all her activities and this is
her right to take decision.
3) Tell her daughter, there are chances of preoperative, intraoperative and postoperative
complications and tell her all risk factors.
4) Tell daughter that I already tell all pros and cons of this surgery and if she is not willing do not
force her.
5) Tell daughter, give her mental support. She needs your care and love, so be patient, rather
than making her mother afraid and anxious.
6) Tell daughter that if ur mother is having osteoarthritis and it has impacted her ability to work,
she may qualify for Social Security Disability benefits.
7) Tell her daughter Osteoarthritis is a chronic disease. There is no cure, but treatments are
available to manage symptoms. Long-term management of the disease will include several
factors: Managing symptoms, such as pain, stiffness and swelling.
8) Prescribe for her mother,,,NSAIDs, including ibuprofen and naproxen sodium at the
recommended doses, typically relieve osteoarthritis pain.
SIGNIFICANT TOPICS WITH SAQs 312
9) Tell her daughter,Social Security Disability Benefits for Osteoarthritis. If you have been
diagnosed with osteoarthritis and the pain and stiffness resulting from the disease make it
impossible for you to work, you may be eligible to receive social Security disability benefits.
10) Tell her daughter avoid these things from diet .Sugar
a. Salt,
b. Fried food,
c. White flour,
d. Omega-6 fatty acids and
e. Dairy.
f. Osteoarthritis and alcohol
11) Tell daughter walk with her mother ,if she exercise, it can eventually make her knees feel
better. ... There are many ways that walking helps ease the pain of knee OA.
12) Tell her to make a diet plan .eat Foods rich in antioxidants – plentifully found in most
vegetables and fruit – may help reduce tissue damage from inflammation. Eat oily fish such as
salmon, or try another source of omega-3 fatty acids such as walnuts or freshly ground flax
seed daily. Regularly use ginger and turmeric.
SIGNIFICANT TOPICS WITH SAQs 313
182.
One of your family friends comes to your clinic and requests for a medical certificate for her child’s
illness as asked by the school. The child was sick 2 weeks ago and was seen by some other doctor and
treated.
ANSWER
• The issuance of medical certificate in back date is illegal and unethical.
• It may be dangerous, may be a legal issue.
• Ethically if you consider it sympathetically or you are pressurized in any way any
recommendation, etc ., ask the parent to submit an application to school that their son was
ill, they should raise objection to submit the certificate and should issue a docket or mark
on this application. Keep this docket as a legal reference.
• The parent should be advised to bring the receipt from other doctor if any.
• First response: Why not did they get certificate from the first doctor. Has he refused or he
was a quack ?
• See the child for existing complaint if any.
• May issue MC if remnant of illness present with history of day since his illness.
• Always advise rest if seems necessary onward from this day.
• Do not take risk to give advice from previous date, which is illegal, unethical and should be
flatly refused.
• Otherwise, in case of compulsion, he should be referred to district standing medical board.
• Always maintain your integrity
SIGNIFICANT TOPICS WITH SAQs 314
183.
A friend of yours approaches you with the request to provide him a copy of the treatment of his brother-
in-law. His sister has filed a divorce claim in court of law and proof of her husband being treated for
erectile dysfunction will support her position.
ANSWER
Case of Medical Ethics:
Four commonly accepted principles of healthcare ethics.
1) Principle of autonomy
2) Principle of non-maleficence
3) Principle of beneficence and
4) Principle of Justice
• Doctors and health institutions and medical personnel have a duty to maintain patient’s
records in confidence but there are some circumstances in which they may be obliged to
give this information to third parties. There are some circumstances in which a health
professional (or hospital) may disclose confidential medical records to others – for example;
if the patient consents to such disclosure or when it is required by a court of law. It seems
that it may also be ethical to disclose medical records if it would be in the patient’s best
interest or if necessary to protect another person or society generally.
• In this case, my patient’s privacy is my first preference and secondly erectile dysfunction is
not an untreatable disease.
• I will explain to my friend the causes of erectile dysfunction. Causes of erectile dysfunction:
o Smoking
o DM
o Neurological causes
o CHD
o Testosterone deficiency
o Psychogenic causes
▪ Depression
▪ Performance anxiety
▪ Relationship failure
▪ Fear of intimacy
o Drugs (antihypertensive, SSRIs, cimetidine, finasteride, tranquillizers, antiandrogenic)
• Reassure him (about positive aspects)
• After all explanation if he insists for any medical record, refer him to medical administration
after bringing the written order from court to need any record.
SIGNIFICANT TOPICS WITH SAQs 315
184.
A 25 years old young man presents with sudden onset of pain on right lower chest and shortness of
breath after a road traffic accident. His blood pressure is 100/70 mmHg and pulse is 100/min, regular.
He is cyanosed.
ANSWER
A
Most Likely Diagnosis:
Tension Pneumothorax
A
Patient Evaluation: For evaluation of this patient, we need detailed history, proper examination
and investigations.
History: As about;
• Site and nature of pain, any history of trauma.
• Duration
• Associated symptoms (breathlessness, cyanosis)
• Provoking and relieving factors
• Past medical history and family history (heart disease), drug history, smoking history
Examination:
• Check BP in both arms.
• General appearance – distress, sweating, pallor, GCS
• JVP and carotid pulse
• Respiratory rate
• Apex beat
• Heart sounds
• Lung fields
• Local tenderness
• Pain on movement of chest
• Skin rashes
• Swelling or tenderness of leg (? DVT or ? PE)
Investigations: ECG and chest x-ray (though in pneumothorax it is clinically diagnosed, just for
confirmation)
• Chest x-ray: Mediastinal shift, trachial deviation away from the side of the pneumothorax
Management:
Sit the patient upright if possible.
Cervical collar, then assess for airway, breathing and then circulation.
Insert a large-bore cannula through the 2nd intercostal space of the chest wall in the
midclavicular line on the side of the pneumothorax to relieve the pressure in the pleural
space.
Main two IV lines, large bore cannula. Initially 2 liters of N/S infused, as internal bleeding
cannot be ruled out in trauma case.
SIGNIFICANT TOPICS WITH SAQs 316
Transfer as an emergency to tertiary hospital with hard board and log roll technique
Recommendations: In pneumothorax;
✓ If the rim of air is < 2 cm and patient is not short of breath, then discharge should be
considered.
✓ If the rim (1-2 cm), aspiration
✓ If the rim > 2 cm, chest drain should be inserted.
SIGNIFICANT TOPICS WITH SAQs 317
185.
A young man was lying on his back when a motorcyclist accident drove over his upper abdomen from
right to left.
On examination: Patient was breathing rapidly, pulse rate was 130/min and the BP was 80/40 mmHg.
The abdomen did not move on respiration and on palpation, it was tender and rigid all over. No bowel
sounds could be heard.
ANSWER
If on scene:
✓ Assess the scene.
✓ Call emergency services immediately.
Immediate Treatment:
Don't allow to move as he could have back injury.
Briefly assess the patient by examination including vital signs especially BP, pulse pallor.
Assess for GI bleeding and state of consciousness.
Lie patient flat and lift legs higher than body.(feet on pillow)
Apply cervical collar, assess for airway, breathing and circulation.
Insert two large bore cannula IV if possible take a sample of FBC and cross match on insertion.
Give 2 L of N/S stat.
Give oxygen if available.
Start IV fluids.
Don't give anything by mouth.
Keep full record of event, action taken and drugs given and send it with patient to hospital.
Transfer as rapidly as possible to the hospital with hard board and log roll technique for chest
tube insertion/fracture management as required.
SIGNIFICANT TOPICS WITH SAQs 318
186.
A two years old boy is unconscious for the last 3 hours. He has shallow respirations. His respiratory rate
is 18 per minute and blood pressure 60/40 mmHg. He has pinpoint pupils and cold skin. His pulse is
thread and low volume.
A. What is the most probable diagnosis? Give justifications for your diagnosis.
B. How will you investigate him?
C. How will you treat him?
ANSWER
A
Most Probable Diagnosis
Accidental organophosphate ingestion (insecticide)
Logical Justification: Pinpoint pupils and muscarinic effects including hypotension and respiratory
distress and unconsciousness points towards OP poisoning.
B
Investigations:
✓ Capillary blood glucose
✓ CBC
✓ Serum electrolytes
✓ BUN
✓ BSR
✓ LFTs
✓ Serum amylase
✓ RBC cholinesterase/plasma cholinesterase activity
✓ ECG
C
Management:
Detailed History:
Examination:
o Vitals recording
o Complete GPE and systemic examination
o Characteristic petroleum like odour, may be helpful in establishing diagnosis.
General Management
Maintenance of ABC
o Airway should be cleared of secretion.
o Breathing
o Circulation
Place the child in lateral, preferably head down position with neck extension to reduce risk
of aspiration.
Obtain IV access
Clothes must be removed and skin vigorously washed to remove contamination and to
avoid more of chemical absorbance.
10% Dextrose – 3.5 mg/kg/min in case of documented hypoglycaemia.
Treatment of acute toxicity
SIGNIFICANT TOPICS WITH SAQs 319
o Aim for heart rate > 70 beats/minute, SBP > 80 mmHg and a clear chest and dilated
pupils
o Atropinization:
▪ Start with 1.8 to 3.0 mg IV fast bolus
▪ After 3-5 minutes, check for 5 points of cholinergic poisoning
1) Poor air entry due to bronchospasm
2) Excessive sweating
3) Bradycardia < 60 beats per minute
4) Hypotension
5) Miosis
o If not corrected:
▪ Double the dose of atropine until at least 3/5 of below parameters achieved.
1) Clear chest without wheeze
2) Dry axillae
3) Heart rate 80 to 100 beats per minute
4) SBP > 80 mmHg
5) Pupils are no longer pinpointed.
o Maintenance Infusion:
▪ Once stable, start infusion of 5% Dextrose containing 10% to 20% of total
initial dose of atropine on hourly basis.
If condition not improving, urgent referral for tertiary care management.
SIGNIFICANT TOPICS WITH SAQs 320
187.
6 months old child is brought by her mother as she is very much concerned about her umbilical hernia.
A. List one condition, which can be associated with this and need to be thought of.
B. Briefly outline management plan for the child’s umbilical hernia, who is otherwise healthy.
ANSWER
A
Early onset of seizure,mental retardation,delayed developement of speech,presence of facial
dysmorphism and undescended testis are associated.
Congenital hypothyroidism must be ruled out bc late diagnosis n treatment will lead to permanent
mental retardation n other problems
B
1-complete history: when did u notice this swelling.
Is the size increasing since u noticed it
Bulge increases with crying or coughing
Treatment taken (hakeems, quacks, stripping)
Past history: constipation
Prolonged neonatal jaindice
Poor feeding
Birth wt n wt gain
Sleepy/ less cry than normal babies
Any problem noticed on neonatal examination or any later doctor visit
Neonatal screening done? Reports?
Milestones acheived, neck holding, sits with support,
Family h/o unexplained retardation
2-physical examination local
Site, size, shape, contents, reducibility, bulging on crying
Wt, GPE, dysmorphic features, macroglosia
Developmental assesment
3-systmic examination
CNS: ant frontanel wideopen, hypotonia
CVS: murmer
Abd: gastrochisos, omphelocele, diastasis recti
Investigate: T4,TSH
4-reassurance of the mother that umbilical hernia usually close on their own before a baby is 1
year old.if it is not closed uptill 5 year your child will need a surgery to close it.surgery can be done
before 5 year if
1-when hernia show no sign of closing till 2 year
2-the way the hernia look bothers you and child.
3-if your child has pain,swollen belly or any sign of incarcenated hernia.
4-proper follow up and seek medical and surgical advice in case of any emergency related with
that hernia.
Advice: after ruling out any associated problem I’ll inform mother
There is no emergency, she must wait n observe
SIGNIFICANT TOPICS WITH SAQs 321
This problem is due to weakness or imperfect closure of muscles of abdominal wall which is
expected to heal by itself n hernia is likely to resolve by max age of 5-6 yr
Although if defect is larger than 2cm, or causes symptoms, becomes progressively larger, or not
resolves by 5 yrs age, than she must take baby to pediatric surgeon for advise/ surgery.
if hernia becomes strangulated, go to emergency dept.
Strapping is ineffective
SIGNIFICANT TOPICS WITH SAQs 322
188.
A 20 years old man presents with diarrhoea and bleeding per rectum since the last one month. He also
reports tenesmus and some joint pains.
ANSWER
A
Further Questions to be Asked: Ask for;
➢ Relapsing-remitting nature of disease.
➢ Volume, consistency, mucus presence, frequency of loose stoolos per day
➢ Does diarrhoea alternate with bouts of constipation too?
➢ Fecal urgency/incontinence
➢ Lower abdominal crampy pain
➢ H/O fever
➢ Easy fatigability and malaise
➢ Recent weight loss
➢ Smoking history
➢ Systemic signs – mouth ulcers, skin rash, eye findings, thyroid disease, joint/bone disease.
➢ H/O lactose intolerance
➢ Recent travel history
➢ Family history of IBD, coeliac disease and colorectal CA
➢ H/O contact with TB inflicted individuals.
➢ Medication history – antibiotics, NSAIDs
➢
Alternative bowel habits
Rule out stress
Rule out APD
B
Investigations:
✓ CBC + ESR (WCC, Hb)
✓ CRP
✓ U/E
✓ LFTs
✓ Stool C/E and culture (for occult blood, cysts, ova, parasites) faecal leukocytes.
✓ Faecal calprotectin – most sensitive
✓ TST to be done prior to therapy initiation.
✓ Radiological:
o Plain x-ray erect abdomen (to look for toxic megacolon and perforation)
✓ Interventional:
o Proctosigmoidoscopy – cobblestone appearance of mucosa
o Colonoscopy
o Biopsy with endoscopy
SIGNIFICANT TOPICS WITH SAQs 323
o Barium enema
C
Management:
Detailed History
Detailed Examination:
o Vital Signs – to look for signs of systemic disturbance.
o GPE – to look for;
o Jaundice
o Pallor
o Signs of weight loss
o Oral ulcers
o Clubbing
o Cachexia
o Nutritional status
o Extraintestinal Manifestations: Look for;
o Musculoskeletal:
▪ Pauciarticular
▪ Polyarticular
o Dermatological
o Hepatopancreatobiliary
o Ocular
o Renal
o Pulmonary
o Abdominal and Perineal Examination:
o Palpate abdomen in all 4 quadrants and look for tenderness, rebound, guarding,
distention, hepatomegaly and surgical scars.
o Observe perianal region for presence of tags, fissures, fistulas and abscesses.
o Perform DRE to assess for anal strictures and rectal masses.
o Treatment:
o Non-Pharmacological:
Address patient’s ideas, concerns and expectations.
Health education and reassurance
Inform patient about disease’s relapsing and remitting nature and lifelong
need for treatment.
Low-residue diet during diarrhoeal phase of disease.
Reduce physical activity during articular flare-ups
o Pharmacological:
Tablet Mesalazine 2-4 grams/day for 3-12 weeks
Topical 5-ASA derivates are useful; adjusted if troublesome rectal symptoms
If symptoms persist, we can add;
o Tablet Predinsolone 40 mg OD PO
o Review 2-weekly and lower dose of steroids over eight weeks.
Immune modifiers, e.g., tablet azathioprine 2-3 mg/kg/day PO used to reduce
steroid course/year, or in case of relapse < 6 weeks after stopping steroids.
o Urgent Referral:
SIGNIFICANT TOPICS WITH SAQs 324
189.
A 40 years old alcoholic presents with sudden onset of severe stabbing epigastric pain that radiates to
the back since last 7 hours. He has had 4 episodes of non bloody vomiting. On examination, PR 130
beats/min and BP 110/70 mmHg. Abdominal examination reveals guarding present and bowel sounds
are present.
ANSWER
A
Most Likely Diagnosis:
Acute pancreatitis
B
Investigations:
✓ Blood tests
o Serum amylase
o Serum lipase
o CBC
o Blood urea and nitrogen level (RFT)
✓ Imaging
o Ultrasound Abdomen
o CT scan abdomen
o Magnetic resonance cholangiopancreatography MRI
o Endoscopic retrograde cholangiopancreatography
C
Immediate Plan: Reassurance and explanation.
Treatment
1) Admit/refer the patient.
2) Fasting (NPO):
a. Stop eating
b. Once the inflammation of pancreas is controlled, patient may begin drinking clear
liquids and eating bland foods.
3) Pain Medications
a. Pancreatitis can cause severe pain, so give strong analgesics.
4) IV Fluids & Antibiotics
a. To dehydrated, give extra fluids through a vein, R/L.
b. Give one dose of IV antibiotics stat, e.g., 1 gram IV Ceftriaxone.
c. Maintain input/output chart.
5) Referral to a tertiary hospital for surgery for treatment of the cause with proper referral
notes. Surgery may include;
a. Endoscopic retrograde cholangiopancreatography (ERCP
b. Gallbladder surgery (cholecystectomy)
c. Pancreas surgery
SIGNIFICANT TOPICS WITH SAQs 326
SIGNIFICANT TOPICS WITH SAQs 327
190.
A 40 year old bank executive presents with a 2 year history of lower abdominal pain associated
with constipation alternating with diarrhea. He passes mucus per rectum regularly. He denies
passing any blood per rectum and has not had any fever or weight loss.
Physical examination is entirely normal.
ANSWER
A
Provisional Diagnosis:
Irritable bowel syndrome *
B
Management:
Confirmation of Diagnosis: According to CKS-NICE
➢ Consider the diagnosis of irritable bowel syndrome (IBS) in a person who has had any of the
following symptoms for at least 6 months:
➢ Abdominal pain or
➢ Bloating, or
➢ Change in bowel habit.
➢ Make a diagnosis of IBS if a person has abdominal pain which is either:
➢ Related to defecation, and/or
➢ Associated with altered stool frequency (increased or decreased), and/or
➢ Associated with altered stool form or appearance (hard, lumpy, loose, or watery); and
there are at least two of the following:
➢ Altered stool passage (straining, urgency, or incomplete evacuation).
➢ Abdominal bloating (more common in women than men), distension, or hardness.
➢ Symptoms worsened by eating.
➢ Passage of rectal mucus, and
➢ Alternative conditions with similar symptoms have been excluded.
➢ IBS may also present with:
➢ Lethargy, nausea, back pain, headache.
➢ Bladder symptoms (such as nocturia, urgency, and incomplete emptying), dyspareunia,
or faecal incontinence.
Investigations: IBS is a diagnosis of exclusion, there is no specific investigation to confirm a
diagnosis.
✓ Full blood count (FBC) — to assess for anaemia, and a raised platelet count may suggest
active inflammation as an alternative cause for symptoms.
✓ Inflammatory markers such as erythrocyte sedimentation rate (ESR) and Creactive protein
(CRP) — may be raised if there is active inflammation or infection.
✓ Coeliac serology — to exclude coeliac disease, particularly if there is diarrhoea predominant
IBS or mixed symptoms.
Nonpharmacological
Explore further hx and physical exam and look for red flag points
SIGNIFICANT TOPICS WITH SAQs 328
191.
A 54 year old airline pilot presents with the complaint of sudden loss of vision in the left eye. It lasted
for approximately a minute or two.
ANSWER
A
Most likely diagnosis is
Amaurosis fugax.
B
Risk factors are:
➢ Age: Risk increases with increasing age.
➢ BP. Risk increases if systolic BP>140 and diastolic>90
➢ DM
➢ Obesity
➢ Alcohol
➢ smoking
➢ MI
➢ Polycythaemia/high altitude
C
Basic Investigations:
As a family physician, I 'll do following basic investigations.
✓ Full Blood count(CBC),ESR
✓ U&E
✓ Creatinine
✓ Lipid profile
✓ Blood glucose level
✓ ECG
✓ CXR
D
Advanced Investigations:
✓ CT or MRI (to confirm diagnosis).
✓ Carotid Doppler if carotid artery territory symptoms.
✓ Echocardiogram.if recent MI,CCF/LVF or murmurs.
E
Plan of Action:
I will take detailed history of patient about:
➢ h/o smoking
➢ h/o DM
➢ h/o of any heart disease(MI,artificial valve)
SIGNIFICANT TOPICS WITH SAQs 330
192.
A 45 years old male comes to you with complaints of irritation, burning and pain in his right eye for 2
days. On examination, he has mucus discharge and excessive lacrimation from the right eye. His
conjunctiva is hyperemic. He has no other complaints and is healthy otherwise.
ANSWER
A
Most Likely Diagnosis:
Viral conjunctivitis
B
Three More Differential Diagnosis:
❖ Bacterial conjunctivitis
❖ Allergic conjunctivitis
❖ Blepharitis
❖ Foreign body in the eye
C
Management Plan for Most Likely Diagnosis:
Viral causes of conjunctivitis generally are self limited and treated supportively with cool or
warm compresses for comfort
Topical antihistamines and topical vasoconstrictors to limit redness and itching.
Chilled artificial tears for comfort
Topical antibiotics as necessary to prevent bacterial superinfection.
The infection will usually clear up in 7-14 days without treatment and without any long-
term consequences.
Stop wearing contact lenses
Viral Conjunctivitis Treatment
o Successful and rapid treatment involves the use of combined therapy
o 0.1% dexamethasone +0.4% povidone-iodine
o Cidovir 0.5% (combination of dexa and tobramycin)
o Emadastine/Azelastine (antihistamine)
D
Prevention:
1) Both patient and provider should wash hands thoroughly and often
2) Keep hand away from infected eye and contralateral eye
3) Avoid sharing towels, linens, pillows and cosmetics
4) Patient should be advised to stay home and take off from work
5) If patient is wearing contact lenses should be instructed to discontinue lens wear until signs
and symptoms have resolved.
6) Protect eyes from dirt and other irritating substances.
SIGNIFICANT TOPICS WITH SAQs 332
7) Wash any discharge from the eyes several times a day using a fresh cotton ball or paper
towel.
8) Always discharge the cotton ball or paper towel and wash hands with soap and warm
water.
9) Do not use eye drops that were used for an infected eye in a non-infected eye.
10) Wash hands after applying eye drops or ointment to the eye.
11) Wear eye glasses or sunglasses.
12) Avoid touching face since virus can enter eyes, nose and mouth
13) Do not go close to anyone suffering from conjunctivitis
SIGNIFICANT TOPICS WITH SAQs 333
193.
A 60 years old lady attends your clinic with a history of severe pain over the forehead and behind the
right eye, visual disturbance and two episodes of vomiting. On examination, she has redness around the
cornea.
A. What further points in this patient’s history and examination will help you in the diagnosis?
B. What are three most likely diagnoses?
C. How will you manage this patient?
ANSWER
A
Points in History:
➢ unilateral or bilateral
➢ severity
➢ quality( throbbing/ squeezing)
➢ duration and frequency: when started, duration of attack
➢ aggravating or relieving factors : food, rest, menstruation, movement, lack of sleep, stress
➢ Associated symptoms: nausea, vomiting, fever, visual changes like halos, photophobia,
neurological symptoms, phonophobia
➢ sympathetic symptoms: lacrimation, rhinorrhea, ptosis, miosis, nasal congestion, eyelid
edema
➢ awakens pt from sleep
➢ do headaches go completely between attacks
➢ aura( flashing lights, shimmering lines, numbness, strange odors, speech disturbance
➢ general health, hypertension, eye problems,
➢ medicine : Indomethacin, nifedipine, cimetidine, captopril, nitrates, OCPs
➢ drugs used for headache
B
Most Likely Diagnoses:
❖ Cluster headache
❖ Meningitis
❖ Acute close angle glaucoma
C
Management:
• History: Please see above.
• Physical Examination:
-red flags: fever, worsening headache, purpura, thunderclap headache, associated with
postural change, sneezing, head injury within 3 months, neurological deficit, papilledema,
aura for first time or atypical aura, new headache in HIV or cancer pt. if any of these, refer
urgently.
- bp, fundi, visual acuity, palpation of temporal region and sinuses, examine neck
• Investigation: No need in this pt
• Treatment Plan: It is most likely a case of cluster headache; so management includes:
Refer for specialist advice / neuroimaging for first bout of cluster headache
Acute attack:
SIGNIFICANT TOPICS WITH SAQs 334
194.
4 years old Marium is brought by her mother to your clinic, with complaints of high grade fever for 2
days, irritability and left-sided earache. On examination, the child has 102 F temperature and a red
opaque and bulging left tympanic membrane.
ANSWER
A
Diagnosis:
Acute otitis media
B
Management:
Fluids + analgesia + antipyretics like paracetamol, ibuprofen.
Prescribes antibiotics like amoxicillin TDS.
Warm compresses
Topical anesthetic drops (benzocaine)
C
Advice to Mother:
Avoid passive smoking
Avoid supine bottle feeding
Influenza and pneumococcal vaccination.
Send to smaller day care
SIGNIFICANT TOPICS WITH SAQs 336
195.
A 55 years old obese lady presents with pain in her right knee for one year. She denies any associated
redness or swelling. Radiological examination shows joint space narrowing and marginal sclerosis.
ANSWER
A
Most Likely Diagnosis:
Osteoarthritis
B
Questions Related to Impact on Daily Life:
➢ What does the pain feel like? (dull and achy or sharp and intense.
➢ What makes the pain worse and what makes it better? (pain increased during repeated use of
joint or pain reduced after you rest?)
➢ Are any of your other joints involved? or is it bilateral?
➢ Has the joint ever been injured?
➢ Are you taking any medication and is there any improvement?
➢ What activities are you no longer able to do that you wish to do?
➢ Are you able to climb stairs and go for jogging or walk with your family or friends?
➢ When is the pain at its worst?
➢ What triggers the pain?
➢ Do you have morning stiffness and how long it lasts?
➢ What type of work do you do?
➢ What other medical conditions do you have?
➢ Have you had fever, chills or unexplained weight loss?
C
Management of Osteoarthritis:
Detailed History:
Complete Physical Examination:
o GPE
o Neurological Exam
o Systemic Exam
o Joints Exam: Walk, bend and move joints of hands, arms, legs, feet and spine (to know how
many joints are affected)
Investigations:
✓ X-ray: To determine the extent of cartilage loss, bone damage, bone spurs and narrowing
of joint spaces.
✓ MRI: To help detect abnormalities that occur in the early stages of osteoarthritis.
✓ Joint Aspiration: Fluid examined under microscope to check for number and type of white
blood cells present
Non-Pharmacological:
Exercise; It strengthens the muscles around joints and make joints more stable.
SIGNIFICANT TOPICS WITH SAQs 337
Lose weight: Overweightedness increases the stress on weightbearing joints and creates
more pressure and pain.
Use heat and cold: Both heat and cold relieve pain in joint;
o Heat relieves stiffness
o Cold relieves muscle spasms and pain.
Knee taping: Strapping tape may help ease the pain of knee osteoarthritis.
Use Assistive devices: Use cane or stick and do not stress your painful joint.
Advice physical therapy and occupational therapy.
Pharmacological:
Acetaminophen
NSAIDs
Duloxetine
Surgical & Other Procedures:
Cortisone injections: 3-4 injections of cortisone in the joint relieve pain in the joint.
Lubrication Injections: Injections of hyaluronic acid may offer pain relief by providing some
cushioning in the knee.
Realigning bones by knee osteotomy
Joint Repalcement – arthroplasty. Surgical risks include infections and blood clots leading
to PE
SIGNIFICANT TOPICS WITH SAQs 338
196.
A 55 years old man presents with 4-week history of increased urinary frequency, poor stream of urine
and urgency. On examination, you find prostate enlargement.
ANSWER
A
Provisional Diagnosis:
Enlarged prostate (benign prostatic hypertrophy most likely if diagnosed histologically)
B
Investigation
✓ Urinalysis/Dipstick – look for glycosuria, proteinuria, haematuria, nitrites, leucocyte
esterase.
✓ Urea/creatinine – to check renal function
✓ FBC – Hb% with TLC
✓ S. PSA
o Sample must be drawn either before performing DRE or 1 week after DRE.
o Active UTI, ejaculation (within 48 hours) and vigorous exercise also merits as
delaying factors for S. PSA by at least 48 hours.
o PSA around 4.0 ng/ml is normal and values can be correlated with clinical signs and
symptoms
✓ Imaging:
o USG KUB along with PMRV (post-micturition residual volume) – look for
▪ Prostate weight (20 g – normal, increase in weight must be correlated with
presence or absence of LUTS before planning treatment
▪ PMRV
• < 30 ml – normal
• > 40 ml – needs further assessment.
• > 200 ml + signs of hydroureter or hydronephrosis – patient needs
catheterization in a specialized unit.
C
Treatment Options Available
Non-Pharmacological:
-Adress ICE
-let patient know that symptoms could improve spontanously.
-watchful waiting
-decrease evening fluids intake
-decrease caffeine
-bladder retaining
-prevention of constipation.
Pharmacological: Treatment options depend upon scoring symptoms of the patient according to
International Prostate Symptom Score (IPSS):
SIGNIFICANT TOPICS WITH SAQs 339
➢ 7 symptoms to be stratified.
➢ Every symptom is ranked 1-5 (mild to severe) (a net total of 35)-watchful waiting: with mild to
moderate symptoms with no complication of BPH can be observed
1) Irritative LUTS (lower urinary tract symptoms)
• Frequency
• Urgency
• Nocturia
2) Obstructive LUTS
• Intermittency
• Straining
• Poor stream
• Sense of incomplete evacuation
➢ If score is up to 8 – watchful waiting (irrespective of prostate weight)
➢ If score is 8-18 – medical treatment
o Alpha-blockers, e.g., tamsulosin 0.4 mg 1 HS if > obstructive symptoms.
o Anticholinergics, e.g., solifenacin (5-10 mg) BD if > irritative symptoms
➢ If score is > 18
– Finasteride (5 alpha reductase inhibitor) 1 mg HS( if obstructive) for 4 wks.
– surgical management.
Follow-up: Every 4 weeks for assessment of symptoms/finding deterioration.
Referral: If;
• status deteriorated from previous
• PSA > 4.0 ng/ml
• Marked PMRV > 200 ml
• Suspicion of malignancy (nodular prostate on DRE)
• Bilateral hydroureter or hydronephrosis on USG KUB
• Associated vesicle calculus
Urgent Referral: If urinary retention
SIGNIFICANT TOPICS WITH SAQs 340
197.
Mrs. Khan is a 25 years old married lady with complaints of urinary burning with increased frequency
including nocturia for a week. She has had three similar episodes in the past year. On examination, she
is febrile but no other abnormal findings.
ANSWER
A
Management:
Detailed History: Explore the nature of symptoms, e.g.,
➢ Dysuria, urgency, frequency, pattern, haematuria
➢ Relative symptoms, e.g.
o Lower abdominal pain
o Flank pain
o Fever
o Chills or rigors
➢ Associated symptoms like vaginal discharge, dyspareunia, any vesicle or any other kind of
rashes.
➢ H/O previous attacks of UTI/medications
➢ H/O STIs
➢ H/o other medical related problems like HTN, DM
➢ Ask about last menstrual period(LMP) or h/o of missed cycle.
➢ Personal hygiene
Examination:
o GPE/Vital Signs: Signs of oedema (nephrotic syndrome) or puffy eyes
o Lower abdominal palpation of kidneys and suprapubic tenerness or pain
o Vaginal examination – if relative h/o presence of vesicles/discharge or STI
o Renal punch ? in case signs of fever, chills (pyelonephritis)
Investigations:
✓ Urine dipstick for nitrites
✓ MSU (urine bacteriology) for C&S as well (most common E. coli)
✓ HVS – to rule out other relative causes.
✓ Blood urea + creatinine
✓ Renal X-ray in relative lower abdominal pain and fever with rigors and chills.
Treatment Plan
A. Non-Pharmacological:
Address patient’s concerns, ideas and expectations
Educate patient that she might be having recurrent urinary tract infections, which is a
treatable problem.
Advise she needs to have antibiotic course for current infection
Teach voiding techniques like voiding before/after sexual activity
Increase fluid intake
B. Pharmacological:
SIGNIFICANT TOPICS WITH SAQs 341
With history, patient may need empirical antibiotic therapy for ongoing UTI – can be
adjusted after M, C&S report.
Specific Treatment
o Tab. TMP-SMX 200 mg BD 12 hourly for 3 days.
Symptomatic Treatment
o Tab. Paracetamol 1000 mg 8 hourly for 3 days.
C. Follow-up after 3 days with report earlier if symptoms not resolved or develops
deterioration in symptoms.
MSU 2-4 days after starting antibiotics. If positive, patient needs urgent referral for
possible obstruction.
MSU 2 weeks after antibiotics if positive then repeat the course with prophylaxis.
B
Advice to Lower Recurrence of Symptoms:
General Advice:
▪ I will advise to urinate frequently
▪ Increase fluid intake
▪ Double void (i.e. go again after 5-10 minutes)
▪ Void after intercourse
▪ May increase intake of cranberry juice (efficacy is controversial)
Prophylactic Treatment: Following options are advised.
▪ Tab. nitrofurantoin 50 mg stat post coitally OR
▪ Tab. TMP-SMX 100 mg nocte OD OR
▪ Tab. Nitrofurantoin 50 mg nocte OD
▪ Prophylaxis should be continued for 6 months.
SIGNIFICANT TOPICS WITH SAQs 342
198.
A 30 year old school teacher presents with a history of high grade fever for the last 5 days.
ANSWER
A
History Questions: Following questions will be asked
➢ Is fever continuous/intermittent. If intermittent, when it touches baseline (fever free
periods)
➢ Is fever associated with chills/rigors, sweating.
➢ Is fever documented.
➢ Any associated symptoms, like;
o headache, sore throat, runny nose, nausea, vomiting, abdominal pain
o Decreased appetite, myalgias, cough, earache, ear discharge, sputum; color of
sputum?
o Renal origin; burning micturition, suprapubic pain, pain in abdomen post void, pain
radiating from loin to groin
o Loose motions, pain abdomen, blood in stool
o Any rash or boils on skin
o Severe headache with neck stiffness
➢ Any h/o travel to endemic area with severe headache with intermittent fever to rule out
malaria
➢ Continuous use of any drug
➢ Previous History: Any significant past illnesses.
➢ Drug History:
o Taking medications for any reason
o Did try medication for current illness?
B
Differential Diagnosis:
❖ Pneumonia
❖ Malaria (high grade fever with chills, cyclic pattern like after 48 hours), also associated with
nausea/vomiting/headache
❖ Viral syndrome (fever with myalgia, body aches)
❖ UTI (fever with urinary symptoms + nausea + vomiting
C
Investigations:
✓ CBC
✓ MP smear
✓ Urine D/R
✓ Blood culture
✓ ESR and CRP
✓ Chest x-ray
SIGNIFICANT TOPICS WITH SAQs 343
SIGNIFICANT TOPICS WITH SAQs 344
199.
Abdul Ghani a 22 years old electrician attends your clinic with a history of fever for previous one week.
He also complains of generalized aches and pains, headache and a dry cough.
ANSWER
A
Possible Differential Diagnoses:
❖ Cold/influenza
❖ Atypical pneumonia
❖ Acute bronchitis
❖ Acute pneumonitis/allergic alveolitis
❖ Pleural effusion
❖ Meningitis
B
Management Plan:
History: Detalied history.
➢ Fever:
o High grade or low grade
o Continuous or intermittent
o Associated with chills/sweating
o h/o diarrhoea
o Responding to antipyretic?
➢ Cough: As history says it is dry, then I will ask about;
o Pattern of cough
o Variation with day
o Any nocturnal cough?
o Is cough associated with shortness of breath, runny nose, sore throat
➢ Are symptoms associated with any swelling in feet.
➢ Any skin rash
➢ Social History:
o History of smoking
o Any allergen exposure at work place.
o Any alcohol intake
➢ Medical History: DM, HTN
➢ Drug History: History of ACE inhibitor use.
Investigations:
✓ X-ray chest
✓ CBC
✓ Mycoplasma serology
Treatment:
A. Non-Pharmacological
Educate patient
SIGNIFICANT TOPICS WITH SAQs 345
Address ICE
Steam inhalation BD
B. Pharmacological
If diagnosis suggested by history as influenza, then threatment will be separate
Tab. Paracetamol 2 tablets 500 mg TID for 5 days.
If investigation found atypical pneumonia, then Tab. Clarithromycin 500 mg BD for 10
days
SIGNIFICANT TOPICS WITH SAQs 346
200.
A patient of type 2 diabetes on oral hypoglycaemics attends your clinic for a yearly check up.
What clinical and laboratory assessments will you cover at this visit?
ANSWER
Clinical Assessment:
History:
Before history, ask what medication patient is taking for type 2 diabetes. As patient is already
diagnosed for type 2 diabetes, so I am more concerned regarding diabetes complications. I would
like to ask these questions.
1) Do you have tingling, numbness, burning or pain that usually begins at the tips of toes or
fingers and gradually spread upwards (neuropathy)
2) Ask for history of decreased vision? And result of last detailed eye examination (as diabetes
increases risk of cataracts and glaucoma).
3) Do you have any cut, blister or wound/ulcer in your toe, foot or leg? (diabetic foot)
4) Do you have any hearing issue? (hearing impairment more common in diabetes)
5) Do you have any recurrent infections? (uncontrolled diabetes)
6) Do you have history of loss of consciousness (hypoglycaemia)
7) Do you self monitor your blood glucose levels on regular basis?
8) What is the ptient’s immunization history? e.g., influenza, pneumococcal, hepatitis B,
tetanus, herpes zoster
9) Does patient give history of recent polyuria, polydipsia, nocturia or weight loss?
(uncontrolled diabetes)
10) Does patient have hypertension?
11) Does patient have coronary artery disease?
12) Has the patient had a stroke or transient ischaemic attack?
13) Does patient have known kidney disease and what were dates and results of last
measurements of urine protein and serum creatinine levels? (nephropathy)
14) Is the patient taking lipid lowering medications?
15) When last HbA1c measured and its result.
16) Ask about drug adherence and side effects
17) Ask dietary schedule
18) Depression screening
Physical Examination:
o Vital Signs: BP, HR, RR, Temp, Height, Weight, BMI, abdominal circumference
o GPE: Anaemia, clubbing, thyroid, injection site (if on insulin), lymph nodes, overall health,
obesity, skin (acanthosis nigricans)
o Cardiovascular:
o Check pulse and BP
o Listen for carotid bruits and to heart sounds/lung fields
o Palpate and record the peripheral pulses of feet?
o Eyes
o Inspect xanthelasmata, cataract formation or ophthalmoplegia
o Check visual acuity
SIGNIFICANT TOPICS WITH SAQs 347
201.
Write a short essay of 200 to 300 words on macrovascular and microvascular complications of diabetes
mellitus.
ANSWER
A
Macrovascular Complications of Diabetes:
Macrovascular complications of diabetes are primarily diseases of;
1) Coronary arteries
2) Peripheral arteries
3) Cerebrovasculature
These can be classified in two stages.
1) Early macrovascular disease is associated with atherosclerotic plaque in the vasculature
supplying blood to the heart, brain, limbs and other organs.
2) Late stages of macrovascular disease involve complete obstruction of these vessels, which
can increase the risks of myocardial infarction, stroke, claudication and gangrene.
Cardiovascular disease (CVD) is the major cause of morbidity and mortality in patients with
diabetes.
202.
Mr. Mahmood, aged 56, comes to your clinic for a routine health check. On urinalysis, he has 2%
glycosuria, but no ketones. His weight is 140 kg and he smokes 25 cigarettes per day.
ANSWER
A
Initial Management Plan:
I 'll take detailed history and do relevant examination.
History includes:
➢ h/o diabetes and for how long symptoms appeared(polyuria,polydypsia)
➢ h/o smoking and duration of smoking.
➢ h/o hypertension
➢ h/o alcohol consumption
➢ h/o of current complication of diabetes, i.e., decreased vision, skin changes
(infection,xanthoma), foot changes or urinary complaints (UTI, dysuria)
➢ drug history: if taking any hypoglycemic or any other medication.
➢ past history of cvd
➢ h/o life style and dietary history.
➢ socioeconomic history
➢ family history of DM,HTN or CVD.
Examination:
o GPE, BP, pulse, BMI, edema, pallor.
o CVS, CNS, chest examination.
o Look for skin changes, foot and eye examination (to rule out diabetic complications)
Investigations:
✓ Blood sugar fasting and random.
✓ HbA1c
✓ Lipid profile
✓ Serum electrolytes, s/urea, creatinine, eGFR
✓ LFTs
✓ Urine dipstick for RBC, protein, albumin:creatinine ratio
✓ ECG if needed
✓ Chest X-ray
✓ Cardiovascular risk assessment
Non-Pharmacological:
Reassure the patient.
Explanation about nature of disease risk factors and complications.
provide written information about disease.
Cessation of smoking
Dietary management by taking healthy diet, take fruit and vegetables, less artificial
sweeteners, canned food and decrease use of fatty foods.
Life style changes and exercise. Advice physical activity and brisk walk for 30 minutes/day.
weight reduction
SIGNIFICANT TOPICS WITH SAQs 351
foot care
sort out and solve psychological problems.
immunization: offer pneumococcal and influenza vaccination.
drug compliance
Pharmacological:
Metformin is first choice as patient is obese. Give 500 mg BD (if there is no renal or liver
disease). Increase dose up to 2 gm as needed.
if not controlled add gliclazide
if not controlled by oral hypoglycemic drugs start subcutaneous insulin.
Follow up:
-Check HBAIc every 3 months and if controlled then every 6 months.
-visit to endocrinologist,ophthalmologist, nephrologist if not controlled or any complication
developed.
SIGNIFICANT TOPICS WITH SAQs 352
203.
A 55 years old postmenopausal obese lady has diabetes mellitus and hypertension for five years and has
presented for annual checkup.
ANSWER
A
Important Findings on Examination:
o GPE: Check for general look, pallor, weight, BMI, JVP, oedema, waist circumference (for
truncal obesity)
o Vital signs including BP, pulse.
o CVS: for heart size,heart rate,rhythm,heart sounds and signs of heart failure.
o Chest: for respiratory rate, crepitations
o CNS: look for neurological signs, numbness, tingling, neuropathic pain., assess fordepression
o Skin changes: Xanthomas, infection, dermopathy. fat atrophy / hypertrophy at insulin inj site if
using insulin, necrobiosis lipoidica, granuloma annulare
o Foot examination: For foot shape,deformity, joints rigidity, foot skin for fragility callus,pulses
and sensitivity to 10 gm monofilament, check footwear.
o Eye examination: Complete ophthalmic examination for cataract, glaucoma, visual acuity,
dilated retinal examination for splinter hemorrhages, hard exudates, microanurysm, cotton
wool spots, venous beading.
o Cardiovascular risk assessment.
B
Lab Investigations:
✓ CBC for Hb, WBC count.
✓ Blood glucose random and fasting.
✓ HBA1c
✓ Lipid fasting profile including TC, LDL, HDL, and TG
✓ Serum/urea, creatinine and eGFR for renal function.
✓ Urine:dipstick for RBC,protein,albumin and albumin:creatinine ratio
✓ ECG and Echo if left ventricular failure suspected.
C
Hypoglycemic drugs:
Metformin is drug of first choice in this patient as she is obese and there is no documented renal or
liver disease.
Antihypertensive drugs:
ACE inhibitors (e.g lisinopril) are drug of first choice as she is diabetic n there is risk of renal disease.
SIGNIFICANT TOPICS WITH SAQs 353
204.
A 40 year old man presents to the clinic for a routine checkup. He is hypertensive and takes tab atenolol
50 mg x OD. His last check up was 1 year ago and he has no current issues. He is adherent to his
treatment.
ANSWER
A
Examination:
o Vital Signs: BP measurement in both arms; weight; height; BMI (for obesity)
o GPE: Look for;
o Sweating
o Tremors
o Palpitations
o Edema (proteinuria)
o Pallor or cyanosis
o Systemic Examination:
o Fundoscopy: To check hypertensive retinopathy
o Visceromegaly: To rule out secondary cause of hypertension
o Peripheral edema – to check renal disease.
o Heart Exam – to check left ventricular hypertrophy or ischaemic heart disease or
murmur.
o Assess cardiovascular risk by ASCVD or QRISK2
B
Immediate Treatment: If patient is stable, no immediate treatment is needed, but in following
cases upon assessment;
o If initial SBP is < 180 mmHg and DBP < 110 mmHg
o Make the patient lie supine
o Tab. Capoten 25 mg S/L stat
o Inj. Lasix 20 mg IV stat
o Measure BP after 15-20 minutes. Aim is to drop the BP by 5-10 mmHg
o Repeat if required
o If initial SBP ≥ 180 mmHg but less than 200 mmHg and DBP ≥ 110 mmHg but less than 120
mmHg
o Make patient lie supine in a calm quiet surrounding
o Inj. Labetalol 20 mg slow IV over 2 minutes.
o Inj. Lasix 20 mg IV
o Measure BP @ 5 and 10 minutes interval.
o Additional injection of 40-80 mg of IV labetalol and 20 mg of IV Lasix can be repeated
after 10 minutes of first dose if indicated.
o Goal of BP reduction is by 10-15 mmHg.
SIGNIFICANT TOPICS WITH SAQs 354
205.
A 60 years old man who has a long history of emphysema visits you with complaints of increasing
dyspnea on exertion. Examination reveals mild hepatomegaly and tachycardia. The lungs are clear.
ANSWER
A
Initial Assessment:
Advanced COPD complicated by cor pulmonale
Ask for;
➢ Worsening exertional breathlessness
➢ Audible wheeze on exertion
➢ Increased fatigue, decreased exercise tolerability
➢ Increasing nocturnal cough
➢ Sputum production (volume, prevalence)
➢ Paroxysmal nocturnal dyspnea
➢ Chest tightness
➢ Recent weight loss
➢ Recent infection e.g. URTI
➢ New onset oedema
➢ H/O smoking
Clinical Examination:
o This lean, uncomfortable patient using accessory muscles of respiration. Look for signs like
o Dyspnoeic
o Tachypnoiec
o Pursed lip breathing
o Absence of cyanosis
o Jugular venous distention
o Ankle oedema
o Hepatomegaly (already mentioned)
o Chest Examination: According to scenario lungs are clear clinically
B
Investigations:
✓ PFTs via spirometry:
o For measuring FEV1/FVC ratio (<70% in COPD) and for checking severity of COPD
▪ Mild FEV1 ≥ 80% of predicted or best
▪ Moderate FEV1 50% to 80% of predicted
▪ Severe FEV1 30% to 49% of predicted
▪ Very severe FEV1 < 30% of predicted.
o Reversibility Test (using bronchodilation/ICS)
✓ Pulse oximetry: For determining SaO2
✓ ABG: To look for respiratory insufficiency.
SIGNIFICANT TOPICS WITH SAQs 356
206.
1) Mr. Farid Ahmed, 52 years old manager of a textile mill on routine medical checkup has BP of 150/90
mmHg. He smokes 20 cigarettes per day.
2) Mr. Sohail Ali is a 42 years old banker who presents to your clinic for a physical checkup. On
examination, his blood pressure is found to be 170/104 mmHg. In this case:
ANSWER
A
Evaluation of Patient Found to Have Elevated BP on a Routine Visit
➢ Detailed hx
• If the HTN was already diagnosed , duration and treatment
• check symptoms to assess for any end organ damage
• shortness of breath/ ankle oedema/ paroxysmal nocturnal dyspnoea/ orthopnoea/ chest
pain palpitations / dizziness headaches /vision changes /body weakness.
• known or previous imp medical hx like diabetes/ cardiac problems / lipid disorders/ stroke/
poor renal function.
• imp past surgical hx
• congenital diseases
• smoking / alcohol hx
• lifestyle habits especially diet n activities
• sleeping habits
• psychosocial stressors
• family hx esp of premature cardiac disease/ diabetes/ dyslipidemia
• detailed review of all medications
➢ Examination: Detailed examination to assess for end organ damage
• BMI
• vitals -recheck BP using recommended protocol
• fundoscopy
• cardiac and lung exam esp auscultation / bruits
• peripheral pulses
• thyroid
• neuro exam
• extremities especially for edema
➢ Investigations
• ECG
• CXR
• BUN, Cr and Electrolytes
• eGFR
SIGNIFICANT TOPICS WITH SAQs 358
• blood sugars
• lipid profile (For cardiovascular risk assessment)
• urine dipstick
B
Management:
Nonpharmacological:
Reduce salt in diet
Attempt weight loss
Regular exercise
Pharmacological:
ACE inhibitors, start with 5 mg OD. Explain the side effects to the patient.
Can titrate dose according to the response of treatment.
Can add diuretics or CCB if ACE inhibitors fail to provide adequate hypertensive control.
Referral Considerations:
If patient has confirmed secondary HTN based on above investigations, then refer for
further management by an internal/general medicine specialist.
SIGNIFICANT TOPICS WITH SAQs 359
207.
Mr. Hameed is a 58 years old laborer who has just been diagnosed as having angina. He has been
started on atenolol and is awaiting a consultant outpatient appointment. His angina initially seemed to
be well controlled, but now he complains of chest pain on exertion.
What could be the reasons for Mr. Hameed’s angina not improving?
ANSWER
Reasons for Non-Improvement of Angina:
Beta blocker Associated Factors:
▪ Inadequate dosage
o 25 mg/day PO – for initial 1 week
o 100 mg/day PO – from 2nd week onwards
o Some geriatric patients may need up to 200 mg/day for better symptom control.
▪ Abrupt Withdrawal:
o Frequent exacerbtaion of angina can be due to acute coronary insufficiency. Abrupt beta-
blocker withdrawal may worsen the intensity and frequency of anginal episodes by
promoting vasospasm.
o Re-initiation of titrated dose beta-blocker combined with nitrates may bring good
outcomes.
o Heart failure – pre-existing heart failure may get worsened with beta-blocker usage due to
pre-existing or new onset poor LVEF
Other Factors:
• Inadequate anti-ischaemic prescription
• Senile vascular changes.
• Uncontrolled HTN
• Comorbidities, e.g., DM, cerebrovascular events
• Hyperlipidaemia
• Smoking and alcoholism
• Obesity
• Sedentary lifestyle
• Stressors and anxiety.
• Anaemia
• Haemodynamic instability
• New or worsening MR, valvular AS
• Sustained VT
• Frequent use of NSAIDs and COX II inhibitors
SIGNIFICANT TOPICS WITH SAQs 360
208.
A 60 years old patient has sudden onset of chest pain and heaviness along with sweating for the last 45
minutes. He is hypertensive. He has a history of chronic smoking. On examination, patient is profusely
sweating and looks very anxious. His BP is 100/70 mmHg.
ANSWER
A
Diagnosis:
Acute coronary syndrome – acute MI
B
Immediate Treatment:
Quick brief history, risk factor stratification and examination assessment.
Activating ambulance and coronary care unit facility of nearest tertiary care hospital.
ECG stat (look for features of ischaemia, STEMI, NSTEMI)
BP, heart rate, SaO2 monitoring
IV access + draw samples for;
o Trop T&I
o BSR
o FBC
o U/E
o S. creatinine
o BUN
Administer supplementary intranasal O2 (@4-6 L/min). If SaO2 < 94%
Immediate medication
o Aspirin stat 300 mg PO chewable, non-GI protective
o Clopidogrel 300 mg PO
o GTN to be avoided as patient has low BP currently.
o Inj. Diamorphine 2.5 to 5 mg IV over 1-2 minutes (repeat if necessary – maximum 5-
10 mg IV)
o In case of unavailability of morphine, opioid analgesic, e.g., Inj. Tramadol 100 mg IV
stat.
o Inj. Metoclopramide 10 mg IV stat
C
Further Management Plan:
Immediate shifting to tertiary care with high supervision and resuscitation facilities is
mandatory.
Goal is to provide fibrinolysis within 30 minutes of onset of symptoms or PCI within 90 minutes
of onset of symptoms.
En route:
o GTN IV infusion if pain persists.
o Initiation of beta blockers (atenolol 25 to 100 mg PO)
SIGNIFICANT TOPICS WITH SAQs 361
209.
A 45 year old banker presents for checkup. He is asymptomatic but has a BP of 160/96 mmHg.
ANSWER
Evaluation:
Evaluation of patient will need detailed history and examination with investigations.
History:
➢ Is this your first raised reading of BP. Have you ever checked your BP at home, was it normal ?
➢ Is your BP always on higher side when taken in medical setting (hospital or clinic) to rule out
white coat hypertension. Patient may have undiagnosed hypertension for years without
having had the BP checked, so I would like to ask questions to rule out end organ damage.
➢ Any h/o angina, MI, cardiac failure, h/o CKD, peripheral vascular disease or retinopathy.
➢ To rule out secondary causes, I will ask h/o renal disease, abdominal masses, anaemia, h/o
sweating, palpitations (phaechromocytoma), h/o thyroid problems, sleep apnoea, renal artery
stenosis.
➢ Medication history: Over the counter or herbal medications.
➢ H/O lifestyle – h/o smoking/alcohol exercise + physical activity, dietary preferences, salt intake.
➢ Psychosocial stresses
➢ Socioeconomic history: Job, any stress at job or at home.
Examination:
➢ Vital Signs: Pulse, BP, Height/Weight (BMI)
➢ GPE:
o JVP
o Carotid bruits
o Peripheral oedema
o Palpation of all pulses: Check for;
▪ Radiofemoral delay
▪ Coarctation of aorta
▪ Weak or absent pulses
▪ Severe peripheral vascular disease.
▪ Thyroid
▪ Evidence of endocrine disorder
o CVS
▪ Heart rate, rhythm
▪ Apex beat
▪ Murmur
▪ S3 and S4 sounds
▪ Look for evidence of LVF
o Abdomen: Renal arery bruit in upper abdomen (renal artery stenosis)
o Fundoscopy: To look at fundi for;
▪ Haemorrhage
▪ AV mapping
▪ Exudates
SIGNIFICANT TOPICS WITH SAQs 363
▪ Papilloedema
Investigations: I would like to advise investigations. Aim is to identify target organ damage.
➢ Blood glucose/HbA1c
➢ Lipid profile
➢ Serum creatinine + sodium
➢ CBC – haematocrit
➢ Urinalysis: Dipstick for RBC and protein. Lab sample for albumin/creatinine ratio.
➢ ECG
➢ Echocardiography (if LVH is suspected)
Management:
ABPM/HBPM: Patient’s BP is 160/96. If not known to be hypertensive, I would like to offer
ABPM/HBPM.
Patient’s Education: Education of patient about his condition + address his concerns, reassurance
and motivation of patient why treating his BP is important.
Provide verbal + written information.
Disease treatment option. Howe to take medicinces + education about medications side
effects.
Lifestyle Advices:
Encourage regular exercise dynamic is best, e.g., walking, swimming, cycling at least 30
minutes walk 5 days per week.
Diet:
o Lower dietary salt < 6 g salt per day
o Increase dietary fruits + vegetables.
o Total fat intake 30% or less, avoid saturated + trans fats
o Lower refined sugar use
o Fish twice a week.
o Reduced excess coffee + caffeine rich products
Lose weight to optimum for height.
If smoker – advise smoking cessation + help
Encourage relaxation + stress management.
Pharmacological Treatment:
If patient’s BP after ABPM/HBPM is daytime average > 150/90, I will start antihypertensives as
BP should be treated if > 140/90 as per JNC-8
Tablet lisinopril (5 mg) OD (ARB – if ACE not tolerated)
Follow-up: After starting treatment review at 2 weeks.
If BP is controlled: Review at 3, then every 3-6 months.
If BP is not controlled: ABPM/HBPM – not to alter medication or single reading.
If high BP sustaiend, need to alter in medicine.
Format of Annual Review:
• Check BP + look for signs of end organ damage.
• Urine for proteinuria
• Assess and treat modifiable risk factors of CHD/CVA
• Discuss symptoms + medications
• Reinform lifestyle changes.
SIGNIFICANT TOPICS WITH SAQs 364
210.
Mr. Shafiq, a 40 years old smoker has had persistent cough for the last two months. The cough is dry
and irritating. The usual cough medicines have been of no help.
ANSWER
Diagnosis and Management
Detailed History:
➢ Cough:
o Severity, frequency
o Continuous or intermittent (morning bouts/night)
o As history says dry cough (otherwise could ask for sputum)
o Any h/o haemoptysis
o Any wheeze
o Any SOB, chest pain, fever
o Any h/o weight loss, decreased appetite, night sweats
o Any h/o allergy
o Urinary/bowel habits
o Seasonal variation of symptoms.
o Any h/o common cold
➢ Family h/o TB, asthma, COPD, CA lung.
➢ Drug history – ACE inhibitor use.
➢ Social History:
o How many cigarettes per day
o Job
Physical Examination:
o Vitals: Imp RR, pulse oximetry (SaO2 > 94% normal)
o GPE:
o Edema (ankle edema may suggest CHF)
o Peripheral/cyanosis
o Clubbing (positive in COPD, CA lung)
o Anaemia (chronic hypoxia may lead o anaemia)
o Chest Examination:
o Check for any deformity
o Pattern of breathing
o Auscultate any added sound, rhonchi, crepts, wheezes
o Percussion: Any dullness
o CVS: S1+S2, check for murmur or added sound.
o HEENT: Any lymphadenopathy.
Investigations:
✓ Chest x-ray
✓ CBC + ESR
✓ PFTs
✓ Sputum for AFB smear
SIGNIFICANT TOPICS WITH SAQs 365
211.
A 35 year old asthmatic, a lecturer by profession, attends your clinic. He is using Ventolin tablets in sub-
therapeutic doses and has been in and out of hospital, several times for exacerbation of asthma. He is
averse to using inhaler and smokes 20 cigarettes daily.
ANSWER
A
Management Plan:
Reassurance
Explore and address patient’s ideas, concerns and expectations
Extensive Education:
• Asking the patient about his knowledge for the disease, its progression and effects of non-
adherence to treatment plan on disease prognosis.
• Consider any factor (physical or learning disabilities in terms of language understanding,
sight or hearing problems), which may affect patient’s hearing and involvement in
consultation
• Address patient’s concerns about medicines (including concerns about experiencing
dependancy, adverse effects of medicines, likeliness of opting only the non-
pharmacological ways, non-adherence due to multiple dosage, cost issues or only the non-
serious attitude of patient towards treatment.
• Check if patient technique of inhaler usage written in native language) on adverse effects of
non-compliance to treatment plan and how it can worsen disease prognosis.
• Counseling the patient that adherence to control plan will aim at 2 things.
o Reduction/elimination of disturbing symptoms of asthma, which can improve
patient’s quality of life.
o Minimizing future exacerbation, frequent hospital visits and hospital admissions
Non-Pharmacological:
Identify the triggers and avoiding the triggers
Smoking cessation – encourage enrollment in smoking control programs
Weight reduction
Avoidance of infections by yearly influenza vaccination and treating LRTI vigorously without
any delay
Maintaining self hygiene
Use of face masks particularly when outside the house/working place to avoid air pollution
Pharmacological:
Advised afer assessing asthma severity and previous treatment plan.
Rescue Therapy: To minimize acute symptoms.
o SABA salbutamol inhaler 200 mcg PO PRN 4-6 hourly with spacer device
Controller therapy: To prevent future exacerbations:
o ICS fluticasone inhaler 250 mcg BID for 2 months
o Accurate inhaler usage technique must also be taught to the patient.
▪ Dry Powder Inhalation DPI
▪ Budesonide+Salmeterol 200mg/400mg BD
SIGNIFICANT TOPICS WITH SAQs 367
▪ Tiotropium 18 mcg OD
▪ Capsule to be placed in the device and patient is instructed the technique to
suck the contents of capsule.
Self Management of Asthma:
o Written personalized asthma action plan (PAAPs) about recognizing loss of asthma
control assessed by symptoms, peak flow or both must also be provided to the
patient
Follow-up: After 4 weeks.
Urgent Referral: In case of worsening of symptoms or severity despite treatment.
SIGNIFICANT TOPICS WITH SAQs 368
212.
A 46 years old man, driver by profession, presents with 2 months history of low grade fever, cough with
white sputum, reduced appetite and weight loss. On examination, he appears ill, has a temperature of
99.4 F and bronchial breath sounds are heard in the right upper lobe. He has been treated with Amoxil
& Kalaracid but with no improvement.
ANSWER
A
Provisional Diagnosis:
Tuberculosis (pulmonary)
B
Investigations:
a) Sputum for AFB
b).Chest X-Ray
c).CBC with ESR
d).Chest CT (to rule out renal carcinoma with pulmonary mets if chest X-ray comes negative with
negative smear)
C
Management: Treatment is according to cause.
Non-Pharmacological:
Reassure patient and explain to him about nature of disease.
General advice to prevent spread of disease to other people living and working with him like
using mask while at work avoiding spitting in public places.
Improvement of general hygiene.
Inform him about treatment details and duration of treatment.
Give him information about side effects of medicines and provide him written details about his
disease and treatment.
Advise him about drug compliance
Pharmacological:
Treatment is according to cause. If patient is diagnosed as tuberculosis ATT is given for period
of 6 months according to weight of patient.following drug doses are given:
o Pyrazinamide 25mg per kg od PO
o Ethambutol 20mg/kg od
o Streptomycin 15mg/kg od
o Rifampicin 10mg/kg od
o Isoniazid 5mg/kg od.
o Pyrazinamide, rifampicin, ethambutol and isoniazid are given for first 2 months, then
Isoniazid and rifampicin are given for next 4 months.
If patient is diagnosed as a case of carcinoma, he will be referred to specialist (oncologist) for
further management).
SIGNIFICANT TOPICS WITH SAQs 369
213.
Your next patient, Mrs. Farhat Waheed, a 58 year old housewife comes for the result of her recent chest
x-ray. For the last six months she has had a cough and more recently hemoptysis. The report shows a
right hilar mass and a shadow in the right upper zone, consistent with a bronchial carcinoma. As a first
step in management, you have to inform the patient of the probable diagnosis.
ANSWER
A
A doctor in his clinic or in emergency been there to provide best services and treatment to the
patient.
Keeping view above patient following issues has to discuss.
1)communication issues
Patient and doctor often find it difficult to discuss serious ilness like cancer when death is
inevitable.
It creates major barrier to establishing patient centered ,goal directed care plans.
Pt need range from basic information about his prognosis and what symptoms to expect to help
with complex issues such as unresolved family coflicts or making arrangements.
2)Advance care planning.
Important for patient to complete advance care directive form it will help in terminal disease
crises so it will be easy by family that patient choose treatment option by himself.
3)Breaking Bad news.
Physician who is skilled and comfortable at breaking bad news will take better decission during
treatment.
3)Psychosocial issues.
Sensitivity of psychosocial needs of payient.
4)Spiritual issues.
This spritual need support patient to praying with and for patient.
5)Family issues.
Families often need financial support,spiritual and psycological counselling.
Patient care by family ie bathing,toileting,mobility and dressing.
》Conclusion.
Medical profession truly dedicated for sick, best available care at the time of greatest need is an
integral role of family
Physician.
SIGNIFICANT TOPICS WITH SAQs 370
214.
A 30 year old gentleman with no comorbids has come with myalgias, low grade fever, and dry
hacking cough for a week. His throat appears normal and his chest has no major findings. His X-
ray chest shows scattered infiltrates at the left base.
ANSWER
A
Most Likely Diagnosis:
Atypical Pneumonia
B
Management:
Detailed History: I 'll take detailed history including:
➢ h/o of cough,is it persistent and comes in fits throughout the day.
➢ h/o sore,dry itchy throat.
➢ h/o chest pain and does it get worse when he takes breathe or cough?
➢ h/o headache?
➢ loss of appetite,low energy and fatigue.
➢ h/o muscles pain(already in history),joints stiffness or sweating.
➢ h/o smoking
Examination:
o Vital signs, level of consciousness, pallor, skin rashes.
o Chest examination for respiratory rate, crepts and breath sounds.
o Systemic examination to rule out any other cause
Investigations:
✓ CBC
✓ CXR (already done
✓ Sputum C&S
✓ Urinary antigen for Legionella
Non-Pharmacological Treatment:
reassure the patient,address his ICE.
explain to him about nature of disease and its treatment.
advice him to drink plenty of water
Take adequate rest
Avoid physical and mental stress.
Take healthy diet and stop smoking(if smoker).
Pharmacological Treatment:
Amoxicillin 500 mg TDS
Clarithromycin 500 mg BD
Antipyretic + analgesia
Referral: To hospital if;
• Hypotension
• RR above 30 BPM
SIGNIFICANT TOPICS WITH SAQs 371
• Confusion
SIGNIFICANT TOPICS WITH SAQs 372
215.
A 2 year old child with a 2 days history of fever is brought to your clinic by her mother from a poor
locality. The child has had cough with rapid breathing.
ANSWER
A
Likely Diagnoses:
❖ Pneumonia
❖ Bronchiolitis/croup (viral infections)
B
Further Points in History:
➢ Fever: How long has fever, is it intermittent, is it associated with rigors or chills; more in
the night; fever > 38 C or 100.4 F
➢ Cough: Is it dry or productive; any wheeze or stridor
➢ Nasal discharge: If nose is stuffed or there is nasal discharge.
➢ Breathing: is shallow or rapid
➢ Any vomiting associated with cough.
➢ Frequency of urination
➢ Appetite if decreased
➢ Any rash on the body
➢ h/o RTI in family (mother, etc.)
➢ Developmental milestones and immunization
C
Clinical Features to Consider Prior to Referral:
1) R/R ≥ 40 per minute
2) Cyanosis
3) Anaemia
4) Flaring of ala of nose
5) Chest indrawing
6) Signs of dehydration, anterior fontanelle/capillary refill
7) Severe distress
8) Not able to drink
9) Stridor
10) Pneumonia – follow up in 2 days
a. Syr. Amoxil 50 mg/kg/day
b. Syr. PCM 10 mg/kg/dose
c. Severe pneumonia – refer
11) Bronchiolitis/croup (viral infection): Treatment depends upon severity of symptoms
a. -oral fluids
b. -oral antipyretic( e.g. paracetamol suspension)
SIGNIFICANT TOPICS WITH SAQs 373
216.
List essential diagnostic equipment and accessory articles to keep in a “Doctor’s Bag” that you would
carry as a Family Physician on house visits.
ANSWER
Diagnostic Equipment:
• Stethoscope and pocket diagnostic set.
• Sphygmomanometer and infrared thermometers.
• Pulse oximeter
• Glucometer including appropriate strips and lancets.
• Alcohol gel for hands.
• Additional sphygmomanometer cuffs.
• Reflex hammer
• Urine dipsticks
• Tongue depressors, preferably wrapped
• Small torch
• Peak flow meter, preferably low reading
• Specimen bottles (urine/faeces) and swabs.
• Otoscope
• Ophthalmoscope
• Tourniquet, syringes and needles.
• Vaginal speculum +/- sponge forceps
• Fetal stethoscope/Doppler
• Fluorescein sticks (to detect damage to cornea)
• Airways
• The two-way Resuscitube
• Laerdal pocket mask
• Syringes, 2, 5, 10 ml; needles 19, 21, 23, 25 gauge; IV cannulae 16, 18, 20 gauge
Accessory Articles
• Photocard ID
• Cellphone + charger
• Controlled drugs record book
• Envelopes
• Headed notepaper
• Local map
• Pathology/X-ray form
• Referral Forms and Hospital Admission Forms
• Medical Charts
• Prescription pad
• Sick leave certificates
• List of useful telephone numbers.
• BNF/MIMS
SIGNIFICANT TOPICS WITH SAQs 375
• Quick reference text, e.g., Oxford Handbook of General Practice and dangerour drugs
record book
• Obstetric calculator
• Peak flow chart/wheel
• Book for keeping a record of patient encounters
• Temporary resident records
• List of local chemists and out of home opening times
• Small amount of change for parking
SIGNIFICANT TOPICS WITH SAQs 376
217.
A 5-year-old boy, Bilal, is brought to you by his mother who has 102 F fever for 4 days with watery eyes,
runny nose and cough. This morning, mother noticed fine rash all over the body, but more marked on
the face, forehead and extremities. She says Bilal has had complete regular vaccinations. On
examination, the child is bleary red, the throat is red, the nasal secretions are clear mucoid and the
lungs are clear. The rash is maculopapular.
ANSWER
A
Most Likely Diagnosis:
Measles
B
Relevant Questions From Mother
Detailed History: To assess severity level. Ask;
▪ Is child taking feeds
▪ Any h/o seizure (febrile)
▪ Any h/o shortness of breath
▪ Any ear infection
▪ Any h/o cough with high grade fever
▪ Any h/o drowsiness, convulsions.
▪ Any h/o contact with a person with similar condition.
▪ Any immunosuppresed contact at home.
▪ Immunization history
▪ Birth +/- developmental + nutritional history.
C
Management
History: As above.
Examination:
• Vital Signs: Pulse, RR, temperature.
• GPE: Pallor, jaundice, assessment of hydration level to rule out dehydration. Is child active
or lethargic, sleepy or irritable.
• Mouth Exam: Koplick spots in oral cavity.
• Examination of rash – generalized maculopapular
• HEENT examination
• Chest: breath sounds, any signs of consolidation, chest indrawing
Laboratory Testing
✓ Measles IgM antibodies & IgG serology (ELISA) (serum or oral fluid samples) OR
✓ Isolation of measles virus RNA from respiratory specimens OR
✓ Salivary measles – specific IgA testing OR
✓ Antigen detection by fluorescent antibody or PCR techniques
Non-Pharmacological Treatment:
SIGNIFICANT TOPICS WITH SAQs 377
Discuss with mother the disease course of her child, its contagious nature and address ICE
(explain complications and signs of serious illness)
Mother’s counselling about importance of immunization and explore and reassess hat why
her child is not immunized and information that unimmunized children are at risk of
developing complications.
Treatment is supporetive.
Plenty of fluids to prevent dehydration.
Usually, it resolves in 7-10 days
Avoid passive smoking.
If crustiness of eyes, gently clean with damp warm cloth.
Pharmacological Treatment:
Syr. Acetaminophen 10-15 mg/kg 4-6 hourly.
Vitamin A supplements (200,000 IU) for 2 days + 3rd dose 2-4 weeks apart.
Saline nasal drops if nasal congestion.
Antibiotics if there is secondary infection.
Follow-up: 1 week.
Safety Netting
If patient’s condition does not improve or develops breathing difficulty, drowsiness or
persistent high fever, take your child to A&E.
Referral of child to EPI for immunization according to his age after patient gets better.
Red Flags: Spotting Signs of Serious Illness:
Explain to mother if child condition does not improve and she observes any signs like;
o Shortness of breath
o Chest pain
o Coughing up blood
o Drowsiness
o Confusion
o Fits
To go to nearest hospital
SIGNIFICANT TOPICS WITH SAQs 378
218
Mother of an 8-year-old child is worried about rapid hair loss and a scaly rash on scalp of her child for
last 2 months. On examination, his weight is 22 kg and this is the appearance
ANSWER
A
Provisional Diagnosis:
Tenia Capitis
B
Further Points in History and Physical Examination:
History:
➢ Any pet animals at home (as caused by M. Canis acquired from dogs and cats)
➢ Is there any itching
➢ Is the hair loss significant (to rule out seborrheic dermatitis)
➢ Is there complete alopecia or patchy areas of no hair (to rule out alopecia areata)
➢ Any tight braiding of hair (to rule out traction alopecia)
➢ Does the child pull out his own hair (r/o trichotillomania)
➢ Extent of scaling (more in psoriasis
Examination:
o Look for typical features suggestive of scalp ringworm, for example:
o Scalp scaling.
o Patchy, irregular hair loss.
o Lymphadenopathy (postauricular and cervical).
o Itch.
o In some people, a more severe inflammatory reaction can occur, causing:
o Erythema.
o Pustules.
o Crusting.
o Pustular boggy masses (kerion).
SIGNIFICANT TOPICS WITH SAQs 379
219
An 18-year-old boy visits your clinic with his mother. He has productive cough for the last 6-8 weeks.
Two days back, he had an episode of hemoptysis with a teaspoon of fresh red blood in sputum. On
examination, his temperature is 99.6 F and his weight is 48 kg
ANSWER
A
Provisional Diagnosis
Pulmonary tuberculosis
B
Further Points in History?
➢ Productive cough 6 to 8 weeks along with recent episode of haemoptysis along with low grade
fever are the clues of the pulmonary T.B.
➢ But for the confirmation of my diagnosis i also ask some more questions from the patient and
take the detailed history as
o ask about the details of cough its onset,association aggravating and relieving factors
associations such as vomiting, nausea, headache, weight loss, weakness, shortness of
breath, chest pain.
o ask about the haemoptysis its quantity,nature as it is copious or not any gum bleeding or
mouth infection
o history of fever (by the formula of SOCRATES) severity onset characteristics association
timing exacerbating relieving factors
o Past medical history
o Ask the complete family history personal history any other family members having the
same problem or not ,is he taking some medicines or on vaccinations or any other therapy
o Socioeconomic history
▪ Any contact of person with similar symptoms
▪ Any immunosuppressed contacts
▪ Way of living
▪ H/O personal hygiene, ventilation at home.
C
Investigations
✓ CBC ESR (PT APTT for the coagulation profile as it might be disturbed in some hemorrhagic
fevers like in congo & Ebola fevers Rarely in Pak & Saudia)
✓ Order a fresh chest X ray PA view with at inspiration.
✓ then sputum for AFB smear (3 consecutive samples sent in early morning)
✓ tuberculin skin test (TST) by Mantoux test, heaf secreaning or by time screening
✓ if pt can afford and easily accessible then advise Gene expert for T.B( if still in suspicion)
✓ Interferon gamma release Assay (IGRA)
D
Prescription:
SIGNIFICANT TOPICS WITH SAQs 381
As in question this is mentioned that write a prescription so no need to tell the non
pharmacological measures
Just write that with the instructions of DOTs therapy and preventive instructions i ll give
P E St R I (pastry)
P. pyrazinamide 25mg / kg
E. Ethambutol 2.0mg / kg
St. Streptomycin 1.5 mg/kg
R. Rifampicin 1.0 mg /kg
I. Isoniazid 0.5 Mg/kg
That must b initial phase threapy for 8 weeks then
3 drug threapy for 18 months having R.I.P (rest in peace )
R. Rifampicin
I. Isoniazid
P. Pyrazinamide
Also add vit B6 continuly from day 1 till 6 months threapy.
PRESCRIPTION
Name: XYZ Date: DD/MM/YEAR
AGE: 18 years Diagnosis:
Weight: 48 kg
Calculated Dose
INH (Isoniazid) 300 mg 1+0+0 240 mg
Rifampicin 450 mg 1+0+0 480 mg
Ethambutol 400 mg 2+0+0 720 mg
PZA (pyrazinamide) 500 mg 2+0+0 960 mg
Pyridoxine 50 mg 1+0+0
Explain all possible side effects of above medicines to patient’s mother. If any one of the side
effects appear, then bring patient to clinic or nearest hospital.
SIGNIFICANT TOPICS WITH SAQs 382
220.
A mother brings her 18-year-old son in your clinic with symptoms of irritability, insomnia, nightmares
and flashbacks. His symptoms started 6-7 months back after he was kidnapped for ransom and later on
released.
ANSWER
A
Points in History:
1) Having recurrent nightmares
2) Avoiding people, places, situations that remind you of trauma
3) Having difficult sleep
4) Having difficult time remembering important points of traumatic event.
5) Frequently upsetting thoughts/memories
6) Any drug history – alcohol etc.
B
Most Probable Diagnosis:
Posttraumatic stress disorder (PTSD)
C
Management:
Non-Pharmacological
Reassure the patient
Explore ideas and concerns
CBT
Eye movement desensitization and reprocessing (EMDR)
Pharmacological
Antidepressant – SSRI, SNRI (some benefit)
221.
A 44-year-old married lady presents to your clinic complaining of heavy menstrual cycle.
ANSWER
A
History Questions:
1) Age
2) Area of residence (Afro-Caribbean are at increased risk of fibroids)
3) Presenting complaint, i.e., menorrhagia (duration of complaint)
4) Menstrual history: Age of menarche, duration of cycle, regular/irregular cycles, number of
sanitary towels, LMP, postcoital/intermenstrual bleeding, passing any clots, dysmenorrhoea
5) Gynaecological History:
a. Vaginal discharge
b. History of Pap smear if any
6) Obstetrical history: Parity, abortions, any complications.
7) Associated Symptoms
a. Feeling tired (anaemia)
b. Urinary symptoms (large fibroid)
c. Bowel symptoms (large fibroid)
d. History of hirsuitism
8) History of contraception
9) Any history of bleeding dicharge
10) Thyroid problem
11) Personal or family history of endometrial or cervical cancer.
B
Examination:
Vital Signs: BP (hypotension), pulse (tachycardia), Temp, RR
GPE: Pallor, dehydration
Systemic:
1) Abdominal examination for hepatic/renal
2) Pelvic examination for cervical erosions, polyp, fibroids.
3) Thyroid examination
C
List of Investigations:
1) Pregnancy Test: To rule out pregnancy.
2) CBC for anaemia
3) LFTs, TFTs, serum prolactin
4) Clotting profile to know any bleeding diathesis.
5) Pelvic ultrasound
6) Serum testosterone if there is any history of hirsuitism/virilization
SIGNIFICANT TOPICS WITH SAQs 384
D
Management Plan:
History: Age, area, menstrual, gynae, obs history with associated symptoms
Examination:
1) GPE: Pallor, dehydration
2) Systemic:
a. Pelvic examination
b. Abdominal examination
c. Thyroid examination
Investigations:
CBC, LFTs, TFTs, serum prolactin, pregnancy test, USG abdomen
Treatment Plan: Tranexemic acid can be added.
1) Iron therapy: Tablet ferrous sulphate 190/300 one PO OD
2) NSAIDs, i.e., diclofenac sodium 25 mg 1 PO OD. It can be combined with PPIs to reduce
gastric problem.
3) Combined OCPs, i.e., Diane 35. I will explain how to use Diane -35.
a. Start at 5th day of menstrual cycle and use it with _____.
b. Take tablet daily for 21 days
c. 7 days gap days or you can take iron in 7 days and wait for periods.
4) For further management, gynae referral will be done. Management includes
a. Medroxyprogesterone Injection
b. LNG – IOS
c. Danazol
d. GnRH
5) If refractory to medical treatment, surgical intervention may be done, i.e.,
a. D&C
b. Endometrial ablation
c. Hysterectomy
SIGNIFICANT TOPICS WITH SAQs 385
222.
A 5-year-old girl presents with weakness and anorexia for the last one year. She also tells you that
sometimes she passes small white worms in her stool. On examination, she is pale. Rest of history and
examination are normal.
ANSWER
A
Cost-Effective Investigations:
1) Blood – CBC (anaemia)
2) Tape test/Scotch Test
3) Stool D/R
B
Advise to Mother:
1) Regular handwashing
2) Routine household cleaning
3) Frequent changing of clothes, towels and bed liners
4) Discuss about complications of untreated pinworms like weight loss, bedwetting, diarrhea,
UTI, possibly appendicitis
5) Medications:
a. Mebendazole (Vermox) 100 mg tab or 100 mg in 5 ml suspension (once in a single
dose or twice each day for 3 days OR
b. Albendazole 400 mg PO OD x 2 doses 2 weeks apart
c. Iron 6 m/kg/day for six months for anaemia
d. Vitamins – Syr. B complex, vitamin A, vitamin D
C
Prevention:
1) Remind kids to wash their hands often especially after using toilet, after playing outside and
before eating.
2) Make sure your kid showers by bath every day.
3) Keep kids’ fingernails short and clean
4) Tell kids not to scratch around their bottom or bite nails.
5) Wash your kids’ shirt and trousers every few days.
6) Keep hygiene
7) Entire household be treated regardless of symptoms.
SIGNIFICANT TOPICS WITH SAQs 386
223.
A 28-year-old man is brought to your clinic with history of burns 2 hours back as his clothes caught fire.
On examination, he appears in pain. He has redness and blisters on whole of left arm and chest.
ANSWER
A
History Points:
1) Exact Mechanism
a. Type of burn agent (scald, flame, electrical, chemical)
b. How did it come into contact
c. Is there risk of concomitant injuries.
d. Is there risk of inhalational injuries.
e. What first aid performed
f. Has any treatment started.
2) Exact Timings
a. When did injury occur
b. Duration of cooling applied
c. Duration of energy source exposure to patient.
3) Exact Injury:
a. What was liquid boiling/recently boiled
b. Domestic/Industrial
4) Suspicion of non-accidental injury
B
Assessment of Extent of Injury:
1) Cause, size, thickness of burn
2) Rule of nines to estimate extent of burn
3) Appearance:
a. Partial thickness burns – red, painful, blistered.
b. Full thickness burns – painless, white/grey
4) Always consider non-accidental injury in children
C
Management:
1) Remove clothing from affected area
2) Place patient under cold running water for > 10 minutes until pain relieved
3) Do not burst blisters
4) Prescribe analgesia
5) Refer all but the smallest (< 5%) partial thickness burns for assessment in A&E
6) Referral if inhaled smoke.
7) IF MANAGING BURN IN COMMUNITY
a. Check tetanus immunity – given immunization +/- prophylaxis
SIGNIFICANT TOPICS WITH SAQs 387
224.
A 30-year-old banker presents with fresh bleeding per rectum with hard stools for the last one week.
The blood is small in amount and there is severe rectal pain at the time of defecation. He mostly eats
outside and loves fast food.
ANSWER
A
Most Likely Diagnosis:
Anal fissure with constipation
B
Clinical Examination:
Vital Signs: Pulse, BP, respiratory rate, temperature
GPE: Look for anaemia, jaundice, dehydration, skin rash, thyroid, cyanosis, clubbing, height,
weight, etc.
Abdominal Examination: Inspection, palpation (tenderness/distension, visceromegaly) and bowel
sounds
Rectal Examination: Do not attempt as it is painful due to muscle spasm (DRE)
• On inspection, anal mucosa is torn – usually on posterior aspect of anal canal.
• The fissure is often visible as a sentinel pile (bunched up mucosa over the base of tear)
C
Steps of Pharmacological Plan:
Stool softener (isphagol husk) 2 tablespoons in 1 glass of warm water BD
0.4% nitroglycerine ointment BD
Syr. Lactulose 30 ml HS
Analgesic suppositories
o 5% lidocaine ointment
o OTC haemorrhoid preparations if nitroglycerine not available.
o 2% topical diltiazem (third lin)
Follow up in 1 week.
If these interventions fail, refer to surgeon.
D
Steps of Non-Pharmacological Plan:
Reassure the patient
Increase fluid intake to 8-10 glasses of water per day.
Exercise
High fiber diet
Sitz bath
Increase fruits and vegetables in diet.
SIGNIFICANT TOPICS WITH SAQs 389
225.
A 55-year-old man, hypertensive for last 2 years, presents to your clinic with a 3-month history of
breathlessness. He does not report any chest pain. He is currently on atenolol. His BP is 180/100
mmHg, pulse is 90/min and there are bilateral crepts at lung bases.
ANSWER
A
Likely Diagnosis:
Heart failure (pulmonary edema)
B
Investigations:
1) Full blood count
2) U&E
3) BNP (brain natriuretic peptide)
4) Chest x-ray (to see cardiomegaly, cardiothoracic ratio > 50%, pleural effusion, Kerley’s B
lines, bats wing shadowing
5) ECG: May indicate ischemic MI or ventricular hypertrophy
6) Echocardiography: Key investigation.
a. May indicate MI or any valvular heart disease
b. Can confirm presence/absence of LV dysfunction
C
Medicines to Add
1) ACE inhibitor, i.e. perindopril 10 mg 1 PO OD
2) Diuretics, i.e. furosemide 40 mg PO OD (to reduce pulmonary edema)
D
Non-Pharmacological
1) Eat less salt, i.e. < 6 g/day
2) Encourage exercise, i.e., moderate to intense exercie at least 3-4 days a week.
3) DASH diet.
4) Optimize weight.
SIGNIFICANT TOPICS WITH SAQs 390
226.
A 45-year-old gentleman presents for his annual checkup. He is a smoker and his labs are as follows:
• Fasting blood sugar 116 mg/dl
• Total cholesterol 210 mg/dl
• Serum creatinine 80 mg/dl
Urine D/R
• Glucose Nil
• Proteins Nil
• Bacterias Few
• Leukocytes 2-4/HPF
On examination, his BMI is 32 kg/m2, BP is 140/94 mmHg and his waist circumference is 102 cm.
A. List down the risk factors for cardiovascular disease in this case.
B. What further investigations would you need to manage his condition and why?
C. What is the non-pharmacological management of this patient?
ANSWER
A
Risk Factors for CVD:
1) Smoker
2) Age 45 years
3) High cholesterol (>130), i.e. 210
4) FBS: 116 (normal 80-100)
5) BMI: 32 (normal 18-25)
6) BP: 140/94 (First-stage hypertension)
7) Waist circumference: 102 cm (obesity)
B
Further Investigations:
1) FBC – to rule out anaemia (in chronic renal insufficiency)
2) Chest x-ray PA view (to see any pathology in heart)
3) RBC, HbA1c (to rule out diabetes)
4) Thyroid profile
5) Serum electrolytes, to see level of Na and K, Ca (to exclude renal damage secondary cause
such as aldosteronism)
6) Renal ultrasound (to rule renal artery stenosis)
7) Urinary free cholesterol, reninin, aldosterone (secondary causes of HTN)
8) ECG + echocardiography (LV hypertrophy)
9) Serum triglycerides
10) Total Lipid profile – To check HDL, LDL, cholesterol, triglycerides level, so as hyperlipidaemia
might be managed accordingly.
C
Non-Pharmacological Management:
1) Smoking cessation.
2) Reduce salt (<2.4 g/day)
3) Blood pressure monitoring at home.
4) Mediterranean diet, i.e. low fat diet. Reduce intake of saturated fats
SIGNIFICANT TOPICS WITH SAQs 391
5) Increase exercise, i.e. encourage minimum of moderate to intense 3-4 times weekly.
6) Weight reduction to achieve BMI of 18-24.
7) Diet rich in fruits and vegetables. Avoid whole grains in diet
SIGNIFICANT TOPICS WITH SAQs 392
227.
A 35-year-old female presents with dysphagia to solid foods since the last 2 months. On examination,
her BMI is 18 kg/m2. She brings reports of her hemoglobin. Hb is 8 gm/dL with MCV of 60. Her
menstrual cycles are normal.
ANSWER
A
Differential Diagnosis:
1) Neuromuscular Diseases:
a. Achalasia
b. Parkinson’s disease
c. CVA
d. Diffuse esophageal spasm
e. Medication induced
2) Obstructive Lesion:
a. Esophageal CA
b. Esophageal webs (Plummer Vinson Syndrome)
c. Substernal thyroid
d. Mediastinal mass (anterior)
B
Investigations
1) Barium swallow studies and fluoroscopic evaluation: It will show esophageal lesions uch as
webs, strictures.
2) Serum Iron Studies: To see decreased levels of iron, ferrritin, and increased levels of total
iron binding capacity to see the cause of anaemia
3) Upon specialist/consultant advice:
a. Haemoglobin electrophoresis
b. Fiberoptic endoscopy
c. Oesophageal manometry
d. CT scan of neck, chest, abdomen
C
Management:
Management is on outpatient basis.
1) Educate the patient regarding the disease and treat anaemia by giving iron supplements
and also its underlying cause if there is any.
2) For further management, refer to gastroenterologist (for causative treatment for dilatation)
SIGNIFICANT TOPICS WITH SAQs 393
228.
A 55-year-old post menopausal woman (para 6+2) presents with history of urinary incontinence on
sneezing-coughing. She has no comorbids. Her weight is 80 kg height is 160 cm
ANSWER
A
Diagnosis:
Stress incontinence
B
Investigations:
a) Urinalysis
b) Empty supine stress test to detect intrinsic urethral sphincter dysfunction
c) Cough stress test
d) Postvoid residual measurement to measure postvoidal urine volume by ultrasound
e) Creating images of bladder as it functions
f) To be ordered by specialist:
a. Cystoscopy
b. Urodynamics
C
Management:
History:
1) Detailed history of incontinence
a. Duration
b. Sudden urgency, nocturia
c. Voiding difficulty
d. Constant passive leakage
e. Postvoid dribbling
f. Life activities affected
2) Associated symptoms – fever, constipation, burning micturition, lowr back pain, nausea,
vomiting
3) Comorbids – HTN, DM, hypo/hyperthyroidism, kidney issues, IHD
4) Past medical history – UTI, PID, CVA, etc.
5) Past surgical history – spinal surgery, pelvic surgery
6) Gynaecological/obstetrical history
7) Drug history – anxiolytics, diuretics, etc.
8) Lifestyle – exercise, diet, etc.
9) Family history – Ca, HTN, etc.
10) Psychological history – depression, anxiety.
Physical Examination:
1) Vital Signs: BP, pulse, RR, temperature.
2) GPE: Anaemia, jaundice, thyroid, lymphadenopathy
SIGNIFICANT TOPICS WITH SAQs 394
Non-Pharmacological:
Reassurance
Diet: Cut back caffeine and alcohol (as can irritate bladder). Add fiber, fruits and
vegetables.
Bladder training – schedule for toileting if mixed incontinence.
Lose weight
Kegel exercise/pelvic floor exercise
Exercise like walking 30 to 90 minutes 5-7 times per week (increase tim of slow walking)
Pharmacological:
Anticholinergics, i.e.,
o Oxybutynin
o Tolterodine
o Mirabegron
o Alpha blockers
o Topical estrogen
Devices:
o Urethral insert
o Vaginal pessaries
If it is not corrected with pharmacological, behavioral therapies, then interventional therapies and
surgery can be done (to improve closure of sphincter or support bladder neck).
Referral:
If prolapse
If cause unknown
If symptoms become worsened.
Follow-up:
In 4-6 weeks
Required long-term follow-ups up to 12 to 24 months.
SIGNIFICANT TOPICS WITH SAQs 395
229.
A 58-year-old businessman who has had a myocardial infarction followed by angioplasty presents to you
6 weeks after the event. He is still at bed rest and feels low and agitated. He has quit smoking after the
heart attack. He is on bisoprolol, aspirin, enalapril, atorvastatin. He does not have any cardiac
symptoms now.
230.
An 18-year-old female presents with seasonal nasal blockage, throat irritation, watery rhinorrhoea and
frequent sneezing since the last 2 years. These episodes occur 2-3 times a year, when the season
changes and last for about a month
ANSWER
A
Most Likely Diagnosis:
Seasonal allergic rhinitis
B
Clinical Examination Findings:
1. General Exam: Mouth versus nose breathing
2. Eyes: Swollen (periorbital), red conjunctiva, dark areas under eyes (allergic shiners)
3. Nose:
a. Horizontal nasal crease showing repeated rubbing with hand/palm
b. Nasal mucosa: red, swollen, mucosa of turbinate may be pale.
c. Thin, watery secretion
d. Rule out nasal polyp
e. Presence of nasal discharge blocking nares
4. Throat: Erythema, postnasal drip, any evidence of infection
5. Chest: Signs and symptoms suggestive of atopic disease, i.e., asthma (wheeze, rhonchi)
6. Skin: Signs of eczema (atopic) support diagnosis of allergic rhinitis
C
Management Plan:
1) History: Seasonal, watery rhinorrhoea, nasal blockage, frequent sneezing
2) Examination:
a. Vitals: BP, pulse, RR, temperature
b. GPE: Anaemia, jaundice, dehydration, lymph nodes, skin rash
c. Nose: Nasal discharge
d. Chest: May be any evidence of rhonchi
e. Skin: Any ectopic rash
3) Non-Pharmacologic:
a. Avoidance of allergens, i.e., dust, mite, pollens, smoking
b. Patient education about allergy
c. Nasal irrigation
4) Pharmacologic
a. Intranasal antihistamines, i.e., xylometazoline nasal spray 2 puffs BD (to relieve nasal
blockage) for 3 days
b. Intranasal corticosteroids as first line if moderate symptoms, i.e. mometasone nasal
spray 2 puffs OD
c. In severe cases, both intranasal steroids and oral/nasal antihistamines
SIGNIFICANT TOPICS WITH SAQs 397
5) Prevention
a. Allergen avoidance
b. Nasal irrigation
c. Probiotics
d. Herbal preparation
SIGNIFICANT TOPICS WITH SAQs 398
231.
A 32-year-old woman presents with a nodule in the anterior neck for the last 3 months. On
examination, the nodule is 3-4 cm, non-tender, in central area of neck and moves on swallowing.
ANSWER
A
History Points:
1) Details of swelling/nodule
a. When noticed
b. Increase/decrease in size with time
c. Pain during swelling
d. Shortness of breath
e. Cold intolerance
f. History of tremors
2) Associated Symptoms: Fever, shortness of breath, stridor, pain, chest pain, change in voice,
blurred vision, tremors, constipation/diarrhea, menstrual problems, sweating, weight
loss/increase, etc.
3) Comorbids: HTN, IHD, CVA, DM, asthma
4) Past medical history: Hypo/hyperthyroidism, TB
5) Lifestyle: Drug history, diet, family history of thyroid problems, exercise level.
B
Initial Investigations:
✓ USG neck
✓ Thyroid function tests (TSH, T3, T4)
✓ FBS, RBC, HbA1c
✓ Serum free testosterone (screening for PCOS)
C
Confirmatory Tests to Make Diagnosis
✓ Thyroid profile with TPO antibodies
✓ Thyroid antibodies
✓ Thyroid ultrasound
✓ CXR
✓ Refer to specialist for:
o Fine needle aspiration
o CT scan of thyroid
o Thyroid scan with low dose radioactive iodine
SIGNIFICANT TOPICS WITH SAQs 399
232.
A 40-year-old male, smoker, gets a screening checkup as his father died at the age of 40-year due to an
MI. His BMI is 29 kg/m2, BP is 140/90 mmHg.
His blood reports are as follows:
Hb: 17.2 mg/dL
Cholesterol 236 mg/dL
Triglycerides: 402 mg/dL
HDL: 28 mg/dL
LDL: 128 mg/dL
FBS: 102 mg/dL
Urine report: normal
Serum creatinine 1.2 mg/dL
ANSWER
A
Risk Factors for Heart Disease:
1) Positive family history of coronary artery disease.
2) Smoking
3) Overweight + obesity; BM 29 kg/m2.
4) High cholesterol 236 mg/dL (should be > 130 mg/dL)
5) HDL 23 mg/dL (should be > 35 mg/dL)
6) Stress or depression as his father’s early death with MI.
B
Advice for Patient:
1) Patient Education: Counseling and motivation of patient’s health lifestyle modification,
education and information about risks and complications
2) Smoking: Goal should be for complete cessation of smoking. Provide counselling, nicotine
replacement, bupropion.
3) Physical Activity: Encourage minimum of 30-40 minutes of moderate intensity 3-4 times
weekly physical activitiy (walking, jogging, swimming)
4) Weight Management: Ideal BMI 18.5 to 25 kg/m2. Start intensive diet and appropriate
physical activity intervention. Advise a diet rich in fruits and vegetables and low in
saturated fats.
5) Lipid Management:
a. Primary goals:
i. LDL < 100 mg/dL
ii. HDL > 35 mg/dL
iii. Triglycerides 200 mg/dL
b. Advise patient, he may need medication, physical activity + diet control to achieve
the goal.
C
Medications to Prescribe:
SIGNIFICANT TOPICS WITH SAQs 400
a) Tablet atorvastatin 20 mg OD at night after food. Educate patient about possible side
effects and follow up after three months to check lipid profile and LFTs.
b) Tablet Fenofibrate 145 mg OD
c) Omega-3 fatty acids (fish oils)
SIGNIFICANT TOPICS WITH SAQs 401
233.
A mother brings her 4-year-old daughter with chronic constipation since the last 6 months. The child
passes a stool every 3-4 days which is hard and difficult to pass. She drinks 3-4 glasses of milk in the day.
She has to use glycerin suppositories occasionally to relieve her symptoms. The child’s height and
weight are on the 25th centile for age.
ANSWER
A
Likely Causes of Constipation:
1) Idiopathic constipation
2) Hirschsprung’s disease
3) Coeliac disease
4) Hypothyroidism
5) Anorectal abnormalities
6) Neurological conditions
7) Abdominal tumours
8) Unhealthy/poor diet
B
Investigations to Order:
1) Blood – CBC (anaemia, infection)
2) Thyroid profile – very rare
3) Abdominal radiograph
4) Stool D/R, Culture and sensitivity.
5) Abdomino-pelvic ultrasound
C
Management:
History:
1) Ask about abnormal stool pattern (< 3 stools per week)
a. Hard, large, or rabbit-dropping stool
b. Overflow soiling (> 1 year)
c. Any worm in stool
2) Associated symptoms with defecation
a. Distress/anal pain
b. Bleeding with stool
c. Straining.
d. Poor appetite
e. Abdominal pain improves after stool passed
f. Posture indicating retaining stools
3) H/o previous constipation/anal fissure
4) Birth/developmental/immunization history
5) Family history – constipation, etc.
6) Socioeconomic history (diet, hygiene, any recent travel history, any stress for the child)
SIGNIFICANT TOPICS WITH SAQs 402
Examination:
i) Vitals: Pulse, RR, temperature
ii) GPE: Anaemia, jaundice, lymphadenopathy, height, weight.
iii) Abdominal Examination: Distension, tenderness, bowel sounds.
iv) DRE (if possible)
Non-Pharmacological: Diagnose if serious underlying causes have been excluded.
Educate mother for causes and reasons of constipation
Advise about balanced healthy diet like more fiber rich fruits + vegetables
Advise adequate fluid intake
Train child for proper toilet habits
Promote physical activity regularly to stimulate normal bowel function.
Sometimes get so wrapped up in play that they ignore the urge to have bowel movement,
so remind your child.
Pharmacological:
Macrogols – First-line drug laxative dose according to response.
Lactulose – if macrogols not sufficient.
Stimulant laxatives: If above not tolerated.
o Sodium picosulfate
o Bisacodyl
o Senna
Continue medication for several weeks after regular bowel habit has been established. In
addition to medication, advise about balanced diet and adequate fluid intake.
Follow up after 2 weeks
Refer for specialist assessment:
o If constipation not improved
o If one of the following serious underlying cause
▪ Delay in passing meconium since birth
▪ Abnormal appearance of anus
▪ Ribbon like stool
▪ New leg weakness, deformity/neuromuscular signs
▪ Asymmetrical gluteal muscles
▪ Abdominal distension with vomiting
SIGNIFICANT TOPICS WITH SAQs 403
234.
A 56-year-old female presents with bilateral knee pain for the last 1 year. It aggravates on exertion and
climbing stairs.
A. What risk factors would you explore in this patient for this disease?
B. What knee x-ray findings would you be expecting?
C. How would you manage this patient?
235.
A 25-year-old university student presents with 1 week history of productive cough, low grade fever,
nausea and mild shortness of breath.
On examination, there are crepitations in right lower zone with few rhonchi.
ANSWER
A
Most Likely Diagnosis:
Pneumonia
B
Investigations:
✓ FBC (increased TLC)
✓ Chest x-ray PA view (infiltrates in lung bases/cavitation
✓ Blood culture to confirm organism causing pneumonia
✓ Sputum for microscopy and culture
✓ CRP
C
Management Steps:
1. Oxygen: To maintain O2 saturation PaO2 > 8.0, saturation > 94%
2. IV fluids: To prevent hypotension in case of shock/dehydration.
3. Antibiotics:
a. Co-Amoxiclav 1.2 g/8h
b. Clarithromycin 500 mg/12H
c. If penicillin allergic use ciprofloxacin
4. Analgesia: Paracetamol 1 g/6h (if pleurisy)
5. Follow-up: At 6 weeks with chest x-ray and CRP
SIGNIFICANT TOPICS WITH SAQs 406
236.
A 10-year-old child from a poor socio-economic background attends your clinic with his mother who
says that he has some rashes between the web of his fingers, legs and elbows with a lot of itching at
night for the last two months. There are 3 other children in the family.
ANSWER
A
Likely Diagnoses:
1) Scabies
2) Contact dermatitis
B
Management:
History: Ask more questions like;
a) complete brief history, especially any skin disease history and recent infections
b) relevant aggravating & relieving factors
c) associations with fevers, rashes, boils, blisters, etc.
d) family history; any other person in family ill or likely to be having same or relevant disease
e) complete thorough examination of child along with groin examination
Investigations:
✓ Order CBC as pt belong to poor family background so might b having low Hb levels and
having any other worm infestation then seen by eosinophils. Also check wbc for any other
infection for safe side
Non-Pharmacological:
a) EDUCATION: Educate the pt. and his family his mother about the disease and its course its
spread to other family members.
b) avoid direct contact sleeping with other children of the family until get complete application of
medicine at leats 2 times in a week.
c) all close contact people other brothers and sisters need same treatment prophylacticaly
d) proper washing of clothing bedding etc
Pharmacological:
Local
o Melathion or Permethrin 5% lotion or cream. Should be applied completely on
whole body for 8 to 10 hours then wash. It’s one time measure but can be repeated
after 5 to 7 days or accordingly.
o NOTE: itching of scabies may persist until 1 to 2 week, although infection settled
down
o 25% benzyl benzoate lotion
o Crotamiton with sulphur lotion
o Crotamiton cream
Oral: Single dose of ivermectin 200 mcg/kg. May be repeated
If secondary infection, oral antibiotics.
SIGNIFICANT TOPICS WITH SAQs 407